Está en la página 1de 119

RESUMEN PSICOTERAPIAS – PRIMER PARCIAL

TEÓRICO 1
- American Psychiatric Association (2014). Introducción. En Manual diagnóstico y estadístico de
los trastornos mentales (5ª ed., pp. 5–17). Arlington, VA: American Psychiatric Publishing.
El DSM ha sido diseñado para que clínicos, pacientes, etc. puedan contar con una descripción
clara y concisa de cada trastorno mental. Sirve a los clínicos de guía para identificar los síntomas
más importantes que deberían examinarse cuando se diagnostica un trastorno. Aunque algunos
trastornos mentales pueden tener límites bien definidos, la mayoría de los cuadros están
relacionados y comparten síntomas. Entonces, los límites entre los trastornos son permeables.
El diseño de la anterior clasificación del DSM consideraba que cada diagnóstico estaba
categóricamente separado de los demás diagnósticos sin captar los síntomas que comparten
muchos trastornos. Sus categorías eran excesivamente estrechas y no reflejaban la realidad
clínica.
DSM 5: Agrupación de trastornos según factores de interiorización y exteriorización.
El DSM 5 está organizado teniendo en cuenta el desarrollo y el curso vital. Empieza con los
diagnósticos que son reflejo de procesos de desarrollo que se manifiestan en las fases iniciales
de la vida, a los que siguen los diagnósticos que se manifiestan más frecuentemente en la
adolescencia y en la juventud y finaliza con los diagnósticos relevantes de la edad adulta y la
etapa más tardía.

-Jiménez, J.P. (2017). La terapia psicoanalítica en psiquiatría. Aperturas Psicoanalíticas, 55.


Recuperado de http://www.aperturas.org/articulos.php?id=0000981&a=La-terapiapsicoanalitica-
en-psiquiatria

1- Paradigma psiquiátrico imperante a fines del siglo XIX y principios del XX. sus fundamentos.
El paradigma psiquiátrico imperante se fundamentó en que las ―enfermedades de la mente son
enfermedades del cerebro‖, modelo biomédico, anatomofuncional, que sustenta el método
lesional que surgió de la neurología.
2- Qué sostiene el paradigma freudiano (segundo paradigma imperante en la psiquiatría) y que ha
dado lugar a la Psiquiatría Dinámica.
Integra los factores causales de la dinámica inconsciente a los hallazgos modernos de
neurociencias importancia de la realidad interna, como realidad mental, en la causación de las
síntomas psiquiátricos
3- Qué postula el modelo biopsicosocial de Engel (1977).
Que hay siempre factores causales de los tres ámbitos, el biológico, el psicológico y el social. Sin
embargo, este modelo, al quedarse sólo en correlaciones estadísticas, no ha aportado un método
para estudiar las interacciones en y entre los distintos niveles de mecanismos etiopatogenéticos.
4- Qué plantea el Pluralismo explicativo. KENDLER

1
El pluralismo explicativo es preferible a los enfoques monistas, en especial al reduccionismo
biológico.
Las explicaciones de fenómenos complejos no son lineales, sino que se comportan como red.
Todo fenómeno mental acepta diversos niveles y perspectivas explicativas, desde el nivel
molecular hasta el nivel del significado de la conducta, pasando por distintos niveles funcionales
anatómicos cerebrales. Es importante el juicio clínico para elegir el nivel explicativo adecuado y
pertinente al caso a resolver.
Las últimas décadas han visto en psiquiatría un fuerte reduccionismo en la llamada ―psiquiatría
biológica‖ que en su momento de máxima infatuación postuló que el único enfoque válido para
entender los trastornos psiquiátricos y el funcionamiento psicológico es el nivel de los procesos
neurobiológicos básicos, y otros enfoques como el psa eran rechazados. El paradigma del
pluralismo explicativo plantea la necesidad de considerar múltiples perspectivas –mutuamente
informativas– para abordar los fenómenos mentales/cerebrales.
Sus planteos:
1. Las disciplinas de la mente están irrevocablemente fundadas en experiencias mentales en
primera persona (primacía de la subjetividad).
2. El dualismo y el epifenomenalismo son falsos.
3. La causalidad es de cerebro a mente (―bottom-up‖) y de mente a cerebro (―top- down‖).
4. Los trastornos psiquiátricos son etiológicamente complejos.
5. El pluralismo explicativo es preferible a los enfoques monistas, en especial al reduccionismo
biológico.
6. Nos debemos mover de la ―batalla de paradigmas‖ a un enfoque que asuma la complejidad
de los fenómenos mentales.
7. Por el momento hay que aceptar reduccionismos parciales con la meta de alcanzar
integraciones graduales.
5- OPD(Diagnóstico Operacionalizado Psicodinámico) : cuáles son las 4 dimensiones de
evaluación estructural que distingue
En el OPD estas dimensiones están bien operacionalizadas de modo que, después de un
entrenamiento, no es difícil evaluarlas y confeccionar un ‗perfil‘ estructural de los pacientes.Son:
1) capacidades cognitivas, percepción de sí mismo y del objeto: capacidad para reflexionar sobre
su imagen y la del objeto, diferenciándose del mismo
2) capacidad de autorregulación y regulación del objeto: distanciarse de los impulsos regulando la
autoestima
3) capacidad emocional de comunicación: hacia adentro y hacia afuera: diálogos internos, dejar
que surjan emociones dentro de uno: vivencia de corporeidad propio e intercambio emocional
entre el self y el otro
4) capacidad de establecer vínculos con los objetos internos y externos: capacidad de
relacionarse con los otros intrapsiquicamente o en el contacto interpersonal
6- A qué se denominan factores comunes y factores específicos en psicoterapias.

2
Los factores comunes atraviesan todas las formas y orientaciones en psicoterapia y los segundos
dependen de rasgos propios de las distintas orientaciones.
7- Incidencia de los factores comunes en el resultado psicoterapéutico
La incidencia de los factores comunes es del 30% en la mejoría de los pacientes (catarsis,
relación positiva, alianza terapéutica, experiencia emocional correctiva,etc.) (Lambert, 2013).
8- A qué se llama Revolución de las neurociencias. Principios de la Epigenética.
A un nuevo marco, integrativo, que considera los trastornos mentales como fenómenos complejos
(Morin 2009), surge a fines del siglo XX con lo que podríamos llamar la revolución de las
neurociencias, representada, entre otros, por el ganador del premio Nobel (2000) Eric Kandel.
integra factores psicosociales en los mecanismos patogenéticos cerebrales. Con ello, Kandel
abrió el camino de la epigenética y, de paso, a la investigación de los mecanismos de acción
molecular de la psicoterapia.
9- Investigación en psicoterapia psicodinámica. ¿Cuáles son los hallazgos principales?
1) tres cuartos de los pacientes tratados con psicoterapia están mejor que aquellos no tratados y
librados a la remisión espontánea
2) El tamaño del efecto de la psicoterapia es mayor que la mayoría de los tratamientos en
medicina y casi igual al de la medicación psicoactiva.
3) Hay evidencia del efecto positivo de psicoterapia en seis tipos de trastornos: Trastorno
depresivo mayor, Trastorno de pánico, Trastorno de ansiedad generalizado, Trastorno obsesivo
compulsivo, Bulimia y Trastorno de estrés post traumático. Además,
las buenas cifras sólo se aplican a los pacientes que completan el tratamiento
-Fonagy, P. Roth, A. y Higgitt, A. (2007). Psicoterapias psicodinámicas: práctica basada en la
evidencia y sabiduría clínica. (Trad. M. González Bas). Aperturas Psicoanalíticas, 27 (Original en
inglés, 2005). Recuperado de http://www.aperturas.org/articulos.php?id=61 Sue, D;
Para esta semana leer desde Evidencia para la práctica basada en la evidencia hasta el subtítulo
Depresión mayor.
1- Qué se entiende por terapia basada en la evidencia.
La categorización actual en psicoterapias basadas en la evidencia combina dos grupos de
tratamientos radicalmente diferentes: aquellos que han sido adecuadamente testados y han
resultado inefectivos para un grupo cliente, y aquellos que no han sido testados en absoluto. Es
importante hacer esta distinción, puesto que el que un tratamiento no haya sido sometido al
escrutinio empírico puede tener poco que ver con su probable efectividad
Los criterios que se utilizan para determinar lo que cuenta como práctica basada en la evidencia
deben ser a su vez testados empíricamente. Su especificidad (la probabilidad de identificar
falsamente un tratamiento como efectivo) y sensibilidad (la oportunidad de clasificar por error un
tratamiento efectivo como no efectivo) deberían establecerse
frente a una variada gama de otros criterios de salud pública.
2- A qué se refieren cuando hablan de ausencia de evidencia.

3
-―la ausencia de evidencia para un tratamiento psicoanalítico no debe confundirse con la
evidencia de que sea inefectivo.‖ Jeremy Holmes (2002)
3- Cuál es el estado actual de la investigaciones psicodinámicas
La categorización actual en psicoterapias basadas en la evidencia combina dos grupos de
tratamientos radicalmente diferentes: aquellos que han sido adecuadamente testados y han
resultado inefectivos para un grupo cliente, y aquellos que no han sido testados en absoluto. Es
importante hacer esta distinción, puesto que el que un tratamiento no haya sido sometido al
escrutinio empírico puede tener poco que ver con su probable efectividad.
La práctica basada en la evidencia necesita mirar más allá de la base de datos actual y fijarse en
la evidencia basada en la práctica para establecer abarcativamente una práctica basada en
evidencia.
4- Qué dificultades se encuentran en la investigación en psicoterapias. Controversias entre
la investigación ideal de resultados y la posible.
Alan Kazdin (1998) propuso el programa ideal de investigación de resultados pero requeriría que
repensemos todo nuestro enfoque acerca de los estudios de resultados y la práctica basada en la
evidencia. Sugiere que la investigación del tratamiento debería comenzar con la identificación de
las disfunciones claves sociales con un trastorno y la demostración empírica de estas
disfunciones en una proporción considerable del grupo clínico. también debe establecerse un
vínculo conceptual entre un método de
tratamiento propuesto y el mecanismo disfuncional del que se tiene la hipótesis que apunta El
trastorno. los estudios del proceso resultado puede ser entonces implementados para establecer
componentes claves del tratamiento y la duración necesaria del mismo. es necesario establecer
las condiciones límites para el tratamiento en términos de características del paciente y el entorno
para favorecer o debilitar la efectividad de la terapia.
5- A qué se denomina pruebas pragmáticas.
La respuesta a la controversia entre los estudios de eficacia y efectividad de la psicoterapia puede
residir en las pruebas denominadas pragmáticas o de "mundo real". Estás pruebas de mínimo
esfuerzo requieren experimentación además de la medición continuada de los resultados. El
componente experimental de las pruebas pragmáticas incluye la alteración de distintos métodos
de cuidado.
Las pruebas pragmáticas podrían ser una línea adicional clave de información para la práctica
basada en la evidencia. El establecimiento y el apoyo de una metodología interprofesional para
las pruebas pragmáticas debería considerarse una tarea adicional importante de las iniciativas de
psicoterapia con práctica basada en la evidencia.
 Fonagy, P. Roth, A. y Higgitt, A. (2007). Psicoterapias psicodinámicas: práctica basada
en la evidencia y sabiduría clínica. Para esta semana leer desde Evidencia para la práctica
basada en la evidencia hasta el subtítulo Depresión mayor.

1- Qué se entiende por terapia basada en la evidencia.

4
La Psicoterapia basada en la evidencia, llamada así en los ‘90 y 2000, en la actualidad
psicoterapias con apoyo empírico, tienen como objetivo validar tratamientos específicos para
cuadros clínicos.
Se refiere a la aplicación de un tratamiento psicológico teniendo en cuenta los resultados de la
investigación científica que ha demostrado qué es más eficaz para qué problema. El concepto
integra y combina los planteamientos y resultados obtenidos en la investigación empírica con el
juicio, la habilidad y destrezas clínicas.
2- A qué se refieren cuando hablan de ausencia de evidencia.

Jeremy Holmes (2002) ha sostenido que la ausencia de evidencia para un tratamiento


psicoanalítico no debe confundirse con la evidencia de que sea inefectivo.
Escasa validez externa: (1) la escasa representatividad de los profesionales de la salud que
participan; (2) la poca representatividad de los participantes seleccionados para maximizar la
homogeneidad; (3) el posible uso de tratamientos atípicos indicados para un único trastorno; y (4)
limitar la
medición de los resultados al síntoma foco del estudio y que resulta fácilmente mesurable pero de
dimensiones prácticamente irrelevantes.
3- Cuál es el estado actual de la investigaciones psicodinámicas.
4- Qué dificultades se encuentran en la investigación en psicoterapias. Controversias entre la
investigación ideal de resultados y la posible.
Investigación ideal de resultados: sugiere que la investigación del tratamiento debería comenzar
con la identificación de las disfunciones claves asociadas con un trastorno y la demostración
empírica de estas disfunciones en una proporción considerable del grupo clínico.
5- A qué se denomina pruebas pragmáticas.
Son pruebas de mínimo esfuerzo que requieren experimentación además de la medición
continuada de los resultados. El componente experimental de las pruebas pragmáticas
incluye la alternación de distintos métodos de cuidado.
Las pruebas pragmáticas podrían ser una línea adicional clave de información para la práctica
basada en la evidencia.
-Sue,D W y Sue, St (2010). Conducta anormal. En Psicopatología. Comprendiendo la conducta
anormal. (pp 02-15). México: Cengage Learning

Al parecer, la psicopatología o conducta anormal, no es el resultado de una sola causa, sino una
interacción de muchos factores. Casi todos los trastornos mentales surgen de contribuciones múltiples.

Inquietudes de la psicología anormal


La psicología anormal es el estudio científico cuyos objetivos son describir, explicar, predecir y controlar
conductas que se consideran extrañas o inusuales. Su materia de estudio va desde lo extraño y
espectacular hasta lo más común..

5
psicopatología: término que los psicólogos clínicos utilizan como sinónimo de conducta anormal.
psicología anormal: estudio científico cuyos objetivos son describir, explicar, predecir y controlar
conductas que son consideradas extrañas o inusuales.
psicodiagnóstico: intento por describir, evaluar y esbozar de manera sistemática inferencias acerca del
trastorno psicológico de un individuo

Control de la conducta anormal


La conducta anormal puede controlarse con terapia, que es un programa de intervención sistemática cuyo
propósito es modificar el estado conductual, afectivo (emocional) y cognitivo del paciente.

Determinación de la anormalidad
El DSM-IV, que se usa de manera más amplia, define la conducta anormal como
un síndrome o patrón conductual o psicológico clínicamente significativo que ocurre en un individuo y está
asociado con una angustia presente (p. e., un síntoma doloroso) o incapacidad (p. e., deterioro en una o
más áreas importantes del funcionamiento) o con un riesgo significativamente elevado de sufrir muerte,
dolor, incapacidad o una pérdida importante de la libertad.

A pesar de los problemas para definir conducta anormal, los practicantes tienden a estar de acuerdo con
que representa una conducta que surge de alguna norma y que daña al individuo afectado o a otros. Casi
todas las definiciones de conducta anormal usan alguna forma de promedio estadístico para medir
desviaciones de estándares normativos. Cuatro Formas principales de juzgar la psicopatología incluyen:
angustia, desviación (rareza), disfunción (ineficiencia en los dominios conductual, afectivo y cognitivo) y
peligrosidad.

conducta anormal: una conducta clínicamente significativa o el síndrome o patrón psicológico de un


individuo que se asocia con angustia presente (p. e., un síntoma doloroso), incapacidad (p. e., deterioro en
una o más áreas importantes del funcionamiento) o con un riesgo muy elevado de sufrir muerte, dolor,
discapacidad o una pérdida importante de la libertad

Angustia: Diversas reacciones físicas se derivan de fuertes componentes psicológicos; entre ellos están
los trastornos como el asma, la hipertensión y las úlceras, así como síntomas físicos entre los que se
encuentran la fatiga, la náusea, el dolor y las palpitaciones cardiacas. La angustia también puede
manifestarse en reacciones emocionales extremas o prolongadas, de las que la ansiedad y la depresión
son las más prevalentes y comunes. Por supuesto, es normal que una persona se sienta deprimida
después de haber sufrido una pérdida o decepción. Pero si la reacción es tan intensa, exagerada y
prolongada que interfiere con la capacidad del individuo para funcionar en forma adecuada, debería
considerarse anormal.

Desviación: La anormalidad, por lo tanto, se define en términos de aquellas conductas que ocurren con
menor frecuencia. La conducta extraña o inusual es una desviación anormal de un estándar aceptado de
conducta (como un acto antisocial) o una falsa percepción de la realidad (una alucinación o delirio). Este
criterio puede ser subjetivo en extremo; depende del individuo que se somete al diagnóstico, de la persona
que lo realiza y del contexto cultural particular.

Disfunción
En la vida diaria, se espera que las personas cumplan varios roles. Los problemas emocionales a veces
interfieren con el desempeño de esos roles, y la disfunción de rol resultante puede usarse como un
indicador de anormalidad. Por lo tanto, una forma de evaluar la disfunción es comparar el desempeño del
individuo con los requerimientos del rol.

6
Las pruebas de capacidades específicas e inteligencia son intentos para evaluar el potencial y predecir el
desempeño en escuelas o trabajos

Peligrosidad
Predecir la peligrosidad de los pacientes hacia sí mismos y hacia otros . los terapeutas tienen la
responsabilidad de evaluar la peligrosidad de los pacientes (para sí mismos y para otros) y de proteger a la
víctima en cuestión, los psicólogos han tratado de diseñar procedimientos para evaluar el riesgo y
establecer cuáles acciones debe efectuar un terapeuta para cumplir con el “deber de proteger”.

Limitaciones contextuales y culturales en la definición de la conducta anormal


Casi todos los criterios que se usan para definir la anormalidad utilizan una desviación estadística de algún
estándar normativo. Sin embargo, hacer esto presenta muchos problemas. Un problema que se ha
mencionado antes, es que el criterio estadístico es estático y falla al tomar en cuenta las diferencias entre
lugares, tiempo y estándares comunitarios. Otro es que el criterio estadístico no provee ninguna base para
distinguir entre las desviaciones deseables y las indeseables de una norma.

La frecuencia y la carga de los trastornos mentales - La prevalencia de un trastorno indica el porcentaje


de personas en una población que lo padecen en un periodo dado; prevalencia de por vida se refiere a la
proporción total de personas en una población que ha padecido un trastorno en el transcurso de sus vidas.
Incidencia se refiere al inicio o la ocurrencia de un trastorno dado en algún periodo. A partir de esta
información podemos averiguar qué tan frecuente o infrecuentemente ocurren varios trastornos en una
población. También podemos considerar cómo la prevalencia varía por etnicidad, género y edad, y si las
prácticas actuales de salud mental son suficientes y eficaces.

PRÁCTICO 1
Fiorini, H. (1987). La primera entrevista en psicoterapia. En Teoría y técnica en psicoterapias (pp.
63-80). Buenos Aires: Nueva Visión.
1- Importancia de la primera entrevista en psicoterapias. Qué funciones debería cumplir.
El manejo que haga el terapeuta de la primera entrevista puede tener una influencia decisiva en la
continuidad o abandono del tratamiento y, de ser mantenido, en la eficacia que alcance el proceso
terapéutico.
Esta entrevista no está destinada a cumplir sólo funciones diagnósticas y de fijación de un
contrato, sino que en psicoterapias jugará siempre además un rol terapéutico. Se trata entonces
que su acción terapéutica no se ejerza meramente por el efecto placebo del contacto inicial, sino
que el terapeuta pueda además realizar intervenciones adecuadas potenciadoras de ese efecto.
Es importante crear de entrada una alianza sólida para poner en marcha el proceso terapéutico.
Entonces una entrevista en psicoterapias debe estructurarse definidamente, con rasgos propios
que la distinguen de otros tipos de contacto inicial. Para que pueda ser eficaz debe cumplir en
fases sucesivas varias tareas.
El orden que llevan las fases sigue una progresión lógica pero no puede ser rígida, ni
unidireccional, ya que la discusión de cada punto lleva a reajustar con nuevas precisiones los
anteriores.
El tiempo que deba ocupar el terapeuta en este proceso puede variar según la experiencia, el
grado de formación y el tipo de paciente. Lo esencial no es el tiempo sino el proceso que debe

7
jugarse entre ambos a través de esas etapas, los objetivos a cumplir antes de poner en marcha el
tratamiento.
2- Fases o momentos de la primera entrevista.
1-El diagnóstico: Es función de una primera entrevista establecer el diagnóstico del paciente en
tres planos fundamentales:
a) Clínico y psicodinámico: La búsqueda inicial puede orientarse a una recolección selectiva de
datos referentes a:
1- Síntomas principales que motivan la consulta, tiempo de evolución y circunstancias de
variación, supresión o agravamiento.
2- Grupo familiar del paciente, estructura, jalones en su evolución, enfermedades importantes,
clima emocional, roles.
3- Relación éxito fracaso en la conducta del paciente referida a diversas áreas adaptativas y con
perspectiva evolutiva. Maduración, juegos, estudio o trabajo, sexualidad, sociabilidad, autoestima.
(Equivale a la evaluación del yo)
4- Aspectos interaccionales de la conducta del paciente en la entrevista (Modalidades de la
comunicación, datos transferenciales y contratransferenciales, identificación de estructuras de
conducta predominantes).
La búsqueda de datos debe ser necesariamente selectiva, guiada por la actividad del análisis y la
síntesis constante del terapeuta dirigida hacia la construcción de un modelo comprensivo
preliminar global, etiopatogénico, clínico y psicodinámico.
El paciente experimenta siempre la presencia o ausencia de esta actividad orientadora con una
clara alternativa: Que está frente a alguien que sabe a dónde va, o la sensación opuesta de
malestar, incertidumbre y confusión. Una intervención útil del terapeuta destinada a evitar la
incertidumbre inicial y sus riesgos consiste en comenzar encuadrando esta primera entrevista,
aclarando el sentido de la misma y los pasos que habrán de recorrerse para lograr sus objetivos.
b) El diagnóstico de la motivación y aptitudes del paciente para la psicoterapia: Con este término
se abarcan diversos aspectos de la conducta del paciente, sus expectativas de curación, su
disposición a aceptar la psicoterapia, sus aptitudes para participar de modo activo. Algunos
autores destacan como principal los componentes inconscientes con el terapeuta, es decir, los
aspectos transferenciales y las fantasías de curación y enfermedad.
Otros subrayan el papel que juegan las aptitudes o capacidades yoicas del paciente, reforzables
en sus aspectos conscientes: Con este enfoque un paciente está bien motivado para iniciar una
psicoterapia si se pueden identificar en él:
1. El reconocimiento del carácter psicológico de sus trastornos
2. La capacidad de introspección y su disposición a transmitir con honestidad lo que pueda
reconocer de sí mismo.
3. El deseo de comprender, la actitud de participación activa en la búsqueda.
4. La disposición a experimentar, a ensayar cambios.
5. La esperanza de que el tratamiento logre resultados positivos.

8
6. La disposición a realizar ciertos sacrificios para acceder a esos logros.

Si carece de esto deberá destacarse la indicación de técnica de esclarecimiento. Podrá


beneficiarse en cambio con una psicoterapia de apoyo, directiva, supresiva de síntomas.
Lo que se busca evaluar en suma es la aptitud del paciente para establecer un compromiso de
trabajo y extraer provecho de la experiencia terapéutica.
Ambas series de parámetros (Conscientes e inconscientes) requieren ser tomadas en
consideración como indicadores pronósticos de la respuesta al tratamiento.
Ahora, el autor plantea que los criterios listados en el enfoque ―yoico‖ de la motivación son
suficientes para pronosticar al paciente apto, porque si tales conductas están presentes, llevan
consigo implícitamente fenómenos transferenciales y fantasías de enfermedad y curación
compatibles con el aprovechamiento de la experiencia terapéutica.
Esto no es suficiente a la hora de comprender todo los planos de dificultades de los pacientes
problemáticos. Con ellos es con quienes más hay que recordar que el diagnóstico de esa
motivación no puede ser estático, sino que debe ser puesto a prueba
como fenómeno interaccional, con todo lo que el terapeuta puede hacer para estimular su
motivación.
La motivación es un dato importante para la elección final del tratamiento.
c) El diagnóstico de las condiciones de vida del paciente: Las condiciones que se vinculan
directamente con la posibilidad de que el paciente inicie y pueda mantener con regularidad un
tratamiento que exige en general esfuerzos. (Sopesar geografía, horarios, situación económica,
lugar de residencia, obligaciones familiares, etc). Todos los puntos se ponen a considerar con los
demás planos del diagnóstico.
Además ayuda a identificar los factores patogénicos en esas condiciones de vida, así como los
recursos del medio que puedan contribuir a la curación.
2- La información que devuelve inicialmente el terapéuta:
También es esencial para el paciente saber que piensa el terapeuta de los datos de la
enfermedad, de los trastornos del paciente y las expectativas en cuanto al tratamiento. Es un
segundo momento que pasa por el suministro de información que el terapeuta pueda ofrecer.
Es una reciprocidad de derechos que hace distinta a la primera entrevista. Dentro del desnivel de
roles insalvable, esta entrevista necesita funcionar con la mayor simetría posible. En esta fase de
la entrevista corresponde al terapeuta ofrecer al paciente una imagen global, introductoria, pero lo
más precisa posible, acerca del diagnóstico (Dinámico) en primer lugar, y el pronóstico ligado a
una perspectiva de tratamiento. El diagnóstico se expresa en la formulación interpretativa de una
conflictiva nuclear, unificadora de los problemas relatados por el paciente. Pero ligada a esa
formulación global, se requiere el trazado de una perspectiva terapéutica, que sugiera tipo de
tratamiento, tiempo de duración aproximado y objetivos que se propondrían para el mismo.
3- Confrontación entre las expectativas del paciente y la perspectiva del terapeuta. Reajustes y
búsquedas de acuerdos: En un momento previo a la proposición de cualquier tratamiento es
necesario un diálogo abierto entre ambos acerca de las mutuas expectativas. Se trata de despejar

9
el campo de confusiones, ambigüedades y desacuerdos implícitos, todos factores de interferencia
para un eficaz cumplimiento del contrato y para el funcionamiento del proceso terapéutico.

El terapeuta debe alentar al paciente a que cuestione, plantee dudas ya que el problema no sólo
consiste en ver qué necesita el paciente sino en considerar, además, qué está dispuesto él a
hacer, cuáles son sus disposiciones y sus dificultades para tratarse. Aquí se decide la
consolidación de una alianza terapéutica.
Mientras el tratamiento no satisfaga ciertas expectativas del paciente el riesgo de abandono es
inminente.
Entonces, la clarificación de expectativas no sólo permite consolidar el vínculo; cumple además
una función terapéutica específica: una determinada imagen de futuro pasa a incluirse
activamente en el presente de la tarea. La clarificación de las expectativas se vincula con mejores
resultados.
Es importante que a la apertura a las objeciones del paciente sea real y sincera; que la libertad de
decisión del paciente sea tangible y no meramente formal. Por otro lado
cualquier apresuramiento interpretativo del terapeuta podrá ser vivido como una intrusión aún no
autorizada por el paciente. Sólo mediante este proceso que logre instalar un vínculo, y hacerlo
confiable mediante la clarificación de sus sentidos y alcances, es posible para el paciente aceptar
la proposición concreta de un contrato terapéutico de un modo más sólido.
4- Proposición de un contrato terapéutico: Los aspectos de detalles sobre el tratamiento a iniciar
pueden ocupar un breve lapso.
Comprender especificaciones sobre honorarios, eventualmente honorarios y duración de
tratamiento. El momento final de la entrevista, aceptando este criterio consiste en indagar qué
información tiene el paciente sobre el funcionamiento de la relación terapéutica. Para el paciente
no informado, el terapeuta podrá anticipar someramente el carácter de la tarea y los respectivos
roles de la misma.

3- Cuál sería el papel de la interpretación en una primera entrevista.


La posibilidad de manejar la entrevista con un enfoque interpretativo enfrenta el riesgo de una
reacción negativa del paciente, quien puede sentirse invadido en exceso o descalificado en sus
ideas sobre su propia situación, antes de haber conocido y aceptado las reglas del juego de una
relación terapéutica. Hay que hacer un manejo cuidado de la interpretación. La necesidad de que
el terapeuta interprete resulta ineludible en 2 aspectos:
1- Alrededor del diagnóstico es fundamental que el terapeuta pueda ofrecer una visión
panorámica del sentido de la enfermedad, de la conflictiva central ligada al motivo de consulta.
Esta interpretación proporciona una imagen nueva de los trastornos (función esclarecedora) y a
su vez, ofrece un ejemplo de lo que puede ser el proceso terapéutico (función didáctica y
reforzadora de la motivación). Construye además el cuadro global comprensivo del que se
desprenden los objetivos y la estrategia terapéutica.

10
2- La interpretación transferencial puede desempeñar un papel decisivo frente a obstáculos que
interfieren en la posibilidad del paciente de aceptar un tratamiento y comprometerse en el contrato
terapéutico. Su función es neutralizar las ansiedades ligadas a fantasías transferenciales intensas
que pueden precipitar el abandono a breve plazo. Refuerza la motivación para el tratamiento
-Sue, D; Sue,D W y Sue, St (2010). Modelos de conducta anormal. En Psicopatología.
Comprendiendo la conducta anormal. (pp 30-36). México: Cengage Learning.
Modelos unidimensionales de los trastornos mentales
A fines de siglo XIX y principio del XX dos escuelas de pensamiento sobre las causas de los
trastornos mentales, un primer grupo teorizó que eran causados principalmente por problemas
biológicos donde el individuo perturbado muestra síntomas de enfermedad o daño físico y un
segundo grupo que creía que la conducta anormal era esencialmente psicosocial, originada en
complejidades invisibles de la mente humana o en fuerzas ambientales estresantes.
Estas dos visiones son consideradas muy simplistas porque establecen una falsa dicotomía entre
naturaleza y crianza, fallan al reconocer las influencias recíprocas y enmascaran la importancia
de reconocer la importancia de las dimensiones biológica, psicológica, social y sociocultural en el
origen de los trastornos mentales.
Uso de modelos para describir la psicopatología
Un modelo es una analogía que a menudo los científicos toman para describir un fenómeno o un
proceso que no pueden observar directamente. Los psicólogos han usado los modelos de manera
extensa para conceptualizar las causas de la conducta anormal, formular preguntas de sondeo y
determinar e interpretar información y datos relevantes. También pueden describir ciertos
síntomas externos como signos visibles de un profundo conflicto subyacente.
Los psicólogos usan diversos modelos, cada uno representa un enfoque teórico particular. En
consecuencia, tendemos a usar los términos modelo, teoría, punto de vista y perspectiva como
intercambiables. Los modelos de la psicopatología permiten organizar y dar sentido a la
complejidad de la información relacionada con los trastornos estudiados. Sin embargo, pueden
fomentar una explicación unidimensional y lineal de los trastornos mentales que limita la
capacidad de considerar otras perspectivas.
Un modelo de vías múltiples para los trastornos mentales
El modelo de vías múltiples es interactivo e integrativo. No es una teoría, sino una perspectiva de
la variedad y la complejidad de los factores que contribuyen a la formación de trastornos
mentales. Es un metamodelo, un modelo de modelo que brinda un marco organizacional para
entender numerosas causas de los trastornos mentales, la complejidad de sus componentes
interactivos y la necesidad de visualizar los trastornos a partir de un marco holístico.
El modelo opera de acuerdo con varias suposiciones; ninguna perspectiva teórica por sí sola es
adecuada para explicar la complejidad de la condición humana y el desarrollo de los trastornos
mentales, existen vías múltiples y causas para cualquier trastorno, las explicaciones acerca de la
conducta anormal deben considerar elementos biológicos, psicológicos, sociales y socioculturales
y no todas las dimensiones contribuyen por igual para cualquier trastorno. Este modelo reconoce
que los factores pueden combinarse en formas complejas y recíprocas. Es posible que las
personas expuestas a los mismos factores no desarrollen el mismo trastorno y que los individuos
expuestos a distintos factores adquieran trastornos mentales similares.

11
El modelo opera bajo cuatro dimensiones; factores biológicos (genética, anatomía del cerebro,
desequilibrios bioquímicos, funcionamiento del SNC, reactividad del SNA, entre otros), factores
psicológicos (personalidad, cognición, emociones, aprendizaje, manejo del estrés, autoestima,
autoeficacia, valores, historia del desarrollo, entre otros) factores sociales (familia, relaciones,
apoyo social, pertenencia, amor, estado civil, comunidad, entre otros) factores socioculturales
(raza, género, orientación sexual, religión, nivel socioeconómico, etnicidad, cultura, entre otros).
Estas dimensiones varían de acuerdo con la adopción de un enfoque micro o macro. En cada
dimensión, el modo que se organiza la multiplicidad de los factores para explicar la conducta
anormal depende de una perspectiva teórica en particular. De este modo, es posible que haya
diferencias dentro de una dimensión.
Los factores de las cuatro dimensiones pueden interactuar e influir unos en otros en cualquier
dirección, la interacción de los factores puede involucrar las cuatro dimensiones. También es
posible que las diferentes combinaciones de las cuatro dimensiones causen conductas
anormales. Asimismo, muchos trastornos son de naturaleza heterogénea, por lo tanto, es posible
que haya diferentes tipos o versiones de un trastorno. Finalmente, trastornos diferentes pueden
tener su origen en factores similares (ansiedad y depresión).
Encontrar las causas de la conducta anormal es complejo. Aunque el modelo de vías múltiples
ayuda a conceptualizar las complejidades, la realidad es que la investigación es insuficiente para
relacionar con precisión las diferentes dimensiones de análisis para la mayoría de los trastornos.
Pueden ayudar a predecir la ocurrencia y la no ocurrencia de los trastornos, y brindan un
entendimiento profundo de los medios para controlar los trastornos. Nuestra tarea es identificar
las conexiones en diferentes dimensiones y el significado de tales conexiones en la explicación y
control de las conductas anormales.

-Villar, H. y Valladares, L. (2016). Modelo de formulación de casos. Manuscrito no publicado,


Facultad de Psicología, Universidad de Buenos Aires, Argentina

*Formular un caso clínico es organizar la información de un paciente y de su situación de


padecimiento para poder realizar una hipótesis acerca de:
las causas, los precipitantes y los agentes de mantenimiento de los problemas psicológicos,
interpersonales y conductuales de una persona o grupo de personas.
*La información aportada en el caso clínico debe ser suficiente para permitir al profesional/alumno
que lo lee, pensar al paciente, su contexto, su psicopatologia o problematica generadora del
padecimiento y las posibles estrategias y pasos a seguir para su asistencia psicoterapéutica, es el
punto de partida para la toma de desiciones en la clinica, para poner en marcha un tratamiento
psicologico y para la supervision.
*Es menester respetar el ppio de economía de la información- solo información breve y de utilidad
clinica y psicoterapeutica
*se debe resguardar la confidencialidad del paciente

12
• Datos personales del paciente
-Nombre
-Edad
-Nacionalidad
-Ocupación/Estudios
-Grupo familiar
• Fuente de la derivación
• Contexto de la consulta (ámbito privado, público,condiciones preestablecidas para
el encuadre –cantidad de sesiones, duración, honorarios- etc).
Aclarar si se trata de una entrevista de admisión o psicoterapéutica (persiguen diferentes
objetivos).
Público Obra social / Pre-paga Privado
• Tipo de consulta (demanda espontánea, derivación, interconsulta, ámbito clínico médico,
escolar, judicial, etc.)
• Breve resumen de la historia vital del paciente: acontecimientos significativos o traumáticos,
duelos, separaciones, migraciones, enfermedades, etc.
• Momento actual del paciente: ciclo vital en el que se encuentra y tarea que tiene por resolver (a
qué se enfrenta, ej. cambio de trabajo, parentalidad, etc.)
• Nivel de malestar subjetivo. Grado de urgencia representado por el paciente.
Leve Moderado Severo
• Motivo de consulta: qué busca el paciente y qué espera del tratamiento. Identificar objetivos.
• Sistema atributivo: identificar a qué atribuye el paciente lo que le ocurre.
• Estrategias de afrontamiento (qué ha intentado el paciente para enfrentar lo que le pasa) y
evaluar factores de mantenimiento.
• Grado de disfuncionalidad del paciente (evaluado por el psicoterapeuta) a) Severidad
sintomática y urgencia clínica.
b) Complejidad (cantidad de variables involucradas, distinguir para planificar lo ideal de lo
viable).
c) Cronicidad (hace cuánto le ocurre lo que está padeciendo).
• Red social de apoyo: evaluar red de apoyo y si la misma es amplia o escasa, fuerte o débil.
• Antecedentes clínicos y terapéuticos (enfermedades anteriores o actuales y si realizó

13
tratamientos anteriormente y qué valoración hace de ellos)
• Modos de comunicación (verbal y paraverbal)
• Aspectos semiológicos significativos.
• Diagnóstico situacional y categorial (DSM IV o V a decisión de cada profesional).
• Diferenciar aspectos situacionales o de estado de aquellos rasgos de personalidad que
persisten como característica del paciente a lo largo de su vida y en la actualidad.
El alumno que trabaja con un caso clinico debe poder organizar la informacion de modo tal que
pueda asistir y pensar en el paciente para delimtar los objetivos del tratamiento, modos de lograr
esos obj. y evaluar al paciente en su situacion actual y en la alianza establecida entre el y el
terapeuta.

TEÓRICO 2
-Abello Blanco, A. (2011). Desarrollo emocional temprano. En A. Abello Blanco y A. Liberman
(Comp.), Una introducción a la obra de D. W. Winnicott. Contribuciones al pensamiento relacional
(pp. 53-82). Madrid, ES: Agora relacional.
Winnicott, su obra centrada en el desarrollo emocional del bebe. Pediatra y Psicoanalista. Origen
del funcionamiento psiquico del bebe, por medio de la conformacion del narcisismo primario.
Lo preverbal- preedipico. “His majesty, the baby”: su majestad, el bebe.
COMIENZAN LAS DIVERGENCIAS:
W., 1970: el niño viene al mundo con una potencialidad, pero puede no desarrollarla y
permanecer como algo que podria haber sido pero que no fue.
W, critica el hecho de que F no desarrollo la posibilidad de que la niñez y la temprana infancia
podrian no ser transitadas segun los parametros normales. Dio por sentado que se alcanzarian
los logros necesarios para el desarrollo temprano, para la conformación de un narcisismo primario
compatible con la salud W se va a ocupar del ambiente temprano y sus efectos facilitadores en el
desarrollo temprano del niño. El Infante y el cuidado materno (o de quien cumpla sus funciones)
forman una unidad.
W plantea que no podemos limitarnos a atender o a hacer objeto del psicoanalisis a aquellos S
cuyas madres han brindado condiciones suficientemente buenas en sus primeros años.
Segun Avergbung, W habla de dos tipos de pacientes:
-Los que tuvieron cuidados suficientes y el sufrimiento actual es derivado de clasicos conflictos.
En ellos los trastornos del desarrollo son PROFUNDOS.
-Los que sufrieron un deficit en sus cuidados tempranos (falla temprana de la funcion de la
“madre suficientemente buena”). En ellos los trastornos son TEMPRANOS DEL DESARROLLO.
Que esta funcion no haya sido llevada a cabo en la temprana edad, no quiere decir que no pueda

14
darse luego: Concepto de W de Segunda Oportunidad. En este caso lo Profundo puede originarse
en el tratamiento o en cualquier otro momento de la vida adulta.
DESARROLLO EMOCIONAL TEMPRANO
Primeros 5/6 meses de vida del bebe relevantes desde la teoria psicoanalitica (esto es
innovadorantes no lo consideraban) —> pionera:M. Klein
DE QUE TIPO DE PACIENTES HABLAMOS
Relevancia de primeros años de vida en los pacientes graves, en estos casos debemos remitirnos
a las etapas tempranas del desarrollo emocional del S para entender la etiologia de su
padecimiento.
Neuroticos, personas enteras s/ W.
Cuando las cosas “no fueron bien” en etapas tempranas nos encontramos con Angustias de otro tipo ≠ a las
Angustias de Castracion. Son angustias de caidas interminables, de “partirse en pedazos”, de escisión y
aislamiento:
EJEMPLO (RELATO) MARGARET LITTLE (DE SU ANALISIS CON W.):
“Yo tenia miedo, ese terror que solo los niños pueden experimentar, miedo a la destrucción total,
al desmembramiento de mi cuerpo, a volverme irremediablemente loca, a ser borrada,
abandonada y olvidada por todo el mundo como alguien que nunca hubiese existido, una angustia
impensable que es como experimentar una aniquilacion, una destruccion total”
Es angustia de aniquilacion. Desintegracion en el plano corporal/ gran sufrimiento. Realidad
desdibujada (comprometida). No hay sosten o algo que lo salve.
Abordaje clinico requerido para estos pacientes:
Funcion especular “un espejo mágico capaz de devolverme una imagen mejor de la que podían
ver mis ojos”
“lo implicito suele tener mas fuerza que lo explicito”
Patologias ligadas a lo preverbal, al deficit, en el ambito de lo pre edipico.
TRES PROCESOS BASICOS…
…Que empiezan pronto, siempre y cuando, “las cosas vayan bien”, que quiere decir que el bebe
se encuentra en un ambiente facilitador suficientemente bueno, cuando la madre (o quien haga
las veces) sienta esa preocupación maternal primaria
Los procesos de los que habla W, son aquellos que permitan tres logros fundamentales:
-Integracion (funcion del medio/ambiente que necesita el bebe: Sosten)
-personalizacion (funcion del medio/ambiente que necesita el bebe: Manipuleo)
-realizacion (funcion del medio/ambiente que necesita el bebe: Presentacion del Objeto)
I. LA INTEGRACION
W. plantea una No integración primaria. El niño no tiene una idea, una representación integrada,
conjunta de él mismo ni de sus estados y tampoco de los otros. Ej: el niño no sabe que él es el
mismo que ahora esta tranquilo y relajado pero que antes estaba irritado. No sabe que la madre

15
de quien recibe cuidados es la misma a quien quiso atacar con el bocado predatorio sobre el
pecho. No hay integracion de esas dos caras (y de tantas otras) que forman parte de él mismo y
de los otros. No se puede integrara a quien somos despiertos con quien somos dormidos.
Cuando no se da la Integración durante el desarrollo emocional primitivo, el S queda en estado de
disociación.
El tratamiento representará en estos casos una “Segunda Oportunidad”, en donde se presentará
en diferido y con valor simbólico algunas de las condiciones que estuvieron ausentes durante la
temprana infancia Este proceso se relaciona y hasta incluye los otros dos porque la integración
remite a la dimensión temporal, a la relación con el espacio, con el propio cuerpo y con la
realidad exterior.
Todas estas variables deben ser paulatinamente integradas en el funcionamiento psicofisico del
bebe.
Ejemplos de No Integracion Primaria:
-En relacion al tiempo: un niño de 9 años juega con otro de 2 años y se pregunta sobre el nuevo
bebe que viene en camino, nacera antes o despues que el de 2 años?
-Un paciente psicotico que no consigue armar una rutina porque no sabe si el martes fue el
martes de esta semana , de la pasada o de la proxima.
-Localizacion del ser en el propio cuerpo, es un rdo a conseguir, el logro de un proceso, no viene
incluido al nacer.
-Paciente psicotica que creía que su hna gemela que estaba acostada a su lado era ella misma,
le llamaba la atención que alguien agarrara a la niña y ella no cambiar de lugar.
-Se observa con mucha frecuencia en la clinica pacientes que relatan con lujo de detalles lo que
hicieron, el analista se frustra por no poder interpretar pero el paciente estará mas que satisfecho
porque logra por ese medio cierta integracion de su ser en la mente del otro, logra que el otro
pueda integrarlo a el como ser en su mente y asi logra sentirse asi él mismo.
Dentro de las funciones del analista:
-Proveer al paciente de las vivencias y experiencias que deberia haber recibido de su entorno y
que no recibio, por ejemplo ser pensado por alguien, vivir él mismo integrado en la mente del otro,
ésto suele darse naturalmente por la madre, quien está pendiente del desarrollo del niño, poder
acceder a quienes fuimos por medio del relato de nuestros padres.
Esto implicará que nuestras intervenciones se alejan de la tipica interpretacion caratcteristica del
psicoanalisis.
Hay dos experiencias que facilitan la integración:
-las que provienen del entorno/ambiente (cuidados infantiles, protección del frío, baño, acunarlo y
darle un nombre)y
-las que provienen del interior del S, las vivencias que emanan de la vida pulsional del bebe.
La madre será un conjunto de rasgos y a lo largo del desarrollo el bebe podrá integrar bajo un
concepto de madre

16
EL SOSTEN- HOLDING
Es el aporte del entorno en general y de la madre en particular. Winnicott plantea que esta
operacion inicia con el sosten fisico de la madre en el embarazo.
El ambiente sostiene cuando el bebe esta en estado de dependencia maxima o DEPENDENCIA
ABSOLUTA, en esta fase el bebe no puede siquiera registrar el cuidado materno, no hay registro
yo/ no -yo. Hay una fusion entre la madre y el bebe.
Sosten es proteger, cuidar la temperatura del ambiente y del bebe, cuidar la sensibilidad auditiva
y visual del bebe, impedir que el bebe se caiga, cuidar cada detalle para asegurar el bienestar del
infante.
Sosten, 1ero fisico con la contencion del feto en la matriz y luego psicologico.
La madre sostiene al bebe—> el padre sostiene a la madre y a la vez ésta es sostenida por otros,
el grupo familiar, su propia madre —> la familia es sostenida por la sociedad. Todo esto es
“cuando las cosas van bien”
II. LA PERSONALIZACION
Es HABITAR EL PPIO CUERPO, sentirlo propio, estable, confiable Este como los otros logros no
se dan de una vez y para siempre, el sosten, manipuleo y presentacion de objeto, son funciones
que siguen vigentes en la vida adulta. Por ejemplo el sostén afectivo de nuestra red social, el
placer de las caricias y el contacto corporal con otros, el manipuleo de “la cosa” en el proceso de
aprendizaje.
Contracara: despersonalizacion, no reconocernos en la imagen que nos devuelve el espejo. La
personalizacion es la integracion entre la imagen interna y la externa, reconocerme como eso que
veo de forma estable y confiable.
A veces en situaciones traumaticas esta integracion tambalea, esta integracion que nos eprmite
habitar este cuerpo se desvanece.
1er momento de NO integracion en el bebe—> su integracion y personalizacion ocurre primero
fuera de él: en la madre y es ella con sus acciones y la relacion que la une al bebe la que hará
que el bebe sea el protagonista de sus propias vivencias.
No es lo mismo recibir la imagen de nosotros en el rostro gozoso de una madre alegre, que de
una madre deprimida y triste con rostro opaco, apagado.
En los casos donde “las cosas marchan bien” se ira habitando el cuerpo y se incorporarán los
cuidados maternos como cuidado propio, amor por ese cuerpo integrado del que se siente
pertenencia y propiedad, entonces alli los cuidados maternos que venian de afuera, se
internalizan y el S se cuida a si mismo, se defiende ese cuerpo que se gestó en las interacciones
con el entorno.
MANIPULEO
Handling/ manipuleo—> Cuidados corporales, por parte del entorno facilitador + “madre
suficientemente buena”.
Cuidados corporales, placer del contacto entre la madre y el bebe, los “apapaches”, las caricias,
las miradas felices, las palabras tiernas con que la madre “baña” al bebe. Todo lo atinente a la
relacion tierna entre la madre y el bebe que SUPERAN los cuidados basicos necesarios para la

17
suervivencia, eso es lo que le devuelve al niño su propio cuerpo para que sienta que lo habita y
que es fuente y destino de placer y que goza de cierta unidad.
Relación necesaria entre manipuleo (Handling)y personalizacion.
III. LA REALIZACION
Contacto/ interacción del bebe con la realidad externa, con los objetos que la habitan. Esto
sucede una vez que la integracion se haya instalado bien en el bebe. “Momento de ilusion”:
cuando se da el “feliz encuentro” entre el bebe que anhela y la madre que ofrece el pecho.
Ademas de garantizar la supervivencia del bebe, este momento tiene gran valor simbolico y
repercute en los origenes del psiquismo.
La importancia del momento de ilusion tambien es que marca la “capacidad de ilusionarse” del
niño a futuro en sus experiencias de la vida.
Ilusion en terminos de sentir esperanza y autoconfianza—> cita de F: “Cuando un hombre ha sido
el favorito de su madre logra conservar durante toda su vida un sentimiento de vencedor, esa
confianza en el exito a menudo conduce al exito”.
Esto sucede claro, “cuando las cosas van bien”
LA PRESENTACION DEL OBJETO
El bebe con su anhelo/deseo de comer crea el pecho/lo alucina + presentacion del pecho real =
para W, esto quiere decir que el niño tiene la capacidad de evocar aquello que esta a su alcance.
El bebe crea aquello que se le presenta.
Para conseguir una buena (adecuada) relación con la realidad, esta debe se presentada al niño
de manera ininterrumpida y en fragmentos simplificados.*
Escena de golpear la mesa con la que el niño se golpea—> W dice que si no fuese por el vinculo
empatico que se establece con el niño, que es mas o menos entendido por todos, la escena es
delirante, lo cual es cierto pero es necesaria para mantener la omnipotencia primaria del niño, su
narcisismo y estimularlo a seguir intentando caminar a pesar del golpe.
Esto es parte de la presentacion del mundo, decirle al niño que “la mesa es mala”, esto se deja de
hacer una vez que el niño tiene los recursos cognitivos y motores para esquivar los “obstaculos”
al caminar.
Lamentablemente hay casos en los que esto no pasa, y en lugar de apoyar al niño en este
proceso de entender y conocer el mundo, la realidad y las dimensiones de espacio y tiempo que
la componen, se lo trata de torpe o cosas peores—> estos son los cosas donde las “cosas no van
bien”. no se ha puesto en marcha el proceso para mostrarle/presentarle el mundo/los objetos al
niño.
*La clave es la relación de niño con el entorno, su madre y/o cuidadores, que le presentan al niño
el mundo de una forma constante y comprensible, que se ajuste a las capacidades y variables del
pequeño.
Esto es la PRESENTACION DEL OBJETO s/W y es el proceso necesario para que la
REALIZACION se concrete.
-Fonagy, P. (1999). Persistencias transgeneracionales del apego: una nueva teoría. Aperturas
psicoanalíticas. N° 003, 1999. https://aperturas.php?a.org/articulorticulo=86

18
EL SISTEMA DE APEGO COMO UN MECANISMO DE REGULACION BIO-SOCIAL DE
REGULACION HOMEOSTATICA
La teoria del apego de Bowlby plante la necesidad humana universal de formar vinculos afectivos
estrechos. Reciprocidad del vinculo temprano como precondicion para el desarrollo de la
personalidad normal. Conductas de apego del niño (sonreir, busqueda de proximidad, demanda
por medio de llanto) correspondidas con conductas de apego del adulto (sostener cuidar,
alimentar, etc)- buscar y brindar cuidados, estas conductas del adulto refuerzan el apego del niño
a ese adulto en particular.
La experiencia de seguridad es el objetivo del sistema de apego-> regulador de la experiencia
emocional, por lo cual esta en el centro de muchos trastornos mentales y de la tarea terapeutica.
El infante no nace con la capacidad de regular sus propias reacciones emocionales, se va
regulando poco a poco a medida que el adulto le ofrece la atención y los cuidados necesarios
para que se reestablezca el equilibrio ante la activación de las respuestas neurovegetativas del
niño.
En estados de activacion incontrolable, el infante buscara al dador de cuidados, buscara la
proximidad fisica con la esperanza de ser calmado y recuperar la homeostasis, hacia el final del
primer año de vida, la conducta del infante tiene intencionalidad marcada, basada en sus
experiencias y expectativas.
El sistema de apego asi se configura como un sistema regulador bio-social de la regulación
homeostatica
PATRONES DE APEGO EN LA INFANCIA
Otra autora que trabajo el tema del apego es Mary Ainsworth, estudio en el laboratorio a niños
para evaluar sus pautas de apego.
Los infantes al ser separados de sus padres muestran uno de cuatro diferentes tipos de conducta:
1- Apego Seguro: Los niños exploran el entorno ante la ausencia del cuidador, se ven
perturbados por presencia de un extraño y lo evitan, son perturbados por la ausencia del cuidador
y tratan de evitarla buscando reestablecer el contacto, al ser reasegurados por el cuidador
primario, el infante retoma la exploracion.
Las conductas de los infantes seguros son producto de una relación o experiencia del vinculo con
los cuidadores bien coordinada y sensible. El cuidador es raramente sobreestimulante y es capaz
de devolver la estabilidad ante el desequilibro homeostatico emocional del niño, por lo cual no
pierden el control ante el estres. Las emociones negativas son sentidas como menos
amenazantes.
2- Apego Ansioso Evitativo: Al niño le da igual la presencia del extraño o del cuidador primario, no
se muestran ansiosos o muy poco ansiosos por la separacion del cuidador, no tienden a
reestablecer el contacto con el cuidador al ser separado de el Estas conductas de los niños
ansiosos evitativos son producto de que el cuidador no fue eficaz ante la demanda o busqueda de
cuidados que reestablecerian el equilibrio del niño o bien estos niños fueron sobreestimulados por
conductas intrusivas de los padres, con lo cual sobreregulan su afecto y se desarrollan como
“independientes” para evitar cualquier perturbacion.

19
3- Apego Ansioso Resistente: el niño explora el mundo muy limitadamente, ante la ausencia del
cuidador su nivel de ansiedad y frustración aumenta, intenta reestablecer el contacto con éste, se
ven altamente perturbados por la separacion y tienen dificultad para reponerse ante esto. Los
intentos del cuidador de calmarlo fracasan, parece que la rabia y el llanto son mas fuertes que el
haber recobrado la proximidad con el cuidador.
Estos niños subregulan (regulan de menos) sus afectos, aumentando la expresion de malestar
como intento de despertar la respuesta de sus cuidadores, el niño esta constantemente
preocupado pro recibir cuidados de sus padres, no esta seguro de poder contar con ellos para
recuperar el equilibrio homeostatico emocional en caso de necesitarlo
4- Desorientados/Desorganizados: Inmoviles o expresan el deseo de escapar aun en presencia
de los cuidadores, para estos niños los cuidadores han sido a la vez fuente de temor y de
reaseguramiento, motivo por el cual es sistema de apego pone en marcha una serie de conductas
conflictivas. Casos de abuso infantil o severa desatencion.
LA CONTINUIDAD DE LOS PATRONES DE APEGO
Bowlby planteo que los “modelos activos internos” del self y de los otros funcionan como
prototipos que regulan todos los vínculos que el S sostendrá en su vida.
Modelos relativamente estables durante el ciclo vital basados en las experiencias tempranas. El
sentimiento Autonomo y Seguro del self proviene de relaciones seguras con los padres, le permite
al niño adueñarse de su experiencia, tienen mayor capacidad de resistencia, autoconfiados y
orientados socialmente, son empaticos para el malestar.
APEGO Y CAPACIDAD DE MENTALIZAR: EL CAMBIO DESDE UNA INTERPRETACION
TELEOLOGICA HACIA UNA MENTALISTICA DE LAS RELACIONES DE APEGO EN EL
DESARROLLO
Capacidad de Mentalizacion o Funcion Reflexiva=> Es la capacidad de comprender la propia
conducta y la de los demas en terminos de estados mentales (pensamientos, sentimientos,
creencias y deseos)
-Modelo No mentalista Teleologico de la conducta: El niño no puede predecir el modelo
intencional mental del otro, la conducta no se infiere mas que por resultados visibles. Ej: si Maxi
dejo el chocolate en la caja pero el experimentador lo cambio de lugar sin que él lo sepa, segun el
niño X Maxi va a buscar el chocolate donde lo dejo el experimentador. No hay intervencion de las
creencias sobre la realidad, es lineal: lo que se observa, no hay inferencias.
Hacia los 2/3 años el niño interpreta de forma “mentalizante”, hay comprension mental del deseo,
las intenciones, es capaz de comprender que los otros pueden actuar acorde a un deseo o
intencion que es diferente al propio.
Los padres que puntuaban alto en esta capacidad reflexiva de mentalizacion de intenciones y
deseos de los demas tenian mas posibilidades de tener hijos con apego seguro a diferencia de
aquellos padres cuya capacidad reflexiva era baja-> cuanto mas pueda detectar lo que quiere,
necesita, intenta hacer el otro mejor lectura de lo que necesita voy a poder hacer y esto implica
que puedo devolverle la estabilidad emocional que vaya a necesitar con mayor inmediatez y eso
genera una pauta de apego seguro en el niño.

20
La capacidad de entender los estados mentales que tengan los padres es muy importante para
asistir a los hijos cundo estos estan en condiciones desfavorables
MENTALIZAR Y EL DESARROLLO DEL SELF
La capacidad reflexiva del padre predice la seguridad en el apego del niño y tambien adquisición
precoz de la capacidad reflexiva de este-> la capacidad de los padres de observar e interpretar la
mente de los niños facilita la comprensión general que los niños tienen de las mentes de otros
facilitada por un apego seguro. La capacidad reflexiva en el niño es facilitada por el apego seguro.
Un niño con apego seguro tiene la tranquilidad de salir a explorar las mentes de otros entre ellas
la de su cuidador—> es un proceso INTERSUBJETIVO: el niño conoce la mente de su cuidador
mietras que su cuidador intenta comprender e interpretar la mente del niño.
Es este proceso intervienen tres procesos:
1-El papel de la especularizacion (mirroring)
2-El cambio hacia la interpretación de la conducta del cuidador en términos intencionales mas que
teológicos.
3-La integracion de una forma primitiva dual de realidad psiquica en una representación
mentalizante unica de la mente
1- EL PAPEL DE LA ESPECULACION
Las representaciones simbolicas de estados mentales se desarrollan en un contexto de
relaciones de apego. La representacion que tiene la madre del afecto del infante es representada
por el niño especularmnete y asi la incluye en su propio sistema de representacion, ahora forma
parte de su self.
La representacion internalizada del afecto segun la lectura de la madre empatica, se convierte en
una representacion de la experiencia del niño.
Es un peligro cuando el cuidador exagera las reacciones afectivas del infante, así este las codifica
e internaliza como algo mas fuerte de lo que en verdad se siente. El intercambio de afecto entre el
niño pequeño y el cuidador es una especial fuente de información sobre sus estados internos.
El niño que busca regular su malestar encuentra en la respuesta de su cuidador una
representacion de su estado mental que el puede internalizar y usar como fuente de respuesta o
estrategia para autoregularse en el futuro.
La combinacion entre la representacion de la experiencia del self y la representacion de la
reaccion del cuidador cambian el modo teleologico de la mente del niño y le permite interpretar y
comprender reacciones afectivas propias y de los otros asi como lograr el control o regulacion de
sus ppias emociones.
2. PARENTALIZACION REFLEXIVA Y EL CAMBIO DESDE MODELOS MENTALES
TELEOLOGICOS A MODELOS INTENCIONALES
El cuidado sensible de los padres se basa en poder observar y leer cambios en el estado mental
del niño, lo cual sucede en el apego seguro-> esto a su vez configura la base psico social para
que el niño pueda leer su ppio estado mental/comprender la mente y el de los demas via la
internalizacion de la respuesta intencional mentalizada de los padres sensibles.

21
El niño con apego seguro se siente tranquilo, comodo al hacer atribuciones a los estados
mentales del otro para dar cuenta de sus conductas.
El niño con apego elusivo se sustrae de los estados mentales de su cuidador, del otro. El niño con
apego resistente esta demasiado centrado en su propio estado de malestar. El niño con apego
desorganizado/desorientados sabe perfectamente leer los estados intencionales de sus
progenitores, estan demasiado orientados a interpretar las intenciones de sus cuidadores, es
como que estan alerta todo el tiempo y son buenos para mentalizar los estados afectivos de los
demas.
3.EL CAMBIO DESDE LA REALIDAD PSIQUICA DUAL A LA SINGULAR- UNICA
Capacidad de mentalizar, entender la realidad interna diferenciada de la realidad externa aunque
relacionadas, esto se logra en asistencia de un otro que le presta las representaciones y los
estados mentales a internalizar.
El “como si” tambien es transmitido al niño por un adulto o cuidador, “como si” diferenciado de la
realidad objetiva, jugar a ser un policia no es ser un policia. En este movimeinto el adulto implica
el
mundo interno del niño haciendo como si pero a la vez manteniendo la realidad externa
CONCLUSIONES
Nuestra capacidad reflexiva es una adquisicion transgeneracional, adquirida de nuestros padres
que nos ven como pensantes, nos atribuyen intenciones y es asi como el self propio se constituye
como capaz de mentalizar estados emocionales e intenciones, deseos y creencias en mi mismo y
en otros.
-Marrone, M. (2001). La teoría del apego. En Autor (Ed.), La teoría del apego. Un enfoque actual
(pp. 31-47). Madrid, ES: Psimática.
INTRODUCCION
Bowlby en 1977:
Teoria del Apego, para explicar:
-tendencia de los seres humanos a crear fuertes y duraderos lazos afectivos con ciertas personas
en particular
-diferentes formas de dolor emocional y trastornos de la personalidad como la ansiedad, la ira, la
depresión, el alejamiento emocional como consecuencia de la separacion indeseada y de la
perdida afectiva
Muchas patologias o trastornos psiquiatricos pueden ser atribuidas a las alteraciones en el
desarrollo de la conducta de apego o al fracaso en el desarrollo de la misma.
UN PARADIGMA PSICOANALITICO
El psicoanálisis se ocupa del estudio de la VIDA EMOCIONAL y de las RELACIONES
HUMANAS, sin embargo con los años otras cuestiones como la sexualidad y la pulsion de muerte
tomaron mas protagonismo que el estudio de las relaciones primarias de apego.
-Las emociones fuertes asociadas a la alegria o al dolor emergen de las relaciones de apego

22
-Las relaciones tempranas determinan el desarrollo de la personalidad y de la salud mental
-La forma en que las personas interpretan y manejan las relaciones esta basada en experiencias
de relaciones previas.
Es importante reconocer el APEGO como UNA FUERZA MOTIVACIONAL PRIMARIA para así
luego entender las consecuencias que de la relacion de Apego se derivan
B. planteo la necesidad de crear un nuevo marco teorico que incluya los conceptos freudianos y a
la misma vez, importancia de la vida interpersonal no como resultado de procesos psicológicos
generados internamente sino con su propio papel protagonico.
Para Bowlby, la necesidad de formar y mantener relaciones de apego es PRIMARIA y esta
claramente diferenciada de la necesidad de alimentacion y de la necesidad sexual. La conducta
de apego tiene como funcion, garantizar la supervivencia, el individuo tiene mas chances de
sobrevivir, si en condiciones adversas es asistido por otro ser humano, especialmente si este es
mas sabio o mas experimentado
B en su desarrollo teorico se basa y da importancia a la teoria del Aferramiento al Objeto Primario,
que postula que existe en todo ser humano una necesidad constitucional de aferrarse y tocar a
otro ser humano, es una necesidad tan importante como la necesidad de alimentacion y calor.
Diferencias entre el paradigma freudiano y el paradigma de Bowlby:
-Para F el apego es secundario con respecto a las gratificaciones oral y sexual (necesidades
primarias), en el modelo de Bowlby, el apego es primario y tiene status propio.
-Para F el niño esta “encerrado” en su narcisismo primario, aislado de los estímulos del mundo
exterior, en cambio en el modelo de Bowlby, el niño esta activamente interactuando con el
contexto intersubjetivo desde el momento cero.
-Para F la conducta pulsional se activa cuando por un aumento en la carga de energia se llega a determinado
nivel que requiere una descarga a ≠ de Bowlby para quien la conducta pulsional es activada cuando la
funcion que esta cumple es requerida, esta activacion se realiza interna y externamente.
SISTEMAS CONDUCTUALES Y MOTIVACION
Tesis de Bowlby—> Sistemas conductuales (o motivacionales) para satisfacer y regular
necesidades Basicas:
-Sistema de Apego: Busqueda de proteccion por personas especificas
-Sistema de afiliacion a grupos
-Sistema de alimentacion
-Sistema sexual
-Sistema Exploratorio
Los sistemas de Apego y Afilacion estan relacionados con la necesidad de SENTIRSE
PROTEGIDO
El de Exploracion con RECONOCER EL AMBIENTE, APRENDER Y ADQUIRIR NUEVAS
EXPERIENCIAS, AFIANZAR LA IDENTIDAD COMO SER AUTONOMO E INDEPENDIENTE.

23
Desde el pto de vista de la teoria de Bleichmar de los sistemas motivacionales modulares, el
Apego sirve a los Sistemas Narcisista (destinado a mantener el autoestima, la autoafirmacion y el
sentido subjetivo del Self como una entidad cohesionada) y al de Hetero-Autoconservacion
(destinado a garantizar la supervicencia psico-corporea de uno mismo y del otro)
SISTEMA—> es un conjunto de Respuestas/ Conductas activadas para resolver determinadas
necesidades.
Cada sistema puede activarse ante estímulos internos o externos. Algunos sistemas pueden
activarse al mismo tiempo por ejemplo el de Apego y el Sexual en una Relacion de Pareja pero
por ejemplo el sistema de Apego y el Exploratorio son mutuamente excluyentes:
Ejemplo: Un niño jugando en parque (bajo la activacion de su sist exploratorio) se lastima, activa
su sistema de apego y busca a la madre en busqueda de proteccion y cuidado—> debido al
accidente se desactiva el sistema exploratorio y se activa el de apego. La conducta del niño se
orienta a metas.
La conducta de apego podria desglosarce en dos momentos: mantener una relacion con una
distancia determinada y en otro momento recurrir a una proximidad mayor ante situaciones
adversas. Por ejemplo en un adulto seria mantener la relación de apego con visitas regulares,
llamadas, etc y buscar una proximidad mayor cuando algo sucede y se recurre a la figura de
apego repentinamente en busqueda de proteccion y cuidado.
La representacion de la relacion de apego se genera con una persona especifica, esta
representacion es DURADERA, se mantiene aun cuando la relacion haya terminado debido a los
componentes emocionales.
La relacion de apego mas importante es la parento-filial por la importancia que ejerce en el
desarrollo temprano de la personalidad pero hay otras relaciones de apego a lo largo de la vida
por ejemplo con hermanos, amigos y pareja. Estas relaciones se organizan jerarquicamente de
acuerdo a la importancia y valoracion que el S les de a cada una de ellas por la influencia que
tienen en su vida emocional.
ETOLOGIA * Estudio científico del comportamiento humano y animal
B se interesó por la etologia porque descubrio que a través de ella podria encontrar respuestas
sobre la conducta pulsional.
Investigacion con Monos:
Los Harlow realizaron una investigacion con monos, se proveia un mono-madre de alambre con
un sistema que permitia alimentar a las crias y otro mono-madre sin este sistema para alimentar
pero recubierto de felpa. Los monitos recurrian al mono de felpa, acurrucandose y abrazandose a
el cuando estaban asustados y usandolo como base para las exploraciones. Recurrian al mono
de alambre solo cuando tenian hambre. Este experimnto pone en tela de juicio la teoria de que la
alimentacion y la oralidad son las que originan la conducta de apego ya que para los monos al
menos era mas importante el contacto calido a menos que tuviesen hambre.
El contacto fisico intimo es muy importante para el desarrollo del sentimiento de seguridad. Los
experimeintos demostarron que la relacion madre-hijo con los primates es duradera y su
alteracion causa dolor psiquico y lo puede dañar seriamente si esta alteracion se prolonga. Como
se manifiesta la conducta de apego en el pequeño?:

24
-Intentos de busqueda de proximidad como el chupeteo, el aferramiento, imitacion
-Conducta visual y locomotroz de seguimiento
B utilizó ciertos aspectos de la relación entre progenitores y sus crias que eran comunes en varias
especies y penso que seria probable que tambien se suceda entre los seres humanos, marco
cierta continuidad entre la conducta animal y la humana
LA TEORIA DEL APEGO COMO…
A) UN MODELO EVOLUTIVO
La teoria de apego enfatiza:
1- La importancia de los vinculos intimos y emocionales entre los individuos (particularmente del
niño con sus figuras paternas/ cuidadores pero tambien las relaciones con otros a lo largo del
ciclo vital)
2- La poderosa influencia que tienen las relaciones de apego en el desarrollo de la personalidad y
en algunos casos, en el origen de la psicopatologia. Cuando el niño nace tiene un amplio abanico
de vias posibles, “vias evolutivas”, la elección del camino estará determinada por la interacción
del individuo con el ambiente.
La patologia implica que el camino elegido fue por debajo del desarrollo optimo en la niñez o en la
adolescencia. Sin embargo el cambio hacia una via mas optima puede ocurrir en cualquier punto
del desarrollo dependiendo de cambios en las relaciones signifcativas del S. Cuanto mas tiempo
se haya demorado en el “camino desviado” menos chances de un desarrollo cercano a lo optimo.
B)LA RESPUESTA SENSIBLE COMO ORGANIZADOR PSIQUICO
Lo que importa para determinar el desarrollo del niño es la respuesta sensible del cuidador.
Entiendase por respuesta sensible:
-notar las señales del bebe
-interpretarlas adecuadamente
-responde adecuada y rapidamente
La falta de sensibilidad puede o no estar acompañada de una conducta hostil o desagradable por
parte del cuidador. Es cuando el cuidador fracasa en leer los estados mentales del bebe y cuando
no puede apoyar al bebe en el logro de sus deseos.
Mas adelante a lo largo de la vida del S, la respuesta sensible cumple un papel fundamental
porque de haber existido…
Le permite al sujeto tener un sentimiento de integracion del self y autovaloración asi como
también Le permite al S dar una respuesta amorosa, cooperativa y reciproca La respuesta
sensible de los padres durante la niñez y la adolescencia de sus hijos es reconocer a estos S
como poseedores de sus propias necesidades- separadas de las de los demas, seres humanos
con su propia individualidad—> esto es brindarles una base segura y permitirle el desarrollo de
conductas exploratorias. “libertad guiada”, libertad no descuidada sino acompañando e
interactuando reflexivamente con el hijo.

25
El cuidador insensible lo que le transmite al niño es que sus señales de pedido de cuidado no son
efectivas o que son algo que no deberia suceder, haciendo sentir mal al niño consigo mismo cada
vez que necesite ayuda.
La respuesta sensible conlleva al menos dos operaciones:
-Acceder al estado mental del niño
-Darle una significacion a ese estado mental
Rta sensible= Rta empatica, aunque empatia implica que hay cierta identificación con el estado
mental del otro, en cambio la rta sensible es negociar internamente entre el sentir como el otro
pero a la vez tener la capacidad de reaccionar como un ser separado de él.
C) UNA TEORIA DE LA INTERNALIZACION Y DE LA REPRESENTACION
La teoria del apego apoya la idea de que la interaccion del niño con sus padres se internaliza
como un sistema representacional/ modelo operativo interno del self y del otro.
Este modelo operativo de la interaccion, de sus progenitores y de si mismo se crea durante los
primeros años de vida y se van consolidando formandose como estructuras cognitivas influyentes.
Como es él con cada uno de sus progenitores, que espera de cada uno de ellos, como es él con
cada uno de ellos, todo esto gobierna la conducta del niño.
No seria correcto hablar de internalizacion, donde uno hace formar parte del interior algo que le
es externo, esto es diferente, aca nos referimos a representar en la mente algo que no ha sido
totalmente externo ni totalmente interno, es la relacion lo que se representa en la mente, es el
“estar con”, uno no se representa al padre como persona, sino la relacion con el padre, el modo
en que el niño es tratado por este padre.
Estas relaciones de apego son muy influyentes en el desarrollo de la personalidad.
D) UNA TEORIA DE LA ANGUSTIA O ANSIEDAD
Angustia y ansiedad como sinonimos. F, en Inhibicion sintoma y angustia: Angustia, sentimiento
displacentero. Es un aumento de excitación que por un lado provoca displacer y por el otro motiva
a la descarga en actos por donde encuantra alivio.
Angustia como señal para ser rescatado de un peligro o de una situacion de peligro que se
anticipa—> para F seria la perdida del objeto en la niñez temprana, la castracion en la etapa fálica
(angustia de castracion) y la samenazas dle SYo en la etapa de latencia. Otro peligro es la perdia
del amor del objeto.
Bowlby retoma la idea del peligro o amenaza de la perdida del objeto en el futuro como fuente de
angustia.
Para él la angustia es una respuesta o una reacción ante
- Las amenazas de perdidas y
- La inseguridad en las relaciones de apego.
Esto ocurre cuando el sujeto es llevado implicita o explicitamente a creer que no es nada ante los
ojos del otro significativo. La angustia no es ante la amenaza concreta de supervivencia fisica sino
amenaza a la existencia psicologia, a la supervivencia del self.

26
-Vernengo, M. P. (2005). Apego. Psicoanálisis: ayer y hoy, 4. Recuperado de
https://www.elpsicoanalisis.org.ar/old/numero4/resenaapego4.htm

1. Definición de Apego
El comportamiento de apego se define como toda conducta por la cual un individuo mantiene o
busca proximidad con otra persona considerada como más fuerte. Se caracteriza también por la
tendencia a utilizar al cuidador principal como una base segura, desde la cual explorar los
entornos desconocidos, y hacia la cual retornar como refugio en momentos de alarma.
El mantenimiento de estos vínculos de apego es considerado como una fuente de seguridad que
permite tolerar esos sentimientos. El apego es observable en la preocupación intensa que los
niños pequeños muestran con respecto a la localización exacta de las figuras parentales cuando
se encuentran en entornos poco familiares.
2. Etapas señaladas por Bowlby en las reacciones de los niños frente a la separación prolongada.
Etapas señaladas por Bowlby:
1) etapa inicial de protesta caracterizada por una preocupación marcada acerca de la ubicación
de la figura de apego que se expresaba en llamadas esperanzadas y llanto.
2) al cabo de unos días los niños que continuaban separados atravesado en una
fase de desesperación aparentemente todavía preocupados por el progenitor perdido mostraban
llanto débil y paulatinamente más desesperanza.
3) Etapa de desapego: con el transcurrir del tiempo los niños se volvían apáticos y retiraban todo
interés aparente por el entorno. Los Minions que llegaban a este estado ignoraban y evitaban
activamente la figura de apego primaria al llegar el momento de un eventual reencuentro y
algunos parecían no poder recordarla.
3. Modelos de Funcionamiento Interno
El ser humano no nace con la capacidad de regular sus reacciones emocionales sino que
necesita de un sistema regulador diadico en el que las señales del niño sobre sus estados son
entendidas y respondidas por sus figuras significativas. Lo cual le permitirá alcanzar así La
regulación de esos estados. Sus experiencias pasadas con la madre por ejemplo son
incorporados en sus modelos representacionales a los cuales bumblebee
denomina Modelos de Funcionamiento Interno (internal working models).
4. Tipos de apego identificados por Ainsworth
Apego seguro: se caracteriza porque aparece ansiedad frente a la separación y reaseguramiento
al volver a encontrarse con la madre. Supone un modelo de funcionamiento interno de confianza
en el cuidador.
DE LOS POWER:
● Modelo de funcionamiento interno de confianza en el cuidador.
● Situación extraña: infante muestra ansiedad, angustia y disgusto frente a la separación, y
reaseguramiento al volver a encontrarse.

27
● Explora el mundo con seguridad
● Equilibrio entre: protección, estimulación de independencia emocional y autonomía. Regulación
afectiva abierto y flexible (Plasticidad).
● Relación con sensibilidad materna a señales del infante (padres accesibles y
sensibles).
● Permanecer relativamente estable frente a estrés.
● Puede integrar experiencia en narrativas reflexivas (emocional y cognitivo).
Apego ansioso/evitativo: muestra poca ansiedad durante la separación y un claro desinterés en el
posterior reencuentro con la madre. Se relaciona con una desconfianza en la disponibilidad del
cuidador.
DE LOS POWER:
● Desconfianza en la disponibilidad del cuidador.
● Situación extraña: infante muestra poca ansiedad durante la separación y desinterés en el
posterior reencuentro.
● Sobreregulación de los afectos. Tendencia a minimizar los afectos
● Tienden a no esperar apoyo y empatía en relaciones. Autoprotección (autosuficiente).
● Las narrativas de sus experiencias muestran falta correspondencia entre memoria semántica y
episódica.
Apego ansioso/resistente: el niño muestra ansiedad de separación pero no se tranquiliza al
reunirse con la madre. Son niños que muestran limitada exploración y juego,tienden a ser
altamente perturbados por la separación y tienen dificultad en reponerse después. La presencia
de la madre y sus intentos de calmarlo fracasan en reasegurarlo y la ansiedad del Infante y la
rabia parecen impedir que obtengan alivio con la proximidad de la madre.
DE LOS POWER:
● No sabe si el adulto será sensible y accesible o lo ayudará por eso se muestra ansioso e
inseguro ante la exploración.
● Situación extraña: infante muestra ansiedad ante separación. No se tranquiliza
con el reencuentro. Los intentos de calmar de madre fracasan en reasegurarlo
● Limitada exploración y juego.
● Subregulación afectiva. Tendencia a incrementar los afectos
● Propenso al aferramiento, dependencia.
● Establecen relaciones fácilmente (adhesivas, ambivalentes).
● Las narrativas tienden a mostrar preocupación por relaciones pasadas o presentes. Pueden no
ser claras y tener contradicciones, por ej. cuando adjetivos positivos no son respaldados o entran
en contradicción).
APEGO DESORGANIZADO (DESCRITO POR MAIN, 1985) DE LOS POWER

28
● Desconfianza ante la figura parental. Los adultos se ubican como fuente de reaseguro y de
temor simultáneamente, activando intensas motivaciones conflictivas.
● Situación extraña: infante muestra conductas aparentemente no dirigidas hacia un fin
(inmovilización, golpeteo con las manos, golpeteo con la cabeza, altos niveles de ansiedad, deseo
de escapar de la situación aún en presencia de la madre).
● Desorganización y desorientación (Main y Solomon, 1990).
● Desregulación afectiva y conductual
● En AAI presentan narrativas con importantes contradicciones y rupturas de razonamiento.
● Causas probables: episodios traumáticos severos en la infancia.
5. Qué es la Entrevista de Apego de Adultos. Qué rastrea. Clasificación.
Elaborada por Mary Main, se ocupó en correlacionar la conducta del niño en la situación extraña
con el discurso de los padres. Desarrolló mediciones y construcciones teóricas basándose en las
narrativas de padres y madres sobre sus experiencias relacionales.
Busca sobre todo clasificar el estado mental del sujeto en cuanto a sus vínculos.
Main describió tres tipos de apego del adulto: seguro/autónomo, inseguro/desentendido
(despreocupado) e inseguro/preocupado.
DE LOS POWER
Mary Main se ocupó de estudiar el discurso de los padres en relación a su propia historia, para
profundizar en los aspectos representacionales de la teoría del Apego. A partir del estudio de las
narrativas, describió tres tipos de apego del adulto: Seguro–autónomo, Desentendido y
Preocupado.
Diseñaron la Entrevista de Apego Adulto - AAI (Adult Attachment Interview, (George, Kaplan y
Main 1984,1986,): Es una entrevista semi-estructurada, que consiste entre 15 y 18 preguntas. Por
ejemplo, se pide una descripción global de la relación con ambos padres durante la infancia;
luego, se piden 5 adjetivos o frases que describan las relaciones en la infancia con la madre y con
el padre. Después de eso, a los participantes se les pide justificar sus elecciones, adjetivo por
adjetivo con escenas concretas. Se realiza un análisis minucioso de las narrativas tomando
diversos aspectos tales como, claridad, cantidad, relevancia en las mismas.
APEGO SEGURO-AUTÓNOMO: Se mantiene un discurso coherente y colaborador mientras el
entrevistado describe las experiencias relacionadas con el apego y sus efectos, ya sean
favorables o desfavorables.
DESENTENDIDO:Descripciones positivas y normalizantes de los padres (―excelente, una madre
muy normal‖) que no se sostienen o se contradicen con incidentes específicos. Tiende a referir
que las experiencias negativas han tenido poco ningún efecto. Los relatos son cortos, a menudo
debidos a insistencia en la falta de memoria.
PREOCUPADO:Relato que muestra preocupación por experiencias aparentemente de enfadado;
confundido y pasivo o asustado y abrumado. Tendencia a presentar algunas frases complicadas
gramaticalmente, llenas de frases vagas (bla,bla, bla) o jerga psicológica. Los relatos son largos
con algunas respuestas irrelevantes.

29
Fonagy tomará estos estudios para el estudio de la mentalización.
6. Objetivos del proceso terapéutico y funciones de analista en el proceso terapéutico o según
Bowlby.
Bowlby considera que la actitud empática del terapeuta puede producir modificaciones en los
Modelos de Funcionamiento Interno. El objetivo central es ayudar a revisar el paciente los
modelos representacionales de sí mismo y de sus figuras de apego los cuales rigen actualmente
sus percepciones predicciones y actos.
Clasifica las funciones del analista de la siguiente manera:
- Proporcionar una base segura a partir de la cual el paciente pueda explorarse a sí mismo y sus
relaciones. Es decir establecer un vínculo confiable.
- Realizar con el paciente las exploraciones sobre sus relaciones interpersonales.
- Señalar la manera en que éste tiende a construir sus sentimientos sus expectativas en los
vínculos predicciones y consecuencias de las mismas.
- Relacionar sus modos de vincularse, incluso con el terapeuta, con experiencia de la vida real
que tuvo con figuras de apego y así arrojar comprensión sobre sus relaciones actuales.
- En la práctica todas estas funciones se realizan simultáneamente.

PRACTICO 2
-American Psychiatric Association (2014). Introducción. En Manual diagnóstico y estadístico de
los trastornos mentales (5ª ed., pp. 5–17). Arlington, VA: American Psychiatric Publishing.
Texto en Teórico semana 1
-Valladares, L., Giusti, S., Helmich, N., Etchevers, M. (2016). Diagnóstico clínico. Manuscrito no
publicado, Facultad de Psicología, Universidad de Buenos Aires, Argentina.
DIAGNOSTICAR: UNA TAREA COMPLEJA
Diagnostico, para distinguir, lejos de rotular de una vez y para siempre es una construcción a ser revisada
permanentemente. ≠ a Rotular que es una verdad inamovible.
Un manual diagnostico es una herramienta que puede ser bien o mal utilizada.
¿Para que sirve hacer un diagnostico?
Levi Strauss, ‘el pensamiento salvaje’- clasificación: Introducir orden en el mundo natural.
Clasificar/poner orden es una de las maneras del pensamiento humano para conocer. Los
humanos captamos la realidad, la experiencia, el mundo a través de categorías del pensamiento.
La mente se antepone a la experiencia.
La clasificación es inherente al orden simbólico de la cultura—> por ser sujetos de la cultura y de
un mundo simbólico se nos impone clasificar, ordenar, dar una organización a la experiencia, al
mundo para poder abordarlo, es una manera de poner orden al caos de la naturaleza.
Definir un problema clínico de una forma clara y accesible es importante para:

30
-facilitar la comunicacion entre investigadores de distintos marcos teoricos.
-facilitar la investigacion de resultados de diferentes tratamientos a un mismo problema.
-Establecer si el paciente tiene o no un trastorno mental
-Definir una estrategia de abordaje
-Evaluar la evolucion del paciente y la rta al tratamiento para definir la posibilidad o no de una
interconsulta
-Al tener un lenguaje comun/diagnosticos consensuados se puede compartir rdos de trabajo e
investigacion
-Evaluaciones diagnosticas para aplicar en distintos campos, laboral, escolar, judicial, ambitos
que
pueden requerir la aplicacion de la psicologia para la toma de decisiones.
HISTORIA DE LAS CLASIFICACIONES EN PSICOLOGIA
Hipocrates en Grecia (S.IV ac) empezo a describir y clasificar enfermedades mentales como la
epilepsia, mania, paranoia, delirio toxico, psicosis puerperal, fobias e histeria.
Kraepelin, 1899, es el padre de la clasificacion de los trastornos mentales tal como los
conocemos hoy en su Manual de psiquiatria. Sus criterios se fundaban en las causas orgánicas
de la enfermedad: hereditarias, metabolicas, endocrinas, alteraciones cerebrales, es por esto que
las clasifcaciones actuales comenzaron solo por las enfermedades mentales CON causa
organica. La CIE (Clasificacion internacional de Enfermedades) en sus 4 primeras ediciones solo
incluyo enfermedades mentales con causa organica—> Sistema de Clasificacion Europero de la
OMS.
DSM—> Americano y surge luegod e la Sda Guerra Mundial.
SISTEMAS DIAGNOSTICOS OPERATIVOS
CIE 10 y DSM 5 los mas actuales y utilizados en Argentina—> la utilizacion de sistemas
diagnosticos operativos es obligatorio tanto en el ambito hospitalario como en los sistemas de
salud obras sociales y prepagas.
BREVE HISTORIA DEL DSM
Surge ante la necesidad de acuerdo de un sistema de clasificación de los trastornos mentales
consensuado, universalizar criterios diagnósticos y patologías.
Primer versión en 1952 a cargo de la Asociación Americana de Psiquiatria.
1968- version II con gran influencia psicoanalitica, corriente dominante en los psiquiatras
americanos que participaron en su edicion.
1980- version III pretensión de hacer un manual cientifico netamente basado en la evidencia,
viraje hacia el modelo medico de la salud mental alejandose del psicoanalisis
2013- DSM 5- Se expresa que se necesitaran cambios permanentes para mantenerse
actualizados con los avances de investigaciones cientificas en salud mental.

31
Dos enfoques.
Dos posiciones conceptuales distintas:
Categorial: Es la forma mas tradicional. Es establecer categorías sobre propiedades claramente
definidas, presencia/ausencia de dichas propiedades definen la integracion o no en esa categoria.
Un diagnostico desde este enfoque tendria solo dos valores posibles: presencia/ausencia- Método
binario.
Criticas: Reduccionismo y alejamiento de la realidad clinica (compleja).
Dimensional: A diferencia del enfoque categorial, el dimensional implica una medicion cuantitativa de una
dimension, ordenando sintomas segun los grados de intensidad—> esto rompe con la bipolaridad del
enfoque categorial, permitiendo una escala de al menos tres valores. Una dimension es mas un constructo
que puede ser medido ≠ de una propiedad de la cual solo se puede indicar presencia/ausencia.
En la 5ta edicion del DSM nos encontramos con un enfoque Dimensional, antes Categorial, en
donde se habla de la intensidad de ciertos fenómenos Introducción al DSM 5
Definición del Trastorno Mental:
Sindrome (conjunto de sintomas) caracterizado por una alteración clinicamente significativa de -el
estado cognitivo
-la regulacion emocional
-o del comportamiento del individuo
que evidencia una disfunción de los procesos psicológicos, biológicos o del desarrollo que
subyacen a la función mental
Estos trastornos mentales habitualmente van acompañados de un estres significativo o una
discapacidad social, laboral o de otras actividades. Una respuesta predecible o culturalmente
aceptable a un estres usual o una perdida no se considera trastorno mental.
Comportamientos socialmente anómalos (sean religiosos, políticos o sexuales) no constituyen
trastornos mentales
Utilización del manual:
Para el diagnostico no es suficiente comprobar la presencia de los síntomas establecidos en el
criterio diagnostico sino tambien se necesita del Juicio clinico del medico para determinar la
gravedad relativa.
Es el medico quien decide que subtipo del trastorno se considera mas adecuado al paciente
citando la gravedad (leve, moderado, grave, extremo), caracteristicas descriptivas (como por ej
introspeccion buena, en estado controlado, etc) y el curso del trastorno (en remision parcial, total=
remision/recuperacion, en recidiva/ posible vuelta del trastorno luego de un tiempo). Si no se
cumplen todos los criterios para un sub tipo de trastorno el clinico puede determinarlo como un
trastorno especificado (dando las caracteristicas del mismo) o como un trastorno no especificado.
TEÓRICO SEMANA 3
-Barber, J. P. y Solomonov, N. (2016). Psychodynamic Theories. En J. C. Norcross, G. R.
Vandenboss, D. K. Freedheim, Olatunjis, B.O. (Eds.), APA Handbook of clinical psychology.
Theory and research (Vol. 2, 53-77). Washington, DC: American Psychological Association.*

32
El capítulo recorre algunos temas centrales de las teorías psicodinámicas (psicoanalíticas)
1- Dentro de la evolución histórica del psicoanálisis, qué escuelas se señalan y cuáles son sus
aportes más importantes.
Psicología del yo: desarrollo de las funciones del yo, resolver conflictos internos
Relaciones objetales, sostienen que la personalidad se forma por la relación entre el infante y el
amor de objeto - la madre
Teoría del apego, formuló que la personalidad del individuo está formada por la relación temprana
con el cuidador primario. Apego porque refiere al lazo primario de un infante con su madre
Psicología del self, los psicólogos del self se alejaron de la teoría Freudiana enfatizando la
importancia de la experiencia de subjetividad personal del individuo. El
supuesto básico de su teoría es que la motivación básica de los seres humanos no es satisfacer
las pulsiones sexuales y agresivas, sino mantener su autoestima
Psicología interpersonal. En contraste con la premisa de Freud de que la personalidad es formada
ante la lucha entre diferentes instancias psíquicas, Sullivan supuso que la personalidad se
desarrolla desde las relaciones interpersonales con otros. Mientras que Freud y sus predecesores
se enfocaron en las fantasías inconscientes, deseos y el uso de defensas, Sullivan (1953) se
concentró en el surgimiento del yo en el contexto de las relaciones del individuo con otros.
y Psicoanálisis relacional. la teoría relacional surge de la suposición de que la motivación primaria
de los humanos es estar en una relación. El modelo relacional consiste en tres dimensiones: el
yo, el objeto y la interacción entre ellos en un espacio psíquico, que crea la propia experiencia
subjetiva (J.Mitchell, 1986).
2- Problemáticas principales señaladas que caracterizan las consultas en la actualidad. De cada
una (Depresión, Miedo al abandono; Autoestima baja, Trastorno de pánico, Trauma)*, situar lo
siguiente: Conceptualización psicodinámica del cuadro. Autores de referencia. *Tener en cuenta
que algunos de estos cuadros se abordarán más profundamente a lo largo de la cursada.
DEPRESIÓN: Una conceptualización psicodinámica contemporánea (Busch, Rudden y Shapiro,
2004) ha integrado el énfasis de Klein (J. Mitchell, 1986) y S. Freud (1917/1936) en la pérdida de
un objeto y la ira interiorizada hacia él, así como Kohut (1971) se centra en el papel de la baja
autoestima y la autocrítica.
MIEDO AL ABANDONO: las teorías psicodinámicas conceptualizaron el miedo al abandono
dentro del diagnóstico de personalidad límite - límite entre la neurosis y la psicosis- (Kernberg,
1975). Con el desarrollo de la teoría del apego, el miedo al abandono se define más comúnmente
como emergente dentro del contexto más amplio de la relación temprana con el cuidador primario
(Summers y Barber, 2010). Por ejemplo, Margaret Mahler (1972) siguió el marco de apego de
Bowlby (1973) e identificó el miedo al abandono como resultado de la falla del infante en
establecer un vínculo emocional con la madre y desarrollar un sentido de individualización.
Utilizando conceptos de la teoría de las relaciones objetales, Kernberg (1975) sostuvo que el
miedo crónico al abandono es el resultado de predisposiciones constitucionales a la agresión que
se ven agravadas por experiencias traumáticas durante las relaciones tempranas

33
AUTOESTIMA BAJA. Relacionada con el narcisismo. S. Freud (1914-1963) conceptualizó el
narcisismo como una regresión a una etapa temprana del desarrollo del narcisismo primario,
debido a la falta de un ambiente acogedor y receptivo. Conceptualizó la auto-compasión
narcisista como una reacción defensiva contra sentimientos generalizados de soledad, pérdida, y
ansiedad. Kernberg (1975) describió a las personas con personalidad narcisista, como personas
que sufren una contradicción entre sentimientos de grandiosidad e intensos sentimientos de
inferioridad, una necesidad de ser admirados y reconocidos, y persistente ira y resentimiento
hacia los demás. Si bien Kernberg hizo énfasis en la ira, el resentimiento
y los sentimientos de derecho, Kohut (1971, 1977) se centró en la dolorosa experiencia emocional
de soledad, tristeza y vulnerabilidad al rechazo que es compensada con una exagerada auto-
preocupación y sentimientos de grandiosidad.
TRASTORNO DE PÁNICO: Sobre la base de los trabajos tempranos de Freud, se desarrolló una
formulación psicodinámica del trastorno de pánico que integra factores psicológicos y
neurobiológicos junto con estresores externos (Milrod et al., 1997).
Otros teóricos psicodinámicos también hicieron énfasis en la importancia de las relaciones
tempranas con sus cuidadores en los pacientes con pánico (Shear al., 1993).
TRAUMA : Las teorías psicodinámicas se centran en el significado intrapsíquico y personal del
trauma, haciendo énfasis en los conflictos internos y la ambivalencia con las que luchan estos
pacientes. No se ha prestado mucha atención a las formas en que los pacientes enfrentan los
eventos traumáticos externos. Un enfoque más contemporáneo del trauma derivado del
procesamiento biológico y cognitivo y los hallazgos psicoterapéuticos, proporcionan una
formulación psicodinámica sobre la aparición de síntomas post-traumáticos (Summers & Barber,
2010).
3)INVESTIGACIÓN EN PSICOTERAPIA: EVIDENCIA, APORTES: a qué se
llama terapia basada en la evidencia. Que son los RCT? (RAMDONIZE CONTROL TRAILS)
―La APA aprueba los múltiples tipos de evidencia de investigación (p. ej., la vinculada con la
eficacia, la efectividad, el costo-efectividad, el costo-beneficio, la evidencia epidemiológica, la
evidencia vinculada con la utilización de los tratamientos) que contribuyen a la práctica
psicológica efectiva.‖
Tratamientos testados empíricamente: A) especificidad (la probabilidad de identificar falsamente
un tratamiento como efectivo) , B) sensibilidad (la oportunidad de clasificar por error un
tratamiento efectivo como no efectivo)
Tratamientos basados en la evidencia Distinguir los que cumplen con varios indicadores
relevantes :
-coherencia teórica
- impacto en la salud pública
- aceptabilidad usuario/consumidor
Base de datos de estudios clínicos aleatorios controlados (RCT Randomized Control Trials). RCT
que examinan la eficacia de la terapia dinámica se han centrado en STDP en lugar de
psicoterapia o psicoanálisis a largo plazo

34
4- Procesos terapéuticos: Cuáles son los mecanismos de cambio principales en las terapias
psicodinámicas. (Este tema se aborda más adelante en la cursada).
el papel de la alianza terapéutica, en contraste con el papel de la autocomprensión como
potenciales mecanismos de cambio en la psicoterapia. Lo mismo puede decirse de la
mentalización.
5- Perspectivas psicodinámicas de corto plazo. Aportes de Malan.
Malan introdujo dos triángulos conceptuales en los que el terapeuta trabaja activamente con el
paciente. El primer triángulo fue llamado el triángulo del conflicto y consistió en la defensa, la
ansiedad y el impulso oculto ante el cual el paciente se defiende. A través de la interpretación de
la defensa y la ansiedad, el impulso oculto puede ser traído a la conciencia y la ansiedad causada
por el impulso inconsciente puede ser reducida.
El segundo triángulo de Malan (1976) era el triángulo de la persona, también llamado el triángulo
del insight. Este triángulo incluyó tres prototipos de relaciones: la relación con el terapeuta (la
transferencia), las relaciones actuales del paciente y las relaciones pasadas del paciente con
otros significativos. La terapia de Malan se encuentra centrada en la identificación de conexiones
entre estas relaciones a partir de la exploración de conflictos internos subyacentes recurrentes.
5- Psicoterapia de apoyo – expresiva (SET): características.
En la SET, las exploraciones e interpretaciones de los terapeutas se centran en el Tema de los
Conflictos Relacionales Nucleares (Core Conflictual Relational Theme, CCRT; Luborsky, 1984). El
CCRT es un modelo teórico, así como un instrumento de medición clínica, diseñado para ayudar
a los terapeutas e investigadores a alcanzar una comprensión más profunda de los problemas
centrales del paciente. La base de este método es el supuesto de que los problemas psicológicos
del paciente derivan de narrativas internas que dan forma a los patrones de relación del paciente
6- Proceso terapéutico y Fases.
Uno de los principales objetivos de la terapia psicodinámica es facilitar la toma de conciencia y
comprensión de los comportamientos en el presente y sus orígenes en el pasado. Aumentar la
reflexión y facilitar una actitud de autoexplotación y autodescubrimiento. Los estudios que
examinan los procesos terapéuticos se han centrado en 8 mecanismos principales de cambio:
insight, mecanismos de defensa, calidad de relaciones objetales, función reflexiva, rigidez y
transferencia en las relaciones, experiencia emocional correctiva, alianza terapéutica.
Fase de apertura: una evaluación y conceptualización del problema central y las maneras en las
que emergen sus síntomas, acontecimientos significativos y las relaciones interpersonales. Se
crea un plan de tratamiento elaborando de manera aprox. oportunidades de cambio, posibles
obstáculos y resistencias.
Fase intermedia: ambos crean una narrativa comprensiva del curso de la vida del paciente que
incorpora factores biológicos, experiencias infantiles, relaciones interpersonales, sucesos de la
vida centrales. Incrementar la conciencia de sí del paciente para construir una narrativa nueva y
diferente.
Fase de cierre: paciente ha mejorado su capacidad de autocomprensión y reflexión,

35
utilizando defensas más maduras, mejorado elecciones de vida y se sentirá más seguro de sus
capacidades para enfrentar frustraciones o decepciones. Se explora reacciones emocionales ante
la finalización del tratamiento.
-Etchevers, M., Helmich, N., Giusti, S. (2018). Trastornos de la personalidad y apego: diagnóstico
y evolución. Manuscrito no publicado, Facultad de Psicología, Universidad de Buenos Aires,
Argentina.
1- Qué es el modelo de los 5 factores de McCrae y Costa? cuáles son.
El sistema diagnóstico categorial muestra limitaciones, en particular para los clínicos, por ello se integra a
las propuestas diagnósticas la perspectiva dimensional → Asume que los trastornos mentales no son
cualitativamente diferentes de la normalidad (o salud mental) sino que representan un extremo por exceso o
déficit, o gradaciones. Los
TP serían, entonces, expresiones disfuncionales del modo de ser de los individuos, por lo que
también es necesaria una concepción dimensional que permita establecer relaciones entre los
rasgos del funcionamiento normal y psicopatológico de la personalidad. Este modelo no aborda a
los TM por presentar limitaciones propias.
El modelo de los 5 factores proporciona una estructura coherente, conceptual y empírica, y
propone considerar a los TP como variantes desadaptativas de la personalidad. Es un
cuestionario que permite explicar la estructura general de la personalidad, analizando 5 factores:
1- Apertura a la experiencia: Presencia de imaginación activa, sensibilidad estética, capacidad de
introspección o curiosidad intelectual
2- Responsabilidad: Rasgos ligados a la capacidad para controlar los impulsos, actuar con
propósitos o metas claras, planificar, organizar y llevar adelante proyectos e ideas.
3- Extraversión: Rasgos que reflejan la tendencia a comunicarse con las demás personas, a ser
asertivos, activos y verbalizadores.
4- Amabilidad: Rasgos que se asocian a la capacidad para establecer vínculos psicosociales, al
altruismo, y a una amplia disposición a preocuparse por los demás (empatía).
5-Neuroticismo: Predisposición a experimentar emociones negativas como miedos, sentimientos
de culpa, tristeza, enojo, o inestabilidad emocional.
El tratamiento del TLP no aborda ni se centra en toda la estructura de la personalidad, los clínicos
tratan dominios específicos del Modelo de los 5 factores. Es decir, el cambio ocurre respecto a
componentes como por ejemplo, la inestabilidad afectiva, la impulsividad o las conductas
autolíticas, en lugar de abordar todo el constructo global del TLP. Uno de los tratamientos para el
TLP es la Terapia Dialéctica Conductual (TDC; en inglés, DBT), sin embargo se ha demostrado
que es eficaz para muchos componentes de este TP, pero no para todos (disminuye las
conductas autolesivas y la hostilidad, pero no otros aspectos como la desesperanza).
2- Qué factor está más elevado en un paciente con TLP
Los pacientes con TLP tienden a puntuar bajo en nivel de agudeza y conciencia, y alto en nivel de
neuroticismo.
3- Características nucleares del TLP? Está
- Inestabilidad emocional

36
- Impulsividad
- Conflictos interpersonales
- Comorbilidad

-Juri, L. (2013). La psicoterapia psicoanalítica de la teoría del apego. Revista de la Asociación de


Psicoterapia de la República Argentina, 6(1), 1-21.
INTRODUCCION
El aporte de la teoría del apego a la psicoterapia no es en tanto técnica sino mas bien, instala la
idea de un modelo vincular del funcionamiento del psiquismo.
El apego es una motivación psíquica por la cual un S se dirige a otro en busca de protección y
seguridad, acercamiento no solo físico sino tmb emocional. Cuando el apego resulta satisfecho
deviene en un lazo de amor de Bowlby, el apego se activa durante el ciclo vital en situaciones
especifica dnd el S busca protección y seguridad en el vinculo.
Bowlby habla de “Apego Primario” es decir que es original, y no secundario a otras motivaciones
como la búsqueda de alimento o sexo. Este apego primario es una motivación primaria
relacionada a la búsqueda de protección y supervivencia de la cría, con la angustia de separación
que conlleva.
Este apego es algo asi como innato pero se va especializando en el vinculo intersubjetivo de la
cria con su cuidador, sonrisas-sosten, miradas-alivio y calma, etc hasta que hacia el primer año d
evida el niño ya tiene una representacion internalizada de su figura de apego.
El poder patogénico que tienen las privaciones del cuidado materno fue postulado por Bowlby.
LA TEORIA DEL APEGO ES UNA FORMA DE “CONDUCTISMO”?
Critica de los psicoanalistas: la teoría del apego es conductiva, un insulto bastante grande
considerando que B era vice presidente de la asociación psicoanalítica británica y que construyo
la teoría del apego pensando en su uso en la practica clínica.
B reconoce que esta critica se debe a que el ha realizado observación de la conducta en animales
y tmb en humanos para llegar a sus conclusiones y a que no hay realizado una correcta distinción
entre conducta y deseo o disposición psicológica de apego.
Hace una diferenciación entre Conducta de Apego (llanto, búsqueda de proximidad física,
aferramiento, etc) y Apego como disposición psicológica a buscar proximidad con otro individuo
en determinadas circunstancias donde se necesita protección y seguridad, este vinculo de apego
funciona como un regulador de las emociones del apegado
LA TEORIA DEL APEGO NIEGA LA VIDA MENTAL ICCTE?
La teoría del apego tiene su versión del Iccte- con modelos representacionales o modelos
operativos internos-> sistema de creencias sobre si mismo y la figura de apego, el S excluye al
iccte la información conflictiva que atenta contra estas representaciones e impide el retorno de
estas representaciones conflictivas con el fin de mantener la representacion de su figura de apego
y de si mismo.

37
Estos modelos representacionales se construyen por lo vivido y por lo contado por los padres. Un
terapeuta guiado por la teoria del apego acompaña al paciente a descubrir y revisar sus modelos
representacionales operantes, un modelo representacional de si mismo como seguro, fomenta la
exploracion del sujeto en cambio una representacion de si mismo como incapaz o débil
predispone al panico o agorafobia.
LA TEORIA DE APEGO COMO PARADIGMA PSICOANALITICO ALTERNATIVO
Paradigma: Conjunto de teorías, técnicas, creencias y valores compartidos por los miembros de
una comunidad científica, en nuestro caso psicoterapeutas, psicoanalistas. Pero no tiene que ser
compartido y aceptado por todos los miembros de esa comunidad, pueden ser un grupo
particular.
Es en este sentido que la teoría del apego es un paradigma alternativo a la teoría psicoanalítica. Analista
espejo: refleja lo que le llega ≠ Analista selectivo: selecciona y realiza una lectura del material según su
paradigma
Para Bowlby el psiquismo controla el vinculo de apego ≠ Para Freud el psiquismo controla los estimulos.
Asi dadas las cosas, para el primero el peligro no lo representa el aumento de montos de
excitación incontrolable sino la ausencia o amenaza de separación física y emocional respecto de
la figura de apego.
El psiquismo lleva a cabo una tarea conciente e inconciente orientada a sostener el vinculo, es
decir que las motivaciones, mecanismos de defensa, deseos, angustias, miedos, percepcion, la
memoria, la forma de procesar la información conciente e inconcientemente buscan mantener la
distancia optima emocional y fisica con las figuras de apego.
TEORIA DEL DESARROLLO/ TEORIA DE LA PSICOTERAPIA
Para la teoría del apego, se pondera una base segura para el desarrollo de la personalidad de la
cria. Criar a un niño lo suficientemente seguro para emprender actividades exploratorias en su
entorno. Fundamental: empatia y respuesta sensible por parte de la figura de apego.
Fuerza al vinculo de la cria respecto de su figura de apego ≠ Fuerza contraria/antagonica: Motivación
exploratoria Los padres pueden motivar o desalentar una motivación exploratoria Un apego seguro favorece
la autonomía del si mismo y la capacidad exploratoria Una inseguridad en el apego promueve temores e
inhibiciones Exploracion no es solo ambiental, sino tmb psiquica, el niño puede explorar su ppio psiquismo
y el de otros, descubriendo emociones, deseos, motivaciones, intenciones, etc—> esto favorece el proceso
de mentalizacion o reflexion.
Cuando el padre promueve que el niño haga preguntas y exprese sus ideas y deseos lo alienta a
explorar e identificar que es lo que siente y piensa Así como para el desarrollo el niño necesita
una madre/cuidador empatíca y experimentar la seguridad y confianza de una base segura, en
psicoterapia para el paciente es fundamental experimentar confianza con el psicoterapeuta para
aventurarse en sus exploraciones psiquicas de sus pensamientos y sentimientos.
La confianza, la empatia del terapeuta y la aceptación sin prejuicios ni criticas configuran la base
segura que le permite al paciente explorar. La terapia no puede empezar si el terapeuta no puede
brindarle seguridad al paciente.
En funcion a esto, luego el rol del terapeuta es activo, anima al paciente a pensar, sentir y
expresar lo que no pudo en su momento, el terapeuta hace preguntas y señalamientos.

38
Base segura terapeutica= funciona como un ambiente facilitador a partir del cual es posible
emprender exploraciones del psiquismo. Similitud con la alianza terapeutica postulada por F.->
alianza con el Yo racional y colaborador del paciente para llevar a cabo la terapia.
TERAPEUTA- OBSERVADOR TERAPEUTA-EMPATICO
Empatia, ponerse en el lugar del otro y conectar con lo que el otro siente y experimenta. La
empatía como lazo emocional es necesaria para el desarrollo y el sostén emocional. Esta empatia
que interviene en el desarrollo del bebe, es utilizada de modo particular como metodo de obtener
informacion en la practica terapeutica.
A diferencia del terapeuta observador que desde el exterior analiza la conflictiva del paciente a
través de su marco teórico, el terapeuta empatico trata de ubicarse en el interior de la perspectiva
del paciente.
El Terapeuta Empatico prioriza el estado emocional del paciente como acceso a su subjetividad.
El analista no es un “espejo” sino un “selector”, que tiene ante si varios caminos interpretativos,
selecciona y sigue determinado rumbo.
Empatia como método de recolección de datos clínicos sin despojarnos de nuestras premisas
paradigmáticas (no hay empatia puramente neutral, se toma una posicion con lo que se recolecta
a traves de la empatia).
-Killingmo, B. (2005). Una defensa de la afirmación en relación con los estados de afectos no
mentalizados. En A. Maladesky, M. B. López, Z. López Ozores (Comps.), Psicosomática. aportes
teórico-clínicos en el siglo XXI (pp. 169-185). Buenos Aires
Síntomas Somaticos= Afectos no mentalizados, el paciente no vivencia los afectos como propios
o no los puede expresar con palabras y/o simbolos, se expresan en el cuerpo.
No todo afecto no mentalizado produce sintomas somaticos, la hiperactividad y la depresion son
otras formas de expresion de afectos no mentalizados.
Patologia del Deficit o de Afecto no Mentalizado
>>Incapacidad de estos paciente de expresar afectos, emociones xq no los vivencia como ppios
>>El psicoanalisis clasico no puede abordar estas patologias, no se puede acceder al “si mismo”
del paciente por medio de la interpretación
>>Tecnica para abordar esta patologia: AFIRMACION, reemplaza a la interpretación clasica
MENTALIZACION DEFICIENTE
Acompasamiento afectivo—> entre la madre y el hijo, es una forma de compartir una experiencia
afectiva, antes de que el niño adquiera la capacidad de simbolizar, mentalizar o representar
simbolicamente.
Esto sienta las bases para el establecimiento de patrones emocionales duraderos en relación al si
mismo y a los otros
Las fallas en el acompasamiento afectivo son la causa de la patología del Deficit.
Deficit

39
-Fallas en la capacidad de feedback emocional por parte del cuidador—> efectos en el infante: le
provocan incapacidades estructurales
-Tres clases de fallas
1- falta de estimulacion
2- sobre-estimulacion
3- Estimulacion mal orientada
-En analisis es importante:
* detectar el tipo de falla empatica del cuidador por medio de la transferencia
* Entender los efectos de esta falla en la estructura de la personalidad del S, estos defectos
(causa del deficit) pueden ser:
-falta de confianza en la constancia del objeto
-baja capacidad de postergar o controlar impulsos
-falla en la concepcion del sentimiento de si
-fallas en la tolerancia a la frustracion
-fallas en la mantencion estable de la identidad
-todos los aspectos de la personalidad pueden ser influenciados por los efectos de las fallas en el
desarrollo emocional temprano
-MENTALIZACION
Es el proceso por el cual las interacciones interpersonales, excitaciones somaticas y motrices se
modifican cualitativamente y se transforman en contenidos mentales. Al simbolizarse estas
excitaciones interpersonales, somaticas y motrices devienen en afectos mentalizados Una falla o
capacidad deficiente para la mentalizacion implica que:
- el S no puede representar simbolicamente o por medio de palabras sus estados emocionales
- vivir los afectos como propios
- Relacionarse consigo mismo como agente de su propia experiencia
- Significacion cognitiva del afecto baja—>excitacion somatica alta
- ansiedad asociada a la perdida de si mismo
Estos pacientes no pueden beneficiarse de intervenciones Interpretativas—> en lugar de revelar
el significado el analista tiene que ayudar a que el paciente vivencie el significado por medio de
intervenciones AFIRMATIVAS
LA RESPUESTA AFIRMATIVA
-El S necesita vivir la experiencia del si mismo, necesita sentir que ese es él mismo—>
EXPERIMENTAR EL SENTIMIENTO DEL SI MISMO.

40
-Los pacientes que por tener deficiencia en la mentalizacion de afectos o por aislamiento afectivo
no pueden hablar de si mismos desde una experiencia de yo propio necesitan una AFIRMACION
para fortalecer su Yo—> esta afirmacion debe provenir de un otro.
-4 formas de afirmacion:
1- Experiencia de ser VISTO: si el otro me ve, entonces existo- sostiene el SER
2- Experiencia de ser ENTENDIDO: si me entiende un otro, entonces puedo relacionarme con un
otro- sostiene la RELACION c un otro
3- Experiencia de ser ESCUCHADO: si se preocupa por ecucharme entonces soy algo- otorga
sentimeinto de SUSTANCIA
4- Experiencia de que un otro este DE ACUERDO CONMIGO: si esta de acuerdo conmigo, mi
razonamiento es valido- JUSTIFICACION.
No hay un razonamiento conciente de todo esto, sino que hay una codificacion emocional
espontanea en el paciente- el paciente simplemente lo siente.
Cada una de estas modalidades de afirmación contribuye de una manera diferente a la
experiencia subjetiva del si mismo.
Cada una de estas modalidades de afirmación funciona como una retroalimentacion emocional
que fortalece los cimientos del si mismo del paciente
EFECTO TERAPEUTICO DE LA AFIRMACION
Ejemplo: “Puedo entender lo que significa ser como Ud” por medio de una afirmacion, lo que le
pasa al paciente, qué siente, qué le sucede, deja de ser un enigma, puede ser compartido con un
otro, puede ponerse en palabras, hay simbolización del si mismo y sus afectos.
Dos efectos terapeuticos inmediatos- GANANCIA DINAMICA DE LA AFIRMACION en el proceso
terapeutico:
1- El ser entendido, rompe el aislamiento y se piensa al otro inclusivo, hay un “nosotros”.
Terapeuta emocionalmente accesible.
2- La afirmación elimina la “duda paralizante” sobre el si mismo, Yo estable. El Yo debe formarse
como base estructural para una experiencia de si mismo estable y confiable, esto se logra por
medio de las intervenciones afirmativas del analista, confiablidad en la experiencia del si mismo.
La busqueda de significado no es posible, es vivido como algo que se hace “de la boca para
afuera” mientras el Paciente espera que algo verdadero suceda. Una vez que el S por medio de la
afirmacion, recupera el sentimiento de existencia y se adueña de sus afectos y experiencia de si
mismo, su identidad, ahi deviene posible la interpetacion para que el S puede entender de donde
viene su sentimiento de desesperanza y asi producir efectos a largo plazo
LA VOZ DEL ANALISTA
Para que la Rta afirmativa del analista tenga efecto terapéutico debe expresar una verdadera
experiencia en el analista, debe trasmitir su PRESENCIA EMOCIONAL, una escucha
contenedora.

41
Es fundamental el lenguaje no verbal, entonacion, el uso de la voz, la calidad del sonido de la voz
del analista—> la interpretacion lleva un signo de interrogacion al final, la afirmacion un punto a
parte.
Por medio de la ENTONACION de la voz transmitimos el afecto, el significado afectivo.
LA MENTALIZACION Y LA AFIRMACION ESTAN CONECTADAS
Mentalizacion—> representacion simbolica de estados emocionales/afectivos
>>Representacion del afecto y del si mismo provienen de la misma matriz—> como parte del efecto del
proceso de diferenciación del Yo ≠ No Yo que se da en las relaciones intersubjetivas tempranas
La tarea ppal de la Afirmacion es confirmar la experiencia de Si mismo del paciente. Aumenta la
Representacion de si mismo, aumenta la diferenciacion estructural yo/ No Yo por medio del
analisis.

PRACTICO SEMANA 3
Bateman, A. y Fonagy, P. (2016). Qué es mentalizar. En Tratamiento basado en la mentalización
para trastornos de la personalidad. Una Guía Práctica. Bilbao, ES:Desclée De Brouwer. De pág.
27 a pág. 34.
1- Mentalización. Definición. Características.
Mentalización es el proceso por el cual damos sentido a los demás y a nosotros mismos, implícita
y explícitamente, en términos de estados subjetivos y procesos mentales. Es la capacidad de
percibir y comprender la conducta propia y la de los demás en términos de estados mentales
(pensamientos, sentimientos, deseos y anhelos). Permite comprender que a las conductas y
comportamientos de las personas subyacen intenciones y emociones que son inobservables,
cambiantes y dinámicas.
Características: Los estados internos son opacos, por lo tanto se infieren. Las inferencias son
propensas a errores, infalibles. Los estados mentales se pueden cambiar, son sensibles a
influencias ambientales. Principio de mentalización: actitud indagadora. Los estados mentales
influyen en la conducta. La mentalización se dificulta ante estrés y ansiedad. La mentalización nos
permite vernos desde el exterior, y ver a los demás desde el interior.
2- Mentalización y relación con apego.
Se adquiere a través de las relaciones interpersonales tempranas.
La construcción de la capacidad de mentalizar. Tres componentes:
1) Especularización: Espejamiento de la afectividad. Los infantes tienen una capacidad interna
innata de responder a los gestos de sus cuidadores. éstos responden automáticamente y en
forma exagerada a las expresiones de su bebe. Proceso por el cual el cuidador refleja la
experiencia interna emocional y la organiza, al calmar y reforzar en lugar de intensificar sus
emociones. Cuidadores: fn calmante. Constituye un papel importante en el desarrollo de procesos
de control y regulación del afecto.
Fallas: percibe la emoción negativa como real, desregulación afectiva.

42
2) Parentalización reflexiva: cuidado atento y flexible. La atribución al niño de una mente diferente
a la propia favorecerá al desarrollo de la capacidad del yo para aprender a discriminar interior-
exterior.
Capacidad de los padres para percibir cambios en los estados del niño. Cuidado sensible.
3) Cambio desde realidad psíquica dual a singular: Los niños pequeños operan con modelos no
mentalisticos (teológicos de la conducta). La conducta de los objetos se interpreta por lo que se
ve, no infiriendo. Este modo de funcionamiento se llama equivalencia psíquica (igualdad de
interior con exterior). La realidad mental se equipara a la realidad exterior.
Se desarrolla la modalidad del como si. Sentimientos e ideas como representacionales y
simbólicos.
Luego, se simboliza la experiencia afectiva a través del lenguaje. 4to año de vida, establecimiento
de la teoría de la mente. Posibilitado por la función reflexiva de sus otros significativos.
La realidad interna y externa están relacionadas pero se acepta que son diferentes.
3- Origen del trastorno mental desde la teoría de la mentalización.
Los trastornos mentales aparecen cuando la mente malinterpreta la experiencia que uno tiene de
sí mismo y de los demás, en la medida en que inferimos una imagen mental de los otros a partir
de experiencias que tenemos de nosotros mismos.
En situaciones de maltrato, de forma defensiva el niño inhibe su capacidad de mentalizar para
evitar la representación de actitudes manifiestamente hostiles,
El trauma puede generar hipersensibilidad a situaciones de apego y un modo inapropiado de
buscar intimidad con otros.
4- Componentes o dimensiones de la mentalización (multidimensional)
a. Mentalización controlada/ automática: La mentalización automática o implícita alude a un
procesamiento rápido, tiende a ser reflejo y que exige poca conciencia. Asociada al apego
seguro. La mentalización controlada refleja un proceso lento y secuenciado, ligado generalmente
al lenguaje (exige reflexión y atención). Se utiliza cuando son evidentes errores y malentendidos
en la mentalización, y la interacción requiere atención.
b. Mentalización sobre otros/ sobre uno mismo: Implica la capacidad de mentalizar nuestro propio
estado o el de otras personas. Debe haber equilibrio entre ambas. Cuando predomina sin
equilibrio, el foco en el otro, se tiende a una mayor susceptibilidad, al contagio emocional. Se
asocia en estos casos, a la precisión en la lectura de las mentes ajenas sin ninguna comprensión
real del propio mundo interior. El foco en uno mismo: se puede relacionar con la
hipermentalización del propio estado, o un interés o capacidad limitada para percibir estados
ajenos.
c. Mentalización estados internos/ externos: Mentalizar puede implicar tener que hacer inferencias
sobre estados internos a partir de indicadores externos (por ej. expresión facial) o a partir de lo
que conocemos de esa persona y su contexto.Foco Interno: alude a la capacidad de formular
juicios sobre el estado mental basándose en los estados internos (Ej., “Por lo que ha ocurrido
otras veces imagino que estás dolido”). Se aplica tanto a los otros como a uno mismo. Puede
asociarse a la hipermentalización acerca de posibles motivaciones y estados mentales de los
demás y de uno mismo. Foco externo: Mayor sensibilidad a la comunicación no verbal. Tendencia

43
a hacer juicios basándose en las percepciones y características externas. Saca conclusiones de
un gesto exterior.
d. Mentalización cognitiva/ afectiva: La mentalización cognitiva implica capacidad de razonar,
reconocer y nombrar estados mentales. La mentalización afectiva supone la capacidad de
comprender los sentimientos que acompañan estos estados. Ambas se complementan. En
momento de estrés una interfiere a la otra.
5- Modos no mentalizadores (pre mentalísticos)
-Equivalencia Psíquica: Madurativamente es propia de niños entre dos y tres años de edad.
Equivalencia mundo interno- mundo externo.La realidad mental se equipara a la realidad
exterior.Hay sólo una manera de ver la realidad: lo que existe en la mente existe en la realidad y
viceversa. Siente que los pensamientos son reales. No pueden aceptar distintos puntos de vista:
en tanto pensamiento y realidad no se diferencian. Se perspectiva es la única posible.
-Modo Teleológico: entendimiento de los demás y de uno mismo desde el punto de vista de las
conductas físicas, observables. (Ej., “me regaló una flor, me quiere”). Sólo se consideran
verdaderos indicadores de las intenciones de los demás los cambios en la dimensión física,
expresados a través de conductas que generan resultados observables. Pérdida momentánea de
la mentalización controlada. Focalización sólo en lo externo. Ausencia de mentalización implícita.
Abuso de mentalización para controlar a otros (fines teleológicos). Se da cuando el foco se centra
exclusivamente en lo externo.
-Modo Simulado o Como Si: Disociación total entre los estados mentales y la realidad física, entre
lo interno y lo externo. Las ideas no tienden un puente entre la realidad externa y la interna. El
discurso del paciente le parece al oyente vacío, carente de significado, inconsecuente y circular.
Con frecuencia los afectos no se corresponden con el contenido de los pensamientos. Pobre
razonamiento sobre las creencias-deseos y vulnerabilidad a la fusión con los demás.
6- Confianza epistémica vs desconfianza epistémica.
Confianza epistémica: Enfatiza la importancia social y emocional de la confianza que depositamos
en la información que nos transmiten otras personas sobre el mundo social. Las señales
ostensivas promueven la confianza epistémica, que a su vez suscita un tipo especial de atención
al conocimiento que me parece relevante para «mí». Permite entender información nueva.
Desconfianza epistémica: En los pacientes con TP tiende a predominar la desconfianza
epistémica, esto hace que tiendan a no poder aceptar e incorporar experiencias o informaciones
nuevas. Por eso proponen pensar al TP como un fallo en la comunicación. El individuo con una
historia traumática tendrá pocas razones para confiar y rechazará información que no sea
consistente con sus creencias (En terapia serán de difícil acceso).La persona no puede cambiar
porque es incapaz de aceptar nueva información.
7- Tres sistemas de cambio de comunicación. Relación con las intervenciones en el tratamiento
(MBT)
Sistema de comunicación 1. Comunicación del modelo terapéutico basado en el contenido.
Fundamental en primera etapa de tratamiento: Se informa cómo se trabajará. Identificación de
vulnerabilidades, discusión diagnóstico, revisión patrones apego, psicoeducación, objetivos con
paciente. Suministra modelo de mente y comprensión de su trastorno, se siente reconocido como
agente para tomar decisiones (entendido y reflejado)

44
Sistema de comunicación 2. Mentalización como factor común. REEMERGENCIA DE LA
MENTALIZACIÓN: Genuina postura de no saber. Exploración desde la perspectiva del paciente.
Validación empática. Contexto de apego para explorar. El paciente tiene en cuenta el punto de
vista del terapeuta y viceversa. Se espera acceda a nueva información.
Sistema de comunicación 3. Aprendizaje social en contexto de confianza epistémica.
Reemergencia de aprendizaje social con una mentalización mejorada: Mejora de la comprensión
de las situaciones sociales propiciada por la mejora de la mentalización, conduce a la posibilidad
de comprender a otras personas importantes en la vida del paciente. Focalización en respuestas
sensibles de los demás y se sienta entendido. Reconocimiento de que las respuestas negativas
no son más que eso. Poder tolerarlas. Las respuestas sociales pueden ser fuentes de aprendizaje
(actualizan conocimiento de si y de los demás). Cambio es posible si puede utilizar al entorno
social de manera positiva. Favorecer la auto-agencia (sentirse dueño de sí). Poder anticipar cómo
puede sentir ante situaciones.
-Vernengo, M. P.; Stordeur, M. (2016). Regulación afectiva y psicoterapia psicoanalítica. De la
investigación a la clínica. En VIII Congreso Internacional de Investigación y Prácticas
Profesionales en Psicología, 8, 177-181. Recuperado de https://www.aacademica.org/000-
044/223.pdf
Regulacion afectiva: es la capacidad de controlar y modular nuestras respuestas afectivas. Se
relaciona con un buen contacto consigo mismo, con el entorno y la capacidad de simbolizacion.
Se va a analizar la relevancia de la Regulacion afectiva en las psicoterapias psicoanaliticas,
respecto al vinculo terapeutico y al proceso terapeutico en si.
AFECTO Y NEUROCIENCIAS:
Las emociones son percepciones de estados corporales, estan mediadas por el sistema nervioso
autonomo. Se incluyen dentro de los afectos a las emociones y los sentimientos.
Emociones= asociadas a procesos biologicos.
Sentimientos= es la experiencia subjetiva de los cambios dados por las respuestas emocionales.
Los afectos incluyen entonces las manifestaciones fisicas y psiquicas.
REGULACION AFECTIVA EN EL PRIMER AÑO DE VIDA
El primer desafio del niño es lograr y mantener la homeostasis fisiologica y emocional. La
regulacion afectiva es un tema importante y esencial durante el primer año de vida, requiere del
andamiaje del o los cuidadores adultos y del ambiente. El logro de una conexión emocional solida
es la base de un desarrollo saludable en el infante y una falla en ese logro puede traer
consecuencias negativas en la salud mental del niño en el corto y largo plazo.
La Autoorganizacion de la mente esta fuertemente determinada por una autorregulacion de
estados emocionales.
El adulto con su rol mediador ayuda leyendo las conductas del infante a que el niño metabolice
los estimulos de su entorno.
Es fundamental para el logro de la autoregulacion afectiva la transformacion de afectos negativos
y displacenteros en placenteros y positivos.

45
En los primeros años de vida es de suma importancia para el desarrollo del self que los padres
logren entrar en sintonía con la experiencia afectiva del niño (entonamiento afectivo), esto implica
no solo reflejar el estado emocional del infante sino tambien devolverlo de forma metabolizada,
elaborada, como algo tolerable e integrable en la experiencia—>ESPEJAMIENTO DE LA
AFECTIVIDAD, es un mecanismo esencial para fomentar la capacidad de regulacion afectiva y
organizar la experiencia emocional del infante: que siente, que quiere, que le pasa, como
calmarse (transformar el afecto negativo displacentero en positivo y placentero).
Es la devolucion de la mirada de la madre donde el niño se ve como si se mirara en un espejo, la
madre devuelve al niño su propia imagen, como lo hace? respondiendo a las necesidades del
niño, madre empatica ≠ rostro inmovil.
Los progenitores y en especial quien cumple la funcion de la madre cumplen un rol especial en la
transformacion de los estados afectivos del bebe ayudando a la transicion de una regulación
diadica a una autoregulacion—> esto coincide con lo que plantea W sobre el papel de la madre y
del ambiente facilitador en la costitucion del self.
REGULACION AFECTIVA Y CONSTITUCION DEL SELF
Fonagy y otros establecen la relacion entre la mentalizacion, la regulacion afectiva y la
constitucion del self buscando integrar aspectos cognitivos y afectivos. La mentalizacion o funcion
reflexiva es la capacidad de percibir y comprender la propia conducta y la de los demas,
componente autorreflexivo + componente interpersonal que permite distinguir la realidad externa
de la externa—> que me pasa a mi y que le pasa al otro, como dos cosas diferentes.
Esta mentalizacion o funcion reflexiva se adquiere de las relaciones interpersonales tempranas
del niño de las cuales adquiere el sentido de agencia, donde se reconoce a él mismo como
agente de determinada accion.
La regulacion afectiva tiene distintos niveles, el mas basico: la homeostasis. El mas complejo: la
mentalizacion, que se da en relacion a otros y permite elaborar afectos y comunicarlos, este nivel
se relaciona con la emergencia del self.
AFECTIVIDAD MENTALIZADA—> es un tipo complejo de regulacion del afecto por la cual se
comprende las experiencias afectivas. Compuesta por tres procesos:
-Identificacion: Es el hecho de nombrar la emoción que uno siente
-Modulacion de los afectos: Es el ajuste de los afectos que pueden estar alterados de algun modo
-Expresion de los afectos: puede ser interior o exterior.
Segun Fonagy y otros autores fallas en la mentalizacion y por consiguiente en la regulación
afectiva estan asociadas con Trastornos de la personalidad.
ESTILOS DE APEGO Y REGULACION AFECTIVA
El Sistema de Apego es un regulador de la experiencia emocional cuyo fin es el sentimiento de
seguridad, el niño aprende que la presencia del cuidador actua como regulador de la experiencia
evitando una desorganización que va mas alla de su capacidad de afrontar, reestablece el
equilibrio. La regulacion afectiva es fundamental para la teoria del apego ya que la relacion entre
el niño y su cuidador constituye en si un vinculo afectivo.
Conexion entre estilos de apego y regulacion afectiva:

46
-Un niño con apego ansioso evitativo/elusivo tendera a una minimizacion de los afectos- esta
sobreregulado
-Un niño con apego ansioso resistente, incrementa los afectos, el afecto se expresa en demasia -
hay una subregulacion
-Un niño con apego seguro tiene una regulacion afectiva abierta y flexible. La autoregulacion
surge de la confianza en el cuidador que se traduce en confianza en el propio self con el cuidador
y despues en confianza en el propio self.
Los deficitis en la mentalizacion y tendencia a la desregulacion afectiva son carateristicos de
personas con patologias borderline, no hay posiilidad de asumir la agencia, la intencionalidad en
las acciones o pensamientos, hay limiter borrosos entre lo externo y lo interno—> las
intervenciones apuntan a discriminar.
REGULACION AFECTIVA Y VINCULO TERAPEUTICO
Todo proceso terapeutico supone desarrollar procesos de autocontinencia—> desde la
continencia y sosten del terapeuta a procesos de autoreflexion y contencion. Los procesos de
regulacion afectiva son importantes en la clinica porque definen el diagnostico y las estrategias y
abordajes terapeuticos. W. planteaba la importancia del vinculo y de la experiencia del vinculo en
el proceso terapéutico como valor curativo. Para Fonagy la psicoterapia tiene el objetivo de
modificar la relacion del paciente con su propios afectos, cuanto mas severa es la patologia
menor es la regulacion de la afectividad por desborde o inhibición de la afectividad—> la
psicoterapia psicoanalitica favorece la afectividad mentalizada.
Hay una regulacion afectiva en el vinculo terapeutico, es el ejercicio de un modo de conectarse
con el otro diferente a los modelos de relacion que proponian los cuidadores en la temprana
infancia para “resolver” esos modos tempranos que generan trastornos. Los trastornos de la
personalidad pueden entenderse como fallas en la regulacion afectiva.

TEORICO SEMANA 4
- Lemma, A ; Target, M y Fonagy, P. Terapia Dinámica Interpersonal Breve. Guía Clínica (2018).
Chile: Editorial Mediterráneo. Capítulo 2: ¿Por qué indicar terapia dinámica interpersonal?
Aproximaciones psicodinámicas y clasificación diagnóstica
Un enfoque dirigido fundamentalmente por una formulación psiquiátrica puede ir contra el
énfasis psicoanalítico tradicional de centrarse en el inconsciente y en los psicodinamismos
individuales de cada caso. Habitualmente nuestro interés primordial es la persona que está
experimentando depresión o ansiedad, más que el diagnóstico en sí mismo. en otras palabras, lo
que consideramos vital en cualquier intervención terapéutica es comprender los síntomas en el
contexto de la estructura de personalidad: por ejemplo, un individuo narcisista y un individuo de
personalidad evitativa experimentan los ataques de pánico de maneras muy diferentes. Tener
presente el contexto de la personalidad.
Cuando los terapeutas psicodinámicos incorporan y utilizan sistemas de clasificación, lo
hacen como una concesión al pragmatismo (registros, informes, reembolsos, investigación, etc),
pero sienten que el uso de categorías diagnósticas tiene el potencial de eliminar lo esencial de la
psicoterapia. La tensión parece no residir tanto en diagnosticar per se útil o no, sino más bien en

47
torno a qué sistema usar. La diferencia entre los dos enfoques de clasificación se encuentra en la
lógica mediante la cual cada uno llega a sus conclusiones. Como señalaban Drew Wester y
Jonathan Shedler (2004), la identificación de una lista definitiva de síntomas y signos no calza
bien con el pensamiento clínico psicodinámico. Este último se basa en categorías derivadas de
prototipos emergentes. Los psicoterapeutas no piensan en la presencia de rasgos necesarios y
suficientes para categorizar, sino en que tan parecido es un individuo a un tipo ideal que tal vez
nunca se ha visto, pero se basa en la acumulación de los casos clínicos tratados y estudiados.
En comparación con la formulación psicodinámica y la decisión sobre la indicación
terapéutica, el diagnóstico ha recibido poca atención en los programas de formación en
psicoterapia. La clasificación del RDoC se basa en tres supuestos:
1. Los trastornos mentales son trastornos de los circuitos cerebrales.
2. mediante las herramientas de las neurociencias es posible identificar dichos trastornos
3. Los datos ofrecerán biofirmas útiles para el manejo clínico de los casos.
Resulta sorprendente que este enfoque sea más cercano al enfoque mecanicista del que Freud
fue pionero, aunque los mecanismos a los cuales apunta el RDoC no son psicológicos sino
fisiológicos.
Salvo que adoptemos una postura dualista, deberíamos esperar que un sistema
psicológico mecanicista sea confiable y sólido. Esto es definitivamente cierto para dos constructos
fundamentales en TDI: el apego y la mentalización, y se cumple para otros aspectos de la
depresión y la ansiedad como son los déficits de la memoria.
¿Dónde se sitúa la TDI en estos debates? Desde nuestro punto de vista, el diagnóstico de
trastorno ansioso o trastorno depresivo mayor (TDM), constituye sólo una (pequeña) parte de una
formulación y plan de tratamiento completo. Nosotros asumimos y recomendamos un enfoque
integrado en el cual los clínicos incorporen en sus formulaciones tanto el enfoque diagnóstico de
investigación como el enfoque psicodinámico.
Trastorno del ánimo: depresión y ansiedad.
La gran mayoría de nuestros pacientes con trastornos del ánimo que busca psicoterapia en
servicios ambulatorios y de atención primaria están aquejados por sintomatología tanto depresiva
como ansiosa. Además, pueden presentar varios otros síntomas diagnosticables, que en
ocasiones se han desarrollado en paralelo con el trastorno del ánimo.
En TDI pensamos que los pacientes del espectro de los trastornos del ánimo, que tienen
diversas combinaciones de etiquetas diagnósticas, comparten una tendencia a organizar las
experiencias sociales de acuerdo con expectativas inconscientes subyacentes con respecto a sí
mismo y los demás que desencadenan afectos específicos. Una vez que están activas en la
mente, dichas configuraciones interpersonales (por lo general inconscientes) conducen a
estrategias defensivas mentales y conductuales que a su vez son desadaptativas. En todo el
espectro de los trastornos del ánimo aplicamos la estructura de focalizar en una FAI, siempre que
no exista otro tipo de tratamiento que se requiera como más urgencia.
La depresión es una condición común y a menudo compleja que habitualmente se
manifiesta tempranamente en la vida: 40% de las personas deprimidas experimentan un primer
episodio antes de los 20 años. Los trastornos de ansiedad también son prevalentes y al igual que
en la depresión, tienden a iniciarse en la niñez. La etiología de la depresión y la ansiedad no se
ha comprendido totalmente, pero probablemente está determinada por un conjunto de procesos

48
psicológicos, sociales y biológicos. como señalamos previamente, también se ha demostrado que
es común que las personas deprimidas tengan un diagnóstico de comorbilidad psiquiátrica (por
ejemplo, ansiedad, abuso de alcohol y/o diversos TP). Al considerar el enfoque de la TDI, es
importante tener presente que se ha demostrado que cuando los pacientes presentan una
comorbilidad de depresión y trastornos ansiosos, la ansiedad habitualmente ha precedido a la
depresión y puede ser, por lo tanto, un foco subyacente de trastorno del ánimo. Los pacientes que
cumplen con los criterios para un TDM tienen nueve veces más probabilidades por sobre el azar
de cumplir con los criterios para otras condiciones.
En paralelo con la evidente complejidad de la depresión y la ansiedad, ha prevalecido en el
sector público de salud un enfoque aparentemente simplista a nivel de la provisión de servicios,
donde el énfasis actual en la práctica basada en la evidencia ha privilegiado la TCC como el
tratamiento de elección para ambos trastornos. Ningún tratamiento por sí solo es la respuesta
para todo el mundo y debieran seguir estando disponibles diversos enfoques que tengan alguna
evidencia de efectividad.
La TDI plantea que los síntomas presentes en los trastornos del ánimo son respuestas a
dificultades interpersonales y/o amenazas percibidas a las relaciones de apego
(pérdida/separación) y por lo tanto, percibidas también como amenazas al self. Este enfoque
conceptualiza la depresión y la ansiedad con bajo ánimo como la desorganización transitoria
subyacente del sistema de apego, causado por problemas relacionales actuales lo que genera
una variedad de distorsiones de pensamientos y sentimientos que es típica de los estados
mentales depresivos y ansiosos crónicos. La terapia mantiene un poco en esta crisis emocional
por medio de una elaboración de pensamientos y sentimientos (CC e ICC). Por medio de la
exploración focalizada en la relación transferencial se le ayuda al paciente a comprender mejor
sus relaciones subjetivas a las amenazas. Mejora la capacidad del paciente de reflexionar sobre
sus pensamientos y sentimientos y los de los demás. Permitiendo explicitar ansiedades y
preocupaciones implícitas, lo que mejora su capacidad de enfrentar las amenazas y desafíos
interpersonales presentes relacionados con el apego.
El punto de inicio de la TDI se apoya en la observación clínica habitual, si bien el paciente
puede presentar un problema como “no puedo dormir ni concentrarme” o “no me atrevo a ir a
lugares llenos de gente ni al trabajo” la terapia TDI reformula tales sintomas de depresion y
ansiedad como las manifestaciones de una problemática relacional que le paciente no logra
comprender o comprender de una manera poco adaptativa. Una vez que se ayuda al paciente a
realizar algunos cambios en la forma de abordar sus dificultades relacionales, por lo general se
observa un alivio de sus síntomas depresivos y ansiosos. Varias características de los trastornos
del ánimo sugieren que un enfoque de atención dinámica centrado en temas interpersonales
probablemente será efectivo en el abordaje de los síntomas.
El trabajo de investigación de algunos psicoanalistas demuestra que además de que la
vulnerabilidad a la depresión se relaciona con la generación inconsciente de estrés interpersonal
de la depresión una importancia comparable a las dimensiones interpersonales, en las que
generalmente se hace más énfasis. El modelo de la triple vulnerabilidad de los trastornos de
ansiedad a los trastornos de ansiedad además del substrato biológico existirían factores de
vulnerabilidad psicológica generalizada, acompañadas por una vulnerabilidad psicológica
específica causada por eventos interpersonales desencadenantes.
Como enfoque la TDI incluye prestar atención a disfunciones evidentes en la cognición
interpersonal relacionadas con la comprensión distorsionada e inadecuada e inadecuada de los

49
pensamientos y sentimientos de los demás por parte de un individuo (es decir, la mentalización).
La medida de mentalización en el contexto del apego también indica que la depresión se asocia
con un déficit en la mentalización. Esto es relevante ya que la TDI supone que las fallas en la
comprensión de sí mismos y del otro que se observan en la depresión están relacionadas con
patrones específicos de interacción self otro que han revolucionado a partir de experiencias
infantiles.
La TDI tiene un foco dual en los temas interpersonales y afectivos. Los temas afectivos de
mayor relevancia se centran en cuestiones relacionadas con el apego. Hay un cuerpo
considerable de trabajos que vinculan la vulnerabilidad a la depresión con el apego. La teoría de
la depresión de Blatt identifica dos tipos de esquemas cognitivos - afectivos basados en la historia
de apego que es más probable encontrar en casos de depresión, dependencia interpersonal y
autocrítica excesiva, que se vinculan con los patrones de apego preocupado y evitativo
respectivamente. Durante décadas se ha ido acumulando evidencia que vincula la adversidad en
la niñez con la vulnerabilidad del adulto a la depresión.
La experiencia de apego también vinculan la mentalización y la depresión. Una capacidad
reducida de pensar acerca de los estados mentales puede estar relacionada con historias
personales pero puede ser también una consecuencia secundaria de un trastorno del ánimo. La
tarea terapéutica es ayudar al paciente a laborar ese estado mental que en lugar de estar
disponible para poder analizarlo y reevaluar lo como si fuera una creencia o un pensamiento, se
experimenta como un síntoma físico. La falta de pulsión, qué es la esencia de la depresión, sería
similar a la corporal ización regresiva de un pensamiento que tiene un efecto debilitante.
Del mismo modo, la intensidad de las preocupaciones acerca del futuro, y la cualidad
abrumadora de la autoculpabilidad Sión asociada la experiencia pasada, implica una pérdida
subyacente de perspectiva simbólica a raíz de la cual el pensamiento adquiere una fuerza
concreta inapropiada porque se le atribuye el mismo estatus que a la realidad física. La ausencia
de la capacidad de reflexionar sobre la experiencia, el autocuestionamiento se transforma en un
ataque persecutorio interminable y sin solución contra la representación del self.
La formulación de la depresión y la ansiedad que se propone como sustento de TDI
considera que las distorsiones en la cognición indican diversas fallas en la mentalización que
pueden corresponder a pensamiento concreto o indicadores de pseudo mentalización (también
llamado hiper mentalización). Es importante señalar que en este último caso la descripción de los
pacientes de los estados mentales de los demás bolsillo propia refleja un pensamiento
aparentemente reflexivo, pero en este caso se exacerba los pensamientos depresivos y ansiosos
mientras que en una auténtica reflexión sobre uno mismo normalmente conduce un alivio de los
estados de ánimo negativo.
De la pertinencia de la indicación de terapia dinámica interpersonal.
Nuestra capacidad de evaluar de manera confiable "qué funciona con quién" sigue siendo
limitada. Efectivamente cuando se trata de evaluar la pertinencia de la indicación, la capacidad de
reconocer que las características previas del paciente no predicen en forma significativa el
resultado de la terapia psicodinámica - independientemente de su "marca" específica - es una
competencia fundamental.
Ciertas dimensiones de la experiencia de los pacientes (intrapsíquica, interpersonal y pragmática)
son pertinentes al momento de evaluar si la TDI puede ser indicada o no para ellos, y qué
adaptaciones habría que hacer para satisfacer las necesidades de cada persona. Así, cada uno

50
de los siguientes dominios ofrece un punto de vista parcial y desde el cual considerar la
pertinencia de la indicación de TDI. Tomados en conjunto equivalen a orientaciones para evaluar
la pertinencia de dicha indicación, y no recomendaciones basadas en la evidencia.
Dimensiones a explorar cuándo se evalúa la pertinencia de la indicación:
● Interés del paciente por trabajar temas interpersonales y afectivos.
● Capacidad del paciente de reflexionar sobre la relación terapéutica.
● Rasgos para el paciente de contactarse con sentimientos y o temas interpersonales
dolorosos.
● Apoyo externo disponible para el paciente durante la TDI
● Respuesta del paciente a una sanación exploratoria y experiencia que el terapeuta tiene
del paciente.
La respuesta del paciente a una aproximación exploratoria.
La terapeuta se aproximaban terapéutica con una actitud analítica para observar la interacción
del paciente con ella y evaluar qué adaptaciones podrían ser necesarias para apoyar la capacidad
del paciente de trabajar dentro de un encuadre analítico. Esto nos llevan nuevamente a la
importancia fundamental de adoptar una aproximación flexible que priorice evaluar dónde está el
paciente en vez de tratar a toda costa de hacerle calzar con un modelo que no corresponde a sus
necesidades.
Hecho de que la terapeuta TDI adopte una actitud analítica plantea demandas especiales al
paciente, lo que puede hacer que este modelo sea inapropiado o insatisfactorios para algunas
personas. Por lo tanto, una señal importante de la pertinencia de la indicación es la forma en que
el paciente actúa cuando la terapeuta no impone una conducción sistemática de la sesión Y hasta
qué punto el paciente responde a la invitación a reflexionar sobre lo que está ocurriendo en su
vida en vez de buscar alivio de los síntomas por medio de estrategias prácticas.
El interés del paciente por trabajar con temas interpersonales y afectivos.
Todos los pacientes acuden a una evaluación con su propio lenguaje y marco de referencia
acerca del malestar emocional, con sus propias teorías consonantes con sus estilos culturales de
expresar dicho malestar. Es posible que el énfasis interpersonal y afectivo de la TDI le haga
sentido y le resulte útil al paciente, y este pueda pasar, por ejemplo, de su explicación bioquímica
previa a una explicación más psicológica. Sin embargo esto no siempre ocurre, y es importante
escuchar atentamente si la narrativa del paciente se corresponde con la TDI buscamos una cierta
compatibilidad entre la lógica del tratamiento y las teorías del paciente.
La TDI se centra inequívocamente en el funcionamiento interpersonal del paciente en su vida
actual, en la interacción terapéutica, y en su experiencia emocional.
La capacidad de reflexionar sobre la relación terapéutica.
Esperamos que nuestros pacientes sean capaces de desarrollar una relación emocionalmente
viva con nosotros. Ella despertar a una variedad de sentimientos positivos y negativos, algunos
de los cuales pueden resultar aterradores. El contacto del paciente con la realidad, y la capacidad
asociada de percibir la cualidad ficticia de la transferencia es, por lo tanto, un prerrequisito
fundamental. Para que el paciente pueda usar y beneficiarse de la TDI es importante que se le
pueda ayudar a reflexionar sobre la relación terapéutica y que al mismo tiempo mantenga el juicio
de realidad. Cuando no es posible distinguir el símbolo de lo simbolizado, esto refleja un quiebre

51
en el funcionamiento simbólico que psíquicamente desbastador y constituye una contraindicación
para la TDI.
Una de las señales más significativas de la pertinencia de la indicación es la respuesta del
paciente a los intentos de la terapeuta de entenderlo. La respuesta del paciente ayudar a hacerse
una idea de la distorsión y motivación de la persona para trabajar en el foco afectiva interpersonal
de la terapia en ese momento, y más específicamente, ayudará a determinar hasta qué punto el
paciente puede:
● Tomar distancia de su experiencia y observar su propia mente.
● Recibir y aprovechar lo que podemos ofrecerle
● Trabajar en torno a un foco
La motivación y actitud del paciente acerca de su propio rol en sus dificultades
La terapeuta TDI ofrece apoyo más explícitamente que en otros modelos psicodinamicos. Sin
embargo, también es desafiante porque la TDI pone el énfasis en ayudar al paciente a hacer
cambios reales la forma en que se relaciona con los demás. Esto requiere una gran motivación de
parte del paciente, en especial cuando los síntomas son egocéntricos (es decir no generan
conflicto). Durante la Terapia los pacientes atraviesan por períodos en que su motivación es alta y
en otros momentos prevalecen las ganancias secundarias de la enfermedad y la motivación
decrece. Un factor importante a evaluar es el predominio relativo de una motivación al cambio por
sobre una gratificación inconsciente a partir de los síntomas que actúa como resistencia.
El riesgo que tiene para el paciente contactarse con sentimientos y/o además interpersonales
dolorosos que podrían resultar le difíciles de manejar se equilibra con los beneficios de explorar
esos temas en terapia.
Sí bien se promueve que la terapeuta adopte una actitud más apoyadora que la qué es
característica de algunos modelos psicodinamicos, la terapia a borra también el conflicto y las
defensas inconscientes. Es importante evaluar el riesgo al que puede verse expuesto el paciente
por este énfasis exploratorio. Por lo tanto una tarea fundamental de la evaluación es considerar
en conjunto con él, y en forma realista, capacidad de trabajar en un encuadre en el lítico de
tiempo limitado. Esto requiere algunos conocimientos y una formulación En las siguientes áreas:
● Conocimiento de la historia de autoagresión o conductas autodestructivas del paciente
● Evaluación de las fortalezas yoicas del paciente. La fortaleza yoica de un paciente se
infiere a partir de la forma en que se presenta en la evaluación. Están refleja los recursos
de su personalidad que le permitirán sobreponerse a las ansiedades y desarrollar defensas
más adaptativas. La debilidad yoica se manifiesta como una baja tolerancia a la frustración
y escaso control de impulsos, baja tolerancia la ansiedad y ausencia de actividades
sublimatorias. El paciente con mayor fortaleza yoica será capaz de pensar acerca de su
rabia y lograrán sublimarla y canalizar la hace una actividad más constructiva (ej.
Ejercicio).
● La capacidad del paciente de persistir con sus relaciones y proyectos laborales o
vocacionales mientras enfrenta desafíos nos ofrece una oportunidad de evaluar
indirectamente la fortaleza yoica.
Los recursos externos que podrían ser un apoyo para el paciente durante el curso de la TDI.
Aquí es importante tener presente que no estamos evaluando si el paciente calza con el modelo,
sino hasta qué punto el modelo y el contexto en que se ofrece pueden adaptarse para satisfacer

52
las necesidades del paciente. Por lo tanto, un rasgo central de la evaluación es la capacidad de
identificar y tomar nota de los recursos externos disponibles para el paciente y el terapeuta
cuando se planifica la intervención.

La experiencia que la terapeuta tiene del paciente durante la sesión.


La Experiencia directa de la terapeuta de estar en la sala de atención con el paciente
probablemente sea el indicador más informativo de todos. Entonces pueden encontrarse paciente
que "en papel" podrían parecer candidatos pocos apropiados para la terapia dinámica
interpersonal y en la interacción con el terapeuta se genera un destello de Esperanza que indica
que con el ritmo adecuado está terapia podría ser beneficiosa o podría funcionar como puente
con una psicoterapia psicodinámica de largo plazo para pacientes que aún no están preparados
para aprovechar esa intervención.
La evaluación de la terapeuta de lo que el paciente puede tolerar Y aprovechar se deduce
también de la reflexión que ella hace sobre el impacto emocional que la presentación del paciente
tiene en ella es decir su contratransferencia.

-Rao, AS, Lemma, A., Fonagy, P., Sosnowska, M., Constantinou, MP, Fijak-Koch, M. y Gelberg,
G. (2019). Desarrollo de la terapia interpersonal dinámica en cuidados complejos (DITCC): un
estudio piloto. Psicoterapia psicoanalítica , 33 (2), 77-98.
No entra, está en inglés.
PRACTICO SEMANA 4
Bowlby, J. (1989). El papel del apego en el desarrollo de la personalidad. En Una base segura
(pp. 140-158). Buenos Aires: Paidós.
Teoria del Apego:
-Importancia de los lazos emocionales íntimos entre individuos—> se utilizan modelos operantes
del si mismo y de la figura de apego (del otro) en la relación.
-Influye en el desarrollo del niño el como este es tratado por sus figuras de apego (padres,
especialmente la madre)
-El desarrollo del bebe y el niño puede tener distintos recorridos no hay un unico destino “normal”
La primacía de los lazos emocionales Intimos:
*La tendencia a establecer lazos emocionales intimos con otros individuos es BASICA para la
naturaleza humana, esta presente en el neonato y lo acompaña hasta la vejez.
*Durante la infancia se recurre a los padres o padres sustitutos en busca de protección, consuelo
y cuidados.
*Durante la adolescencia sana se mantienen los lazos anteriores y se agregan lazos nuevos en
gral de caracter heterosexual.
*RELACION DE APEGO—>FUNCION CLAVE PARA LA SUPERVIVENCIA= PROTECCION! El
niño necesita de un otro que lo proteja, que sea dador de cuidados. Esto es fundamental para el

53
funcionamiento de efectivo de la Personalidad y de la Salud Mental. Es decir que es básico para
la
salud mental y la personalidad que el S tenga la CAPACIDAD DE ESTABLECER LAZOS
EMOCIONALES INTIMOS CON OTROS INDIVIDUOS (sea como dador o proveedor de
cuidados)
* “Dar o buscar cuidados” es un componente basico de la conducta humana->conducta de apego.
Su antiesis es la “exploracion del entorno”, tambien componente basico de la conducta humana.
Cuando un individuo, sea cual sea su edad, se siente seguro, es probable que explore el mundo lejos de su
figura de apego—>exploracion a partir de una “base segura” (el tiempo q dure esta exploracion depende de
la edad) ≠ Cuando un individuo se siente ansioso, cansado o enfermo tiene la necesidad de mantener la
cercania con su figura de apego.
La Base segura es el lugar donde el sujeto puede volver luego de haber explorado, es esencial
para el optimo funcionamiento de la personalidad y de la salud mental.
*El bebe hasta los 6 meses tiene una capacidad innata para establecer vinculo o lazo emocional
con otros, establecer una interaccion social y siente placer al hacerlo, ademas de que lo necesita
para sobrevivir—> esto quiere decir que no existe fase autista o narcisista.
*En los segundos 6 meses del bebe, el niño tiene la capacidad cognitiva de conservar a la madre
en su mente aunque ella no este presente fisicamente—> esto es fundamental para la conducta
de apego organizada que tiene por objetivo guardar la proximidad o accesibilidad a la madre o
figura de apego.
*A los 9 meses el niño tiene la figura de la madre lo suficientemente internalizada como para
rechazar a cualquier otra figura que este a su cuidado que no sea ella.—> el niño tiene funciones
de comunicacion cada vez mas sofisticadas para regular esta proximidad/lejania con la madre (o
figura de apego): SISTEMA DE CONTROL DEL APEGO (por medio del cual se regula la
conexión con esa persona). Este sistema de control del apego mas las representaciones de si
mismo y los odelos operantes en el vinculo con el otro son fundamentales para la personalidad a
lo largo de la vida del S.
PAUTAS DE APEGO Y CIRCUNSTANCIAS QUE DETERMINAN SU DESARROLLO
La Pauta de Apego es el modo en que el niño se relaciona/vincula con su figura de apego y esta
determinada por el modo en que los padres tratan al individuo en sus primeros años de vida.
PAUTA DE APEGO SEGURO: El S confía en que sus figuras de apego serán accesibles,
estables, sensibles y colaboradores si se encuentra en una situación adversa o atemorizante->
esta seguridad le permite explorar el mundo.
PAUTA DE APEGO ANSIOSO- RESISTENTE: El S esta inseguro de si su progenitor estará
presente, accesible y sensible en caso de necesitarlo en búsqueda de protección. Esta
incertidumbre lo lleva a una separación ansiosa para la exploración del mundo, con cierta
tendencia al aferramiento.
PAUTA DE APEGO ANSIOSO- ELUSIVO: El S sabe que cuando busque cuidados no recibirá
atención, recibirá desaires en vez para compensar esta falta de atención y cuidados se
desarrollara emocionalmente autosuficiente y posteriormente puede diagnosticarse narcisista o
como poseedor de un falso si mismo.

54
Aunque la mayoria de las personas pueden identificarse en alguna de estas tres pautas, hay
personas cuyas pautas de apego son “desviadas”.
Como la madre influencia en la pauta de apego que desarrolla el niño depende de:
-El apoyo emocional que ella reciba o no mientras provee cuidados al niño
-Tipo de cuidados maternos que ella recibio cuando era niña
PERSISTENCIA DE LAS PAUTAS
Una vez desarrollada una pauta de apego, esta tiende a persistir, xq?
-Porque el modo en que el adulto trata al niño (sea bien o mal) tambien tiende a permanecer
invariable en el tiempo
-Porque la conducta que desarrolla al niño a causa de la pauta de apego tiende a reproducir la
misma, si el niño esta ansioso y demandante probablemente la madre tienda a alejar al niño aun
mas, no es un niño “amigable” a diferencia de un niño “feliz” a quien es mas fácil atender y cuidar.
Una vez que el niño crece, conserva las pautas y traspola este modo de vinculacion con otros,
esto se refleja en el vinculo con el terapeuta-> INTERNALIZACION DE LA PAUTA: El modo en
que el niño se relaciona con la madre durante los primeros 12 meses de vida, determina y ayuda
a predecir como se relacionara con otros mas tarde
UNA TEORIA DE LA INTERNALIZACION
Modelos operantes:
-De la madre
-Del Padre
-De si mismo como complemento de las anteriores en su interaccion, como soy con mama/con
papa
Estos modelos se constituyen durante los primeros años de vida y luego se transforman en
estructuras cognitivas influyentes de otros vinculos.
Como es tratado, que le dicen, que imagen tienen los padres del niño->determina el modo en que
el niño se siente con sus progenitores y como se siente consigo mismo, determina tambien la
forma en que espera ser tratado por los demas y como organiza su forma de vincularse con otros.
Este modo de interaccion con los padres permanence mas o menos estable y opera a nivel iccte
en el vinculo con otros
Para que una relacion entre individuos se desarrolle armoniosamente es necesario que haya
registro de los objetivos, intensiones, aspiraciones, expectativas, sentimientos del otro y de uno
mismo y ajustar la ppia conducta para lograr convergencia de objetivos-> esto es tener modelos
adecuados de uno mismo y del otro que se mantengan actualizados por medio de la
comunicación libre, directa y no verbal (gestos, contacto visual, etc)-> los padres de niños con
apego seguro son mas exitosos que los de niños con apego ansioso en este terreno.
CAMINOS HACIA EL DESARROLLO DE LA PERSONALIDAD
CAMINOS EN PLURAL! NO HAY UN SOLO TIPO DE DESARROLLO DE LA PERSONALIDAD!

55
hay distintos caminos potenciales de desarrollo, algunos mas saludables que otros. Este modelo
de caminoS de desarrollo indica que se abren ante el niño muchas posibilidades de desarrollo con
su nacimiento, el camino por el cual este niño avance va a estar determinado por la interaccion de
él con su entorno y sobre todo con sus padres, el modo en que este bebe es tratado por sus
padres, en especial la madre y como él responde a eso.
Padres sensibles-> probablemente se encause en un camino de desarrollo de la personalidad
saludable
Padres negligentes o rechazantes-> probablemente se desarrollo por caminos desviados.
Dado que el curso de desarrollo NO es Fijo, si hay un cambio en el modo de ser tratado, el curso
de desarrollo de la personalidad puede variar tambien hacia una direccion mas saludable o
favorable
Esta posibilidad de cambio va disminuyendo con los años, aun asi se mantiene durante todo el
ciclo vital-> esta posibilidad de cambio habilita la posibilidad de una terapia eficaz

-Killingmo, B. (1989). Conflicto y déficit - Implicancias para la técnica. International Journal of


Psychoanalysis, 70, 111-126.
Psicopatologia—> Conflicto Intersistemico entre los 3 sistemas estructurales de la Personalidad
(yo- S Yo- Ello) y la Realidad.
-Hay sistemas diferenciados
-Hay cierta diferenciación entre la representación de si mismo y el objeto
-Represión como mecanismo de defensa, lo cual implica que el S ha alcanzado cierto nivel de
desarrollo estructural.
A pesar de esto, la Psicopatología muestra que hay dos mecanismos Patológicos diferentes:
-Conflicto
-Deficit—> a ≠ de las patologías derivadas del Conflicto, en este mecanismo patológico, la falla se da
Intrasistema:
Hay una estructura Yoica defectuosa- Identidad Difusa
Falla la constancia del objeto- Falta la Capacidad de Rel Emocional con el Objeto
La Teoría del Deficit es un complemento de la teoría del conflicto para dar abordaje al amplio
espectro de manifestaciones Psicopatologicas
El terapeuta para aplicar una correcta estrategia debe primero poder diagnosticar la estructura del
paciente, si se trata de conflicto o deficit. Esto va a disparar diferentes estrategias de abordajes y
tipo de transferencia a instalar.
DOS ESTRATEGIAS TERAPEUTICAS
Patologia basada en el Conflicto
-Hay discriminación de instancias y sistemas, Yo- SYo- Ello- Realidad—> El conflicto se produce
entre ellos.

56
-El S tiene sus funciones yoicas bien conservadas, entre ellas esta la “experiencia de
intencionalidad Primaria”—> es la Representación de si mismo como Responsable de impulsos,
acciones, sentimientos.
El mecanismo de la Represión es la “artimaña” que encontró el Yo para deshacerse de esa
Responsabilidad
Patologia basada en el Deficit
-En las Patologías basadas en el Deficit, no hay Representación de Intencionalidad, no hay
experiencia de Intencionalidad Primaria, con lo cual la defensa no tiene que ver con defenderse
de la angustia que provocan las “malas acciones” que uno mismo se atribuye. En cambio, en
estos casos, en las patologías basadas en el deficit el S se defiende de la angustia de
Fragmentación, se defiende de la propia perdida de sensación de Identidad.
Esto trae consecuencias en cuanto al modo de abordaje e intervención.
I. Patología Basada en Conflictos—> Estrategia Terapéutica: Revelación de Significados
*El Analista apoya al Yo del S en la aventura de enfrentarse a impulsos y afectos arcaicos y
Representaciones Objetales que son proyectadas en el analista Esto implica una alianza entre
terapeuta y paciente para descubrir juntos el SIGNIFICADO
OCULTO DETRAS DE SUS MANIFESTACIONES, hay un significado a develar.
La alianza se forma tanto para buscar el significado del conflicto como de las Resistencias contra
el propio esfuerzo de investigacion—> Se busca develar el significado del conflicto en si y de las
resistencias que se elevan ante el acercamiento al significado del conflicto.
*Tipo de intervention: Interpretativa
Interpretaciones que aunque causan incomodidad en el paciente, éste acepta la invitación a
explorar en su mundo interno (latente) que le hace el analista y el paciente lo decodifica como
intención de ayuda.
II. Patología Basada en el Deficit—> Estrategia Terapéutica: Creación de Significados
*Los esfuerzos del analista NO apuntan a develar significados ocultos sino a ayudar al Yo a
experimentar significados, no se trata de encontrar algo sino de sentir que algo existe.
*El uso de interpretaciones puede resultar invasiva y coartar las posibilidades de instalar una
alianza y así amenazar la continuidad de la terapia.
*La estrategia terapeutica consiste en:
1ero- Corregir y Separar las Representaciones de Si Mismo y del Objeto q estan distorcionadas o
difusas
2do- Estructurar las Relaciones objetales que no se han estructurado en las etapas tempranas del
desarrollo.
*La Herramienta terapéutica es la Intervención Afirmativa
Por medio de esta intervención afirmativa se busca enfatizar alguno de estos elementos:
-La Existencia del Sujeto

57
-La Relación entre el sujeto y un objeto
-El Valor del S.
-La validez de la experiencia del paciente
Dependiendo del caso particular uno o mas elementos deben ser confirmados.
Otros autores llaman esta intervención Empatica o Empatica Reconstructiva, son Intervenciones
que buscan dotar de Significación la experiencia del Sujeto. Afirmar lo que es sujeto vivio o
experimentó para dotarle una significacion—> Por Ej: Eso que Ud vivio le debe haber resultado
tremendamente perturbador.
DOS PATRONES DE TRANSFERENCIA
Transferencia de Conflicto
*Consiste en la Repetición de necesidades y afectos dirigidas a la Representación del Objeto en
la persona del medico.
*Hay Representación del Objeto (el objeto y sus funciones esta internalizado), con lo cual el S es
relativamente independiente de la presencia y gratificación directas del Objeto. Esta
representación permite una relación funcional con el objeto.
Transferencia del Déficit
*Consiste en la Repetición de necesidades dirigidas a objetos que no han sido internalizados (no
hay representación del objeto).
Al no haber representación del objeto hay dependencia compulsiva a la presencia y gratificación
directa del Objeto.
Hay una dependencia compulsiva a la aprobación del Objeto.
En la manifestación clínica se puede encontrar en el mismo paciente ambos tipos de
transferencias, son dos cualidades de transferencias mas que dos tipos opuestos.
IMPLICANCIAS CLINICAS
Señales clínicas para detectar los derivados del deficit:
-Invitar a buscar significado es una tarea que implica un esfuerzo mental continuo y dirigido a ese
objetivo puntual—> esta no es una tarea atractiva para un paciente con patologia del deficit
-La calidad de la transferencia es un buen indicador de la estructura del paciente. Transferencia del deficit ≠
Transferencia del Conflicto
-En la patología del deficit falta variedad de afectos y fantasías que acompaña la orientación objetal
desarrollada ≠ de la patologia del conflicto que es muy florido todo el repertorio de afectos y fantasias
alrededor de la eleccion de objeto.
-En los pacientes con patología del deficit hay un claro estilo directo, la franqueza de quien no
tiene nada que esconder.
-Falta de Intencionalidad Primaria, falta de sentido de responsabilidad por impulsos, acciones y
sentimientos.
-Estilo emocional sin intermedios, polos opuestos o todo o nada.

58
-En conflicto hay responsabilidad compartida entre paciente y analista para el trabajo analítico y
las consecuencias anímicas que el trabajo desencadena. En Deficit, el paciente puede claramente
ubicarse en lugar de victima ya que no se siente implicado en el trabajo analítico, siente que no
tiene la obligación de participar en el.
-Afirmacion como modo de Intervencion tiene los sgtes efectos:
*Objetivación: Por medio de la afirmación se da sustancia/ forma al S, se le da existencia.
*Justificación: Por medio de la relación Causa-Efecto que impone la afirmación, se disminuye la
confusión en el paciente
*Aceptación: El analista por medio de la afirmación transmite aceptación del paciente y esto
favorece que el se acepte a si mismo.
-Los objetivos de la Afirmación como intervención son:
*Permitirle al analista conectarse con el paciente
*Crear una situación emocional optima en donde alguna falla empatica del analista permita
motivar la internalizacion del analista como objeto- Falla optima que deberia haber operado en el
desarrollo emocional temprano inscribiendo al objeto en el interior del S.
*Reducir la dependencia compulsiva a la aceptación del analista
Estos objetivos, una vez logrados deberian permitir al paciente mutar desde la actitud de
exigencia a la observación/examen de la propia actitud de exigencia. Al satisfacer la necesidad de
significado, el Yo del S puede adoptar una actitud mas introspectiva aunque sea
momentaneamente, elevar su nivel de funcionamiento.
+ Se pueden Sumar Intervenciones Interpretativas para interpretar resistencias/ sacarlas a la luz,
interpretar fantasias del paciente y reinterpretar la realidad.
-Se hace una transferencia de la madre no empatica al Analista, esto se exterioriza con
agresividad por la falta de capacidad del analista/ madre no empatica de ayudar al paciente a
cambiar su estado de privacion original—> esta resistencia debe ser trabajada, interpretada y
mostrada al paciente
-La cualidad de la transferencia puede cambiar repentinamente, el analista debe estar en estado
de receptividad constante para percibir este cambio y ajustar su estrategia terapeutica según
corresponda
CARACTERISTICA DE LA RELACION TERAPEUTICA:
*La relacion terapeutica es fundamental para impulsar el progreso terapeutico
*DISPONIBILIDAD EMOCIONAL (Concepto de Bowlby)
Mostrarse disponible, para el paciente esto significa no estar solo, aislado. Esto es necesario para
producir algun cambio.
El terapeuta debe brindar comprension empatica sobre cómo debe haber sido no haber recibido
el reconocimiento añorado cuando mas se lo necesitaba y que eso es lo que causo que hoy se
sienta como se siente—> esto conecta al analista con la necesidad mas urgente del paciente:
necesidad de sentirse “yo soy” o “tengo derecho de ser”

59
Reconocimiento de la falta y del derecho de tener aquello que falta + construccion de estructuras .
*Actitud del analista (tanto para trabajar con patologias derivadas del deficit como del conflicto):
-Tolerancia
-Paciencia
-Objetividad
-Estabilidad
-Neutralidad para enfrentar la transferencia, es la actitud que como analistas nos permite
identificar conflictos

TEÓRICO SEMANA 5
- Etchevers, M., Giusti, S. y Helmich, N. (2017). Revisión de las nociones de la Alianza
Terapéutica, sus antecedentes conceptuales y aportes a la investigación. Revista Universitaria de
Psicoanálisis, 17, 57-67. Disponible en
http://www.psi.uba.ar/investigaciones/revistas/psicoanalisis/trabajos_completos/revista17/h
elmich.pdf
1- Antecedentes conceptuales de la Alianza Terapéutica (la transferencia según Freud, Klein,
Bion y Winnicott)
Desarrollado por Sigmund Freud (1912, 1915, 1917) es introducido para referirse a la repetición,
que se produce en el tratamiento, de modelos de relación infantiles (clisé), dando lugar a la
redirección inconsciente de los mismos de una persona a otra. Freud (1912) conceptualiza la
transferencia en su aspecto positivo (motor del tratamiento) y negativo (obstáculo). En un primer
artículo de 1912, sostiene que la transferencia deviene resistencia al tratamiento no sólo cuando
es de carácter negativo (hostil) sino también cuando es de carácter positivo erótico. Toda
transferencia proviene de una fuente de carácter sexual e infantil; en este sentido, siempre
involucra la reedición de un amor infantil (edípico) en la cual el analista es amado por lo que
representa, dando lugar a lo que Freud (1915) conceptualiza como ―neurosis de transferencia".
En su aspecto positivo, es esperable que ese amor se exprese de forma moderada habiendo sido
sometido al proceso que denomina sublimación, siendo así que bajo esta forma la transferencia
constituye la herramienta fundamental con la que cuenta el analista para poder conducir el
tratamiento.
M. Klein entendía a la transferencia como una reproducción en la figura del analista, de todos los
objetos primitivos y las relaciones de objeto internalizadas en el paciente, de las pulsiones, de las
fantasías inconscientes y de ansiedades. Es la puesta en escena de las fantasías. Su teoría de la
transferencia se basa en las etapas tempranas del desarrollo. El cambio psíquico se logra por la
interpretación de la fantasía en la situación de inmediatez de la situación. Generaliza el concepto
de transferencia a todo lo que sucede en el espacio terapéutico.
Bowlby pensaba que la transferencia era la manifestación directa, en las situaciones
interpersonales, de los modelos operativos internos del individuo. El objetivo al interpretarla es
examinar y si es posible modificarla. Se repite el modelo de cómo se relacionaron con uno, cómo
lo trataron las figuras significativas. No toda comunicación dentro de la sesión analítica es

60
transferencia, hay percepciones adecuadas. Solo ciertas comunicaciones pueden ser indicativas
de fenómenos transferenciales.
Kohut.
El paradigma central de la Psicología del Self es que la respuesta empática de la figura primaria
es esencial para el funcionamiento óptimo del self.
Los procesos transferenciales son el resultado de revivir, en una situación actual, la búsqueda de
objeto-self, y el miedo a que esta necesidad se frustre, como ocurrió en la infancia.
El paciente busca una nueva experiencia que le permita continuar y completar un desarrollo
detenido. Para este autor las necesidades se captan por empatía
2- Origen del concepto de Alianza Terapéutica (Zetzel, Greenson, Luborsky)
Elizabeth Zetzel (1956): la Alianza Terapéutica (therapeutic alliance), es la parte no neurótica de
la relación entre terapeuta y paciente que posibilita el insight y el uso de las interpretaciones del
terapeuta para distinguir entre las experiencias relacionales del pasado y la relación real con el
terapeuta.
Es entendida como "una relación positiva y estable entre el analista y el paciente, relación que les
permite llevar a cabo de manera productiva el trabajo del análisis‖ (Zetzel & Meissner, 1973).
Greenson (1965) ha resaltado la idea de que la Alianza de Trabajo (working alliance) es esencial
para la terapia psicoanalítica y la define como ―el rapport razonable, racional y no neurótico que
el paciente tiene con el analista y que le permite trabajar intencionalmente en la situación analítica
a pesar de sus impulsos transferenciales.‖ Sostiene que para que la alianza se produzca es
necesaria "un área libre de conflictos‖.
De manera similar, ha definido la alianza terapéutica como una relación compuesta por la
"relación real‖ y la "alianza de trabajo‖. La primera refiere al vínculo entre el paciente y el
terapeuta mientras que la segunda alude a la capacidad de ambos para trabajar conjuntamente
según los objetivos previstos (Hartley,1985).
Luborsky (1976) destaca dos tipos de AT según la fase o etapa de la terapia. Alianza tipo I …
sensación de apoyo y de ayuda inicio del tratamiento)
Alianza tipo II...Sensación de trabajo conjunto hacia la superación de los impedimentos que
producen malestares.
- Factores de cambio en Psicoterapia: La Alianza Terapéutica como factor común.
Investigadores de distintos marcos teóricos de la psicoterapia, postulan que la alianza terapéutica
es un elemento esencial del proceso terapéutico, ya que han encontrado consistentemente que la
alianza se relaciona con el resultado del tratamiento (Horvath & Symonds, 1991; Høglend, 2014).
La calidad de la alianza es más importante que el tipo de tratamiento en la predicción de los
resultados terapéuticos positivos (por ej., Safran & Muran, 1995).
La Alianza terapéutica es identificada como factor común en las psicoterapias.
Diferentes investigaciones se ocupan del modo en que las características del vínculo terapéutico
propician la adherencia a los tratamientos (Jiménez, 2005).

61
Safran (2003) se interesó por la ruptura de la alianza terapéutica, considerándola como períodos
de tensión o quiebre en la comunicación y la colaboración entre paciente y terapeuta.
Retoma el concepto de —negociación ―, desarrollado por Bordin, para explicar la necesidad de
que terapeuta y paciente se beneficien de la colaboración constructiva diseñando metas y tareas
específicas Una alianza deteriorada o pobre puede llevar a la terminación prematura del
tratamiento.
En relación a esto, Hilsenroth y ot. (1998) sugieren que la capacidad o necesidad del paciente de
establecer vínculos afectivos es un predictor en la continuación de la terapia, mientras que el
egocentrismo y la indiferencia predicen el abandono.
4- Alianza terapéutica según Bordín (componentes)
Bordin (1979), la delimita como un constructo multidimensional. La define como el encaje y la
colaboración entre cliente y terapeuta.
3 componentes:
1 Acuerdo de tareas. 2 vinculo positivo
3 acuerdo de objetivos
5- Alianza terapéutica según Luborsky. Tipos.
(1976) destaca dos tipos de AT según la fase o etapa de la terapia.
Alianza tipo I … sensación de apoyo y de ayuda inicio del tratamiento)
tipo II...Sensación de trabajo conjunto hacia la superación de los impedimentos que producen
malestares.
6- Fases de la Alianza terapéutica para Gunderson
Gunderson (2002) en los Procesos Terapéuticos en el Trastorno Límite de la Personalidad
desarrolla tres formas de Alianza Terapéuticas que se dan de forma consecutiva durante la
terapia.
Construcción de la alianza terapéutica. tipo de alianzas consecutivas:
CONTRACTUAL. Fase 1 del tratamiento. Acuerdo de objetivos terapéuticos entre paciente y
psicoterapeuta.
RELACIONAL. Fase 2. Empática. Terapéutica como accesible. Lo percibe amable, comprensible
y auténtico.
DE TRABAJO. Fase 3 y 4. (Cognitivo- emocional). Considera al terapeuta como colaborador en
quien se puede confiar para entenderse a sí mismo.
- Etchevers, M., Helmich, N., Giusti, S. D. y Putrino, N. I. (2016). Alianza terapéutica, sus estudios
actuales, y desarrollos en el contexto local. VIII Congreso Internacional de Investigación y
Práctica Profesional en Psicología, 8, 35-38. Recuperado de https://www.aacademica.org/000-
044/183.pdf
La Alianza Terapeutica adquiere importancia actualmente porque independientemente del marco
teorico o se trate de un psicologo o un psiquiatra ésta tiene relación directa con los resultados del
tratamiento. Buena Alianza—>Rdo Positivo

62
Esto motivó a generar mas estudios para evaluar que factores ayudan a fortalecer la AT.
Ademas, se observo como constante en todos los tipos de tratamientos que el factor comun de la
Alianza terapeutica era el que podia explicar en general los buenos resultados terapeuticos de
todas ellas.
ANTECEDENTES DE LA AT
Transferencia de F. evolucionó en Alianza Terapeutica o Relacion Terapeutica. La AT es valorada por ≠´s
marcos teoricos. El concepto en si de ‘Alianza Terapéutica’ surgió en la Psicologia del Yo, corriente
psicoanalítica de US.
Greenson define a AT como “la relación racional y relativamente no neurotica que tiene el
paciente con su analista”
Hartley dice que la AT es una relación compuesta por la “Relación Real” y por la “Alianza de
Trabajo”, hablando del vinculo entre paciente y terapeuta mas la capacidad de ambos para
trabajar conjuntamente segun los objetivos acordados.
Para el campo de la psicoterapia no psicoanalitica…
-Luborsky plantea dos tipo de AT según la fase del tratamiento:
Alianza de tipo I: que tiene lugar al inicio del tratamiento cuando el paciente siente que cuenta con
el apoyo del terapeuta
Alianza de tipo II: Que se da mas avanzado el tratamiento y consiste en la sensación de trabajo
en conjunto con el terapeuta
-Bordin, uno de los autores mas importantes en relación al tema de la AT definió la AT como un
CONSTRUCTO MULTIDIMENSIONAL, con 3 componentes:
1-Acuerdo en las Tareas
2-Vinculo Positivo
3-Acuerdo en los objetivos del tratamiento
-Beck, autor de la terapia cognitiva hace una distincion en lo que es la AT para el enfoque
cognitivo vs lo que es para el psicodinamico:
El enfoque cognitivo hace mayor énfasis en el trabajo colaborativo entre paciente y terapeuta
tanto para plantear objetivos terapeuticos como para trabajar para alcanzarlos. Una 2da diferencia
es que el enfoque cognitivo toma a la AT como una variable NO especifica, no es ni técnica ni
instrumental sino auxiliar a otras técnicas que son las que producen el cambio, es como una
variable facilitadora pero no ejerce cambios por si sola en la terapia. Por ejemplo cuando la
presencia del terapeuta disminuye la ansiedad y facilita la aplicacion de otras técnicas
comportamentales especificas.
DESARROLLO EMPIRICO DE LA AT
Escalas para evaluar la AT:
-Working Alliance Inventory, toma los tres elementos que integran el constructo multidimensional
segun Bodin.

63
-Helping Alliance Questionaire
-California Psychotherapy Alliance Scale
Son las mas usadas y todas toman en cuenta el caracter de Relacion de colaboracion en
confianza de la AT
Recientemente se han desarrollado tecnicas para PROMOVER la AT, apuntan a fomentar la
relacion colaborativa, el acuerdo en los objetivos y tareas y rol activo del paciente en su propio
tratamiento, detectando problemas y monitoreando los objetivos permanentemente.
Otra investigación arrojó como Rdo que la calidad de la AT está directamente relacionada con el
resultado de la Psicoterapia (independientemente del tipo de tratamiento).
La investigacion de Corbella & Botella tambien concluye que Gran parte del éxito terapéutico
depende de la relacion interpersonal entre el paciente y el terapeuta (uno de los componentes
esenciales de esa relacion es la AT).
ESTUDIOS DE LA AT EN EL CONTEXTO LOCAL
En Arg los marcos teoricos en psicoterapia predominantes son: el Psicoanalisis y la Teoría
cognitivo-Conductual.
-Se estudió como se lleva a cabo el proceso inferencial clínico de los psicoanalistas y de los terapeutas
cognitivos con ≠´s niveles de experiencia y ≠´s tipos de intervenciones.
-Se estudiaron tambien la interaccion entre distintos tipos de intervenciones de ≠´s marcos teoricos y
factores comunes a las psicoterapias
-Se estudio la relacion entre: Grado de AT- Percepcion de cambio/mejoria- Frecuencia de
sesiones y estilo de intervencion analitica en pacientes y analistas. El autor de la investigacion
(Rubén Zukerfeld) planteó que el grado de alianza percibido esta relacionado con la mejoria y no
esta vinculado con la frecuencia de sesiones ni la modalidad de intervenciones
-Localmente se usa la Escala de Evaluacion de la Alianza Terapeutica (HQR) y la versión
adaptada de Corbella & Botella de la Working Alliance Theory of Change Inventory para evaluar la
AT pero estas no permiten evaluar la variabilidad de la AT entre las sesiones, por eso estos
autores realizaron la validacion de la Working Alliance Inventory en su version observador en el
ambito local.
-El equipo de Etchevers esta comenzando a explorar las nociones de los psicoterapeutas de
CABA sobre la AT y su influencia en los tratamientos psicoterapeuticos.
Segun los antecedentes, las carateristicas de los terapeutas influyen en el inicio, mantenimiento y
finalización de la Alianza Terapeutica, siguiendo este razonamiento los profesionales consideran
que la AT es escencial en el inicio, mantenimiento y finalizacion del tratamiento, siendo tambien
un
buen predictor de los resultados del mismo.
El equipo de Etchevers actualmente investiga:
1- si existe relacion entre las caracteristicas personales y profesionales del terapeuta y la AT
2- como las caracteristicas profesionales del terapeuta influyen en el vinculo terapeutico

64
- Marrone, M. y Diamond, N. (2001). Sobre la transferencia: aspectos de fondo. En
M. Marrone (Ed.), La teoría del apego. Un enfoque actual (pp. 187-204). Madrid: Psimática.
INTRODUCCION
Para la teoría psicoanalítica clásica la Transferencia era una forma de desplazamiento en la cual
sentimientos, deseos y expectativas son derivados de una persona a otra. Freud en “Estudios
sobre la histeria” lo llamo falso enlace, ya que esos sentimientos originalmente asociados a los
padres ahora son trasladados a la figura del medico.
El afecto se muda de lugar, la persona cree que eso le pertenece a una figura actual pero en
verdad corresponde a la figura paterna. Quien observó este fenómeno en la clínica primero fue
Breuer en el tratamiento de Anna O.
Embarazo fantaseado por parte de su terapeuta, amor hacia su terapeuta, luego de haberse
dedicado exclusivamente a su padre por una enfermedad. El afecto hacia su padre y sus
conflictos con el fueron revividos en la persona del medico.
Lo esencial de la transferencia es experimentar sentimientos hacia una persona que no
corresponden a esa persona especifica sino a otra. Se reacciona con una persona del presente
como si fuera con una persona del pasado. Esta persona del pasado que origina la transferencia
son las personas mas significativas y destacadas de la infancia.
Lo transferido es cualquier componente de una relación de objeto: afecto, deseo, fantasias,
sentimientos, fuerza, miedos, ideas, actitudes, defensas, etc.
Inicialmente para F. la transferencia era un obstaculo a sortear en la terapia, como una resistencia
que se opone a la emergencia del recuerdo reprimido. Sin embargo en un 2do momento reconoce
la importancia de la transferencia para COMPRENDER Y ANALIZAR al paciente.
Al revivir temas de la infancia se da en terapia una neurosis de transferencia que reemplaza la
neurosis original del paciente, donde le paciente revive sus temas infantiles en la persona del
medico en transferencia.
El S se encuentra siempre en relaciones de transferencia, la diferencia es que en el análisis el S
puede anoticiarse de ello y notar la diferencia de las relaciones pasadas y presentes. El
tratamiento analitico no crea las transferencias del S sino que las saca a la luz.

LO QUE LA TRANSFERENCIA NO PUEDE SER


Pensar que todo lo que sucede en la sesion es manifestacion transferencial es una vision muy
reduccionista de la transferencia porque hace pensar que la transferencia solo se da en situación
terapeutica (que no es real) y excluye la posibilidad de pensar a las comunicaciones del paciente
con validez en si mismas por lo que son y no por lo que pueden llegar a querer expresar sobre el
pasado.
Solo ciertas comunicaciones pueden considerarse fenomenos transferenciales.
Ademas darle toda la importancia al mundo interno del paciente deja el mundo real donde esta
inmerso sin explorar, hay que ponderar todo. Tanto su mundo de representaciones interno como

65
la realidad donde se encuentra inmerso, su red de comunicaciones y relaciones interpersonales
actuales.
Aqui y Ahora—> es donde se “actua”, donde se Representan las relaciones Tempranas. Hay
conexion entre el presente con el pasado y se anticipa el futuro, pero no todo el dispositivo a
anlitico se reduce a ello.
Otro uso equivoco es el de transferencia positiva y negativa respecto del paciente hacia el
analista y los sentimientos/afectos que éste le transfiere, sin ponderar que el analista y sus
características personales y profesionales especificas pueden producir ciertas reacciones en el
paciente influyendo en sus actitudes, no puede hablarse de una relación entre dos personas sin
estar consciente de que ambos influyen sobre ella.
LA VISION KLEINIANA DE LA TRANSFERENCIA
Aporte de M. Klein al concepto de Transferencia:
Resalta la importancia de..
- la Fantasia
- de la destructividad innata que tiene como fuente la pulsion de muerte
- de las ansiedades
- y de las defensas tempranas
Las fantasias son representaciones mentales de las pulsiones, se dan internamente —> Las
fantasias son la fuente de la Transferencia. La transferencia es la puesta en escena de estas
fantasias.
Origen de la transferencia—> Hay relación entre las experiencias interpersonales pasadas y
presentes, esta relación es integrada junto con ansiedades y defensas tempranas. Se revive la
ansiedad persecutoria temprana y se manifiesta como TRANSFERENCIA NEGATIVA,
proyectandose el pasado en el presente.
Es importante analizar la transferencia para descubrir los aspectos reales y los fantaseados y
entender que toda fantasia tiene elementos de la experiencia real, por ende su analisis ayuda a
lograr el cambio psicologico
KOHUT Y LA TRANSFERENCIA
(psicologia del Self)
Aporte central del paradigma de Kohut es que la figura de apego debe tener la capacidad de dar
respuestas empaticas, esto es fundamental para la formacion y formcionamiento optimo del Self.
En el marco de la teoria de Kohut, la transferencia es el resultado de revivir en una RELACION
actual la busqueda de los objetos self y a la misma vez el miedo a que esta necesidad se frustre
como ocurrio en la infancia (fracaso optimo que da lugar a la internalizacion y asi poco a poco se
da lugar a un Self diferenciado del mundo exterior/ ambiente)—> Transferencia propiamente
dicha.
Transferencias Especulares: Son aquellas en las que el otro viene a confirmar mi identidad/Self,
buscar la admiración en el analista

66
Transferencia Idealizadora: Donde se idealiza al terapeuta tal como el paciente hizo con sus
imagos parentales.
Estas son el surgimiento de lo arcaico en el paciente y NO deben ser interpretadas porque
SOLAMENTE SE TRATA DE UNA REACTIVACION DE LA CONDUCTA DE APEGO en el
contexto de la Relacion Analitica
EL CONCEPTO DE LACAN SOBRE LA TRANSFERENCIA
Lacan desde el inicio rechazó la idea de la transferencia como un afecto que se gesta en el
interior del S y que se externaliza. p/ L es una relacion estructural donde hay un INTERCAMBIO
que cambia a los DOS S´s inmersos en dicha relacion, esto quiere decir que la transferencia no
puede entenderse como un fenomeno INDIVIDUAL, no corresponde solo a una de las partes y
tiene que entendersela como resultado de una relacion entre DOS.
Fenomeno clave en la transferencia —> Sujeto Supuesto Saber, es el lugar que para el paciente
ocupa el analista, él cree que el analista conoce el significado oculto de sus palabras, sintomas,
padecer.
En la practica clinica este supuesto puede aparecer de diversas maneras, antes de arrancar el
ratamiento, apenas incia o incluso el S puede pensar que el analista no es inteligete
suficientemente y que no sirve para entenderlo.
*Es una funcion que el analista tiene que representar en el tratamiento, cuando esta funcion se
establece, la transferencia ya esta en funcionamiento.
El analista debe realizar el manejo de estar transferencia rehusar este poder que el paciente le
otorga, no se debe permitir que el paciente haga de él el centro del mundo ni el depositario de
todos los significados, no debe aceptar ser puesto en este lugar de “semidios”. Quedarse ahi seria
ponerse en el lugar dek objeto que puede llenar toda falta y colmar el deseo del Paciente creando
una ilusion y obturando el reconocimiento de la FALTA.
FALTA ESTRUCTURAL Y NECESARIA, sin dicha falta no se motoriza el deseo, no hay espacio
para realizar movimeintos. Esta Incompletur es INHERENTE a la CONDICION HUMANA, no hay
objeto que obture ese vacio.
El analista no debe mostrarse como un Objeto Omnipotente, sino que debe ayudar al paciente a
enfrentarse a ese vacio, a esa falta a esa castracion en él mismo y en el otro. El analista debe
enfrentar al S con lo que este quiere evitar: la Perdida y la Falta.
LA TRASNSFERENCIA EN EL CONTEXTO DE LA TERAPIA ANALITICA DE GRUPO
Los integrantes del grupo pueden tomar como figura de transferencia a otros miembros del grupo
(a quien en gral se los pone en el lugar de hermanos) o al conductor del mismo (quien es
generalmente visto como una figura parental, autoridad normativa y de quien se espera cuidado y
consuelo como recompensas y castigos).
Hay mas opciones para elegir donde depositar ideas, expectativas y afectos, etc. muchas veces
la apariencia fisica o caracts de la personalidad facilitan mas la transferencia en una persona que
en otra.
LA OPINION DE BOWLBY SOBRE LA TRANSFERENCIA

67
Para Bowlby, la transferencia es la manifestacion en la interaccion interpersonal de los Modelos
Operativos internos del S. Al interpretar la transferencia, el objetivo es sacar a la luz y si es
posible (y necesario) modificar estos modelos operativos.
El analista esta siendo asimilado a algun modelo operativo preexistente que el paciente tiene
sobre lo que espera de cualquier cuidador (dado que se establece con el analista una relacion de
cuidados) y espera de él (del analista) lo que esperaria de las figuras significativas de su infancia.
Ejemplo: Mujer que iba a terapia 5 veces a la semana, un dia pidio al analista si el viernes podrían
suspender porque tenia actividad con sus amigos, asi fue como se angustio y rompio a llorar,
luego se entendio que se debia a que su padre, de pequeña la obigaba a tener muchas
actividades extraescolares, la terapia habia debenido en una obligacion “extracurricular” mas que
no le permitia disfrutar ni tener otros espacios ludicos. Se forman modelos operativos de niños
que tienden a mantenerse de adultos.
Niño que se golpea y la madre en vez de ser compasiva y cuidarlo lo golpea diciendole que
“siempre sos tan estupido” este niño forma un modelo operativo de si mismo como incompetente
y de su madre como poco compasiva y punitiva.
TRANSFERENCIA Y PROYECCION
La transferencia es habitualmente entendida como una proyeccion en la fguar dle analista de
objetos y sentimientos internos.
La proyeccion es un proceso donde una persona atribuye a otra un rasgo de su ppio Self,
habitualmente algo con lo que no esta a gusto de si mismo.
“Proyecto en el otro o sobre el otro algo que rechazo sobre mi mismo xq me resulta intolerable”
EL PELIGRO DE INTERPRETAR DEMASIADO LA PROYECCION
Interpretar todo como Proyeccion en analisis es un problema:
-Es reduccionista—> porque la gran diversidad de interacciones que el el S tiene con su mundo
se limitan a un particular mecanismo de defensa: la proyeccion.
-Afecta el autoestima del paciente—> teniendo en cuenta que uno rechaza lo que no tolera de si
mismo y lo traspone a la persona del otro, al interpretar todo como proyeccion, le damos a
entender que lo que rechaza del otro le es propio, le pertenece.
-Falta de confirmacion de las afirmaciones/comunicaciones del paciente, estaríamos
permanentemente poniendo en duda al paciente y sus percepciones, a la larga el paciente podría
desarrollar dudas o desconfianza sobre sus propias percepciones.
SOBRE LA IDENTIFICACION PROYECTIVA
Id Proyectiva—> donde el otro se termina adueñando de eso que el otro le atribuye. Se da
Intrapsiquicamente e interpersonalmente en 3 pasos:
-proyeccion en el otro
-en la interaccion personal se presiona al destinatario a pensar, sentir y actuar en concordancia
con la proyeccion
-El destinatario cede ante la presion y se adueña de lo proyectado actuando, sintiendo y
pensando acorde a la proyeccion.

68
Esta dinamica puede darse entre analista y paciente, el analista puede proyectar su propia
vulnerabilidad en el analizado y este identificarse con ella.
ESTRATEGIAS QUE SE ACTIVAN EN SITUACION DE TRATAMIENTO
Los niños aprenden estrategias para recibir atencion de sus padres o figuras de apego cuando
estan ansiosos, inseguros o necesitan cuidados.
Es importante detectar cuales son las estrategias disfuncionales que los S´s adultos siguen
utilizando para la vida, identificarlas y traerlas a la luz como parte del analisis.
Ejemplo de la paciente que se comportaba como payaso con sus colegas y en analisis, en verdad
esa era su estrategia para recibir atencion de sus padres cuando era niña.
EL PAPEL CENTRAL DEL ANALISTA
para que el analisis sea exitoso el paciente debe estar comprometido y el analista NO debe estar
en una posicion de absoluta centralidad, el papel de importancia del analista es otorgado por el
paciente y nunca debe ser reclamado por el analista.
El analista debe analizar las vicisitudes de la relacion analista-analizado pero no desde una
posicion central absoluta. Como el ejemplo del analista que no aceptaba que el paciente quiera
hacer terapia de pareja ni consultar con sus amigos sobre sus problemas por creer que él debía
ser el centro de la vida psiquica de su paciente y unica fuente de comprension.

- Putrino, N., Etchevers, M. & Mesurado, B. (2017). El vínculo terapéutico: el rol del
entrenamiento en empatía. Acta Psiquiátrica Psicológica de América Latina, 63 (4):276-28

1- Definición de empatía
La empatía es un componente de la cognición social, que implica la capacidad para comprender
al otro y ponerse en su lugar. La empatía es la capacidad para conectar
con la experiencia de otra persona. Ser empáticos es imprescindible para el funcionamiento de
una sociedad, dado que está en juego en varios comportamientos en que las personas conviven
entre sí, como la cooperación, la compasión y la búsqueda del bienestar general.
La empatía hace referencia a la tendencia a experimentar de forma vicaria (aprendizaje
observacional) los estados emocionales de otros.
Se trata de una compleja forma de inferencia psicológica en la que la observación, la memoria, el
conocimiento y el razonamiento se combinan para poder comprender los pensamientos y
sentimientos de los demás.
Se relaciona con el contagio emocional, la teoría de la mente (TdM) y la toma de perspectiva.
La empatía humana, a diferencia de la observada en otros animales, implica Teoría de la Mente y
mentalización, es decir, la capacidad de predecir la conducta de otros por medio de la atribución
de estados mentales independientes.
Davis propone una definición multidimensional de la empatía, considerando que esta capacidad
se compone de aspectos cognitivos y afectivos:

69
a) la empatía cognitiva basada en la capacidad de adoptar la perspectiva del otro, proceso en el
cual las funciones ejecutivas serían relevantes, permitiendo mayor flexibilidad cognitiva y
autorregulación.
b) la empatía emocional o afectiva, que sería la reacción ante el estado emocional de otra
persona, en la cual sería relevante la percepción e imitación de expresiones emocionales.
2- Empatía y relación terapéutica
Empatía terapéutica
Rogers define la empatía terapéutica como la capacidad de percibir con precisión el marco interno
de referencia del otro, incluyendo sus componentes emocionales y significativos.
Watson y Greenberg proponen que la manera en que los psicoterapeutas atienden a la
experiencia y subjetividad de los pacientes ayuda a que estos internalicen sus problemáticas, con
la finalidad de que aprendan a regular y moderar la forma de actuar Steckley mostró que los
pacientes que perciben a sus terapeutas como empáticos experimentan cambios estructurales en
sus modelos internos y tratan a sí mismos y a otros menos negativamente (por ejemplo, son
menos destructivos, controladores, oprimidos) al final de la terapia. Todos estos auto cambios
positivos también se asociaron con resultados del tratamiento más favorables.
Según Angus, Lewin, Bouffard, y Rotondi-Trevisan, Elliott, Watson, Goldman, y
Greenberg, y Westra, las principales características de la empatía terapéutica son que
el paciente se sienta confiado en contar sus historias personales, sienta alivio y mayor regulación
emocional y genere motivación para el cambio.
Rogers considera la actuación del psicólogo a modo de un «como si», es decir, el terapeuta tiene
que sentir el mundo privado del paciente como si fuera suyo, pero sin perder nunca la claridad de
que eso le sucede a su paciente.
En el área de la medicina, Mercer y Reynolds describen la empatía del médico como la capacidad
del clínico para:
a) comprender la situación, la perspectiva del paciente y los sentimientos,
b) comunicar esa comprensión y comprobar su exactitud y
c) actuar en ese entendimiento con el paciente en una forma útil (terapéutica).
Empatía y relación terapéutica
Lo expresado anteriormente se encuentra estrechamente relacionado al concepto de relación
terapéutica (RT). La RT es un constructo multidimensional y transteórico, en el cual se identifican
tres componentes: acuerdo en las tareas, vínculo positivo y acuerdo en los objetivos .
Estudios sobre la eficacia en psicoterapia y los factores asociados a los resultados terapéuticos,
han concluido que la relación terapéutica explica un 30% de la mejoría de los pacientes,
funcionando como un factor común a la mayoría de las psicoterapias.
Si bien la evaluación de la RT es ampliamente estudiada en psicoterapia, por el momento se
conoce poco acerca de las características personales y habilidades de los terapeutas(empatía)
que influyen en una buena RT. La mayoría de los estudios en el área se enfocan en evaluar la
relación entre la RT y los resultados del tratamiento.

70
3- Empatía cognitiva y emocional en profesionales de la salud
El entrenamiento de empatía en psicólogos aún no se encuentra desarrollado en profundidad, ni
se accede a manuales para mejorar esta capacidad. Esto genera que la empatía en psicoterapia
dependa en gran medida de la capacidad a priori que posee el psicólogo, es decir, de las
competencias interpersonales.
A su vez, Shapiro sostiene que, en ausencia de un discurso apropiado en la forma de gestionar
emocionalmente aspectos inquietantes de la condición humana, es probable que los alumnos
recurran a mecanismos de adaptación que resultan en la distancia y el desapego hacia los
pacientes.
La empatía humana, a diferencia de la observada en otros animales, implica mentalización
Una definición multidimensional de la empatía, considera que esta capacidad se compone de
aspectos cognitivos y afectivos (DAVIS):
a) La empatía cognitiva basada en la capacidad de adoptar la perspectiva del otro, proceso en el
cual las funciones ejecutivas serían relevantes, permitiendo mayor flexibilidad cognitiva y
autorregulación.
b) La empatía emocional o afectiva, que sería la reacción ante el estado emocional de otra
persona, en la cual sería relevante la percepción e imitación de expresiones emocionales.
En profesionales:
1) La empatía cognitiva Identifica estados mentales con precisión y se comunica de manera
efectiva y congruente.
2) La empatía emocional Reconocer el estado emocional y adecuar el estilo de comunicación con
ese estado.
4- Sobre Empatización. Efectos posibles
Un riesgo que puede correr el terapeuta es el del distrés emocional o la sobre
empatización. Esta ocurre cuando el terapeuta no logra poner la necesaria distancia con su
paciente y la emoción lo invade de tal manera que olvida que el problema o la situación conflictiva
«le pertenece» al paciente. En la experiencia de sobre empatización, la persona pierde
temporalmente la capacidad de regular la propia implicación emocional y olvida temporalmente
que lo que le sucede al otro, es diferente a su perspectiva.
La activación emocional de una sobre empatización puede ser conceptualizada como el exceso
de identificación con la otra persona. En cambio, la capacidad para regular y controlar su
compromiso emocional es inherente a la concepción de la empatía terapéutica.

PRACTICO SEMANA 5
-Grasso, Jimena E.; Barbieri, Micaela B. & Etchevers, Martín J. TERAPIA DINÁMICA art
INTERPERSONAL (DIT) (2021). Ficha de cátedra.
DIT

71
La Terapia Dinámica Interpersonal retoma pilares del marco psicoanalítico, tales como: el impacto
de experiencias de la vida temprana en el funcionamiento adulto, los factores externos e internos
que modelan la mente y nuestra percepción sobre nosotros mismos en relación con otros, la
presencia de aspectos inconscientes que constituyen fuerzas motivadoras, los procesos
proyectivos e introyectivos que apuntalan la experiencia subjetiva de los vínculos, y la ubicuidad
de la transferencia, desde la cual los pacientes se relacionan con los otros, y, en particular, con el
terapeuta (Lemma et al., 2013) .
Bases de la Terapia Dinámica Interpersonal
La Terapia Dinámica Interpersonal es una psicoterapia con orientación psicodinámica, que
recupera en su desarrollo las nociones de las teorías del apego y de la mentalización (Fonagy et
al., 2020). Le da un rol crucial al ambiente interpersonal del paciente, en miras a comprender su
conducta y su experiencia
Considera los síntomas de depresión y de ansiedad como respuestas a dificultades
interpersonales, o como percepciones de amenazas de pérdida o separación en relación a figuras
de apego, y, por lo tanto, percibidas también por el sujeto como amenazas para él mismo. La DIT
tiene entonces como objetivo ayudar a los pacientes a mejorar su capacidad para hacer frente a
los desafíos interpersonales, relacionados con el apego, a través de una mejor comprensión de
sus reacciones subjetivas ante lo que se percibe como una amenaza. Esto se va logrando en un
proceso que va haciendo explícitas las ansiedades y preocupaciones implícitas del sujeto, a la
vez que va mejorando su capacidad para comprender, identificar y nombrar sus propios
pensamientos y sentimientos, y los de los demás
En el proceso del tratamiento, se trabaja con los pacientes para que puedan comprender la
conexión entre los síntomas que presentan y sus relaciones interpersonales, al identificar un
núcleo, un patrón de relación no consciente y repetitivo. Este patrón (Fonagy et al., 2020) se
convierte en el foco central de la terapia (Foco Afectivo Interpersonal), y el terapeuta trabaja en el
perjuicio consciente que tienen para el sujeto esos patrones repetitivos
Entonces, la DIT se dirige a los supuestos orígenes dinámicos de los síntomas depresivos,
identificando patrones y modalidades de apego, para ir luego a la mejora del funcionamiento
interpersonal, a través del incremento de la capacidad de mentalización del paciente sobre los
propios estados mentales, así como los de los demás.
Esta psicoterapia tiene dos objetivos clave, llevados adelante en tres fases del tratamiento (Rao
et al., 2019):
El primer objetivo es que el paciente comprenda la conexión entre los síntomas que tiene en el
presente con lo que está ocurriendo en sus relaciones interpersonales, al identificar un núcleo
inconsciente y repetitivo.
El segundo objetivo implica mejorar la capacidad del paciente para reflexionar sobre sus estados
mentales, y así aumentar su capacidad de manejar las dificultades interpersonales que encuentra
en su vida diaria.
Fase inicial (sesiones 1 a 4) - Fase de compromiso / evaluación
En la primera fase de tratamiento se busca explorar los síntomas presentes, así como identificar
las fortalezas y recursos con las que el paciente cuenta, tanto en sí mismo como en sus
relaciones interpersonales. Además, se busca generar un compromiso con el paciente mediante

72
un trabajo colaborativo, que permita asimismo la delimitación de un área en la que se focalizará el
trabajo terapéutico.
La relevancia de esta fase radica en la evaluación del problema que se lleva a cabo en estas
primeras sesiones, que incluye la determinación de las relaciones interpersonales significativas
del paciente, la caracterización del estilo de apego en base a las descripciones que el paciente
profiere sobre sí mismo, la identificación de eventos vitales desencadenantes y la delimitación del
funcionamiento interpersonal pasado y presente
Fase intermedia (sesiones 5 a 12)
se encuentra centrada en el área interpersonal en la que se ha decidido focalizar el trabajo
terapéutico. Se aplica una serie de estrategias en pos de analizar el lugar del paciente al
mantenimiento del problema o los síntomas que atraviesa, cuál es la función que se pone en
juego y el perjuicio que conlleva
Fase de término (sesiones 13 a 16)
en la exploración de conflictos referentes a la pérdida, la separación y la independencia,
generados a partir de la separación del terapeuta prevista. También se realiza un balance de los
logros alcanzados y se delinea un plan a futuro. Algunas de las estrategias utilizadas para dichos
fines son: la revisión del progreso, la expresión de ansiedades y fantasías del paciente acerca del
final de la terapia, la elaboración de una “carta de despedida” y la anticipación de posibles áreas
de vulnerabilidad y obstáculos futuros.

Técnicas
El apoyo y la empatía son indispensables, sin dejar de lado la importancia que representan
también la confrontación y el desafío en las situaciones necesarias. El uso juicioso del silencio es
primordial para que el paciente pueda desarrollar las relaciones que ubica en su sintomatología y
su realidad cotidiana, a modo de hipótesis causales. Sin embargo, el terapeuta DIT es más activo
que en las terapias analíticas de larga duración.
es de utilidad la aplicación de técnicas directivas (no específicas), tales como el incentivo a probar
determinados abordajes ante ciertos conflictos interpersonales. En aquellos casos de más alto
nivel de funcionamiento interpersonal premórbido, es más probable el uso de técnicas expresivas.
Las técnicas expresivas utilizadas en la DIT son la clarificación, la confrontación y la
interpretación. Presenta especial importancia el foco en la identificación y la ayuda brindada al
paciente para la verbalización de sus sentimientos.
Terapia Dinámica Interpersonal en Cuidados Complejos
es una modificación del modelo de la DIT, que surge ante la necesidad que existía en la clínica
de adaptar el modelo para casos de depresión recurrente, con características de cronicidad,
comorbilidad y severidad. Se dirige a individuos con quienes anticipamos una resistencia
significativa a nivel de la implementación de cambios interpersonales, a pesar de la comprensión
que ellos ya tienen sobre sus patrones relacionales recurrentes, y cuya fijeza les da cierta
persistencia a unos patrones de respuesta desadaptativos, a los cuales denomina
“comportamientos que interfieren en la relación”

73
Toma los mismos supuestos que el modelo original, y sostiene que las relaciones interpersonales
de un individuo pueden restringir su capacidad para funcionar adaptativa y adecuadamente,
llevando a la persona a sentirse incapaz de modificar sus situaciones personales, sintiéndose
atrapada y desesperanzada. En casos de esta índole, los pacientes suelen responder de forma
tipificada y poco flexible ante cambios y eventos nuevos en su entorno
Mientras que DIT indica que llevar la conciencia del paciente a las cualidades desadaptativas de
tales patrones relacionales arraigados es suficiente para modificar sus expectativas, la DITCC
agrega que, además, la actitud general del individuo hacia el aprendizaje de sus experiencias
también debe cambiar.

Es por eso que la DITCC tiene dos objetivos en su enfoque terapéutico, siendo las RIBs el foco
de intervención de la terapia (Fonagy et al., 2019). El primero de los objetivos coincide con los
principios de la DIT ya mencionados, relacionados principalmente con identificar un núcleo de
patrones relacionales, y abordar sus características desadaptativas. El segundo objetivo busca
modificar la desconfianza epistémica que siente el individuo en relación con las comunicaciones
sociales que recibe en su experiencia diaria, y que contribuye aún más a fijar los RIBs.
Vínculo terapéutico y técnicas de mentalización en Cuidados Complejos
En la DITCC, se aborda este desafío centrándose específicamente en mejorar la confianza que
un individuo tiene en el terapeuta, al enfocar sistemáticamente la atención (sobre todo en las
primeras fases del tratamiento), en crear una experiencia de agencia en el paciente, al permitirle
percibir con claridad la relación entre sus estados internos y su comportamiento
implica que el terapeuta pueda prestar su mentalización como técnica de abordaje, incluirla en el
vínculo terapéutico, y trabajar desde allí la regulación afectiva. La diferencia entre la DIT y la
DITCC en este punto, radica en la atención particular que se pone en esta última (la DITCC), en
el vínculo terapéutico, para que el paciente pueda mentalizar

-Lemma, A ; Target, M y Fonagy, P. Terapia Dinámica Interpersonal Breve. Guía Clínica


(2018).Chile: Editorial Mediterráneo. Capítulo 3: características y estrategias centrales

Características y estrategias centrales


La TDI se diferencia a nivel de su estrategia global porque es una terapia de tiempo limitado qué
consiste en 16 sesiones semanales estructuradas en torno a la formulación y elaboración de un
patrón afectiva interpersonal recurrente y problemático que se convierte en el foco de trabajo es
decir el FAI. Se trata de un proceso de tres fases cada una con objetivos y estrategias
específicas.
Objetivos
La TDI ha sido desarrollada para satisfacer las necesidades de pacientes que están deprimidos
y/o ansiosos. Se orienta fundamentalmente hacia el funcionamiento interpersonal del paciente y
su capacidad de pensar y comprender cambios en su estado de ánimo desencadenados por la
activación de una representación self - otro específica en su mente, lo que constituye el principal

74
medio para reducir los síntomas de depresión y ansiedad. Por lo tanto, la TDI tiene dos objetivos
primordiales:
● Ayudar al paciente a entender la conexión entre sus síntomas y lo que está sucediendo en
sus relaciones, identificando un patrón relacional nuclear, inconciente y repetitivo que se
transforma en el foco de la terapia.
● Promover la capacidad del paciente para reflexionar sobre sus estados mentales y mejorar
así su habilidad para manejar sus dificultades interpersonales.
Tabla 3.1 características centrales de la TDI
● Trabaja en un foco afectivo interpersonal (FAI) circunscrito.
● Se focaliza en la mente del Paciente más que en su comportamiento
● Un tiempo limitado 16 sesiones
● Aborda y utiliza la relación terapéutica para ayudar al paciente a explorar el FAI
● Utiliza técnicas expresivas, de apoyo, mentalización y cuándo es adecuado técnicas
directivas para maximizar el cambio en el marco del formato de una terapia breve.
● Apunta principalmente a mejorar el funcionamiento interpersonal y la capacidad de
comprenderse a sí mismo y al otro más que a un cambio a nivel de carácter.
Trayectoria de la terapia
La TDI como un proceso de 3 fases: una fase inicial qué se dedica principalmente a involucrar al
paciente en la terapia y abordar sus dificultades, una fase intermedia durante la cual la terapeuta
se centra en el problema o patrón identificado, y una fase final en que se revisa el progreso
realizado y se ayuda al paciente a despedirse, enfocándose en el significado consciente e
inconsciente que tiene para el separarse de su terapeuta.
Fase inicial (sesiones 1 a 4)
Se dedican a la evaluación del problema y su formulación dinámica, lo que nosotros llamamos
FAI- y a motivar al paciente mediante un trabajo explícitamente colaborativo, involucrándole en el
proceso de afinar la formulación. La fase inicial es crucial para el desarrollo posterior de la TDI. La
fase inicial es la que resulta más difícil para los y las terapeutas que se están iniciando en esta
práctica de TDI, pero al mismo tiempo es la más útil ya que formular explícitamente un foco ofrece
luego una brújula muy valiosa para las intervenciones terapéuticas durante las siguientes dos
fases.
Tabla 3.2 Fase inicial. Sesiones 1 a 4
Objetivos.
● Motivar y comprometer la participación del paciente
● Explorar síntomas depresivos y ansiosos incluyendo factores de riesgo y haciendo énfasis
en los orígenes y el significado psicológico de los síntomas.
● Identificar fortalezas y o recursos del paciente y su red interpersonal más amplia
● Formular el área focal del trabajo
Estrategias
● Identificar el mapa interpersonal del paciente, lo que incluye un cuadro detallado de sus
relaciones significativas y su conexión con los problemas que motivan la consulta.
● Usar auto descripciones sobre el estilo básico de apego

75
● Focalizar en circunstancias interpersonales y eventos vitales significativos que preceden al
inicio de la depresión y ansiedad y moderadores del estado del ánimo y/o ansiedad.
● Discutir y acordar con el paciente la formulación, la fundamentación del tratamiento y las
metas u objetivos.

La fase intermedia (Sesiones 5 a 12)


El principal objetivo es intervenir usando diversas técnicas para facilitar la elaboración del FAI
acordado.La estrategia terapéutica fundamental es que la terapeuta mantenga firmemente su
atención en el FAI y en el proceso de analizarlo y comprenderlo, ayudando al paciente a pensar
en cómo contribuye ese patrón relacional a mantener el problema y/o que está experimentando,
la función que esté cumple en su mente (incluyendo de que lo defiende), y su costo emocional.
Tabla 3.3 fase intermedia. Sesiones 5 a 12.
Objetivo: trabajar en el área focal interpersonal acordada.
Estrategias:
- ayudar al paciente a mantener el foco de la exploración de narrativa sobre sus relaciones reales
o imaginarias
- mantener el foco en el estado mental del paciente y no en su comportamiento (vincular procesos
interpersonales con Estados mentales del)
- a descubrir lo que está sintiendo en este momento y como esto se relaciona con experiencias
interpersonales presentes y pasadas incluyendo la relación con el terapeuta
- ayudar al paciente a establecer conexiones entre los síntomas y los eventos interpersonales
- mantener una actitud activa y apoyadora para alentar al paciente a probar nuevas maneras de
resolver sus dificultades.

Fase de termino (sesiones 13 a 16)


Dado que la terapia dinámica interpersonal enfatiza la importancia de los apegos, el significado de
la pérdida de la relación con el terapeuta se revela para explorarlo en las sesiones finales. La
tarea de la terapeuta es ayudar al paciente a darle un sentido a su experiencia de término.
Frecuencia en FAI qué se ha trabajado es relevante para comprender la forma idiosincrática que
tiene el paciente de vivir la separación.
La terapeuta también usa las últimas sesiones como oportunidad para llevar al paciente a revisar
el trabajo en torno al FAi (lo que incluye escribirle una carta de despedida con un resumen de
dicho foco de trabajo) y consolidar los logros alcanzados, además de anticipar futuras
vulnerabilidades. Los terapeutas estructura en sus intervenciones guiándose en cinco pasos
estratégicos relativamente simples en el transcurso de un trabajo terapéutico breve:
● Identificar un problema vinculado al apego con un foco relacional específico que el
paciente considera que actualmente lo está haciendo sentir deprimir y/o ansioso.
● Trabajar en forma colaborativa con el paciente para crear una imagen mental cada vez
más clara de las temáticas interpersonales que el problema plantea.

76
● Animar al paciente a explorar la posibilidad de formas alternativas de sentirse o de pensar
usando activamente la relación transferencias para poner el primer plano en las formas de
relacionarse características del paciente.
● Asegurar que se reflexione acerca del proceso terapéutico (sobre el cambio en el self)
● Hacia el final del tratamiento, presentar al paciente un resumen escrito con la visión creada
en forma colaborativa sobre la persona y el área de conflicto icc elegida, para que el
paciente conserve dicha visión, reduciendo así el riesgo de recaída.
Tabla 3. Fase de término. Sesiones 13 a 16
Objetivos
● Permitir al paciente explotar conflictos relacionados con la pérdida, la separación y la
independencia desencadenados por la anticipación de la separación de la terapeuta.
● Evaluar lo que ha logrado y planificar para el futuro.
Estrategias
● Facilitar la expresión de las ansiedades y fantasías del paciente acerca del término
● Revisar el trabajo realizado
● Reconocer los progresos alcanzados
● Anticipar futuras dificultades y áreas de vulnerabilidad
● Escribir una "carta de despedida" que resume el trabajo
Los focos en terapia dinámica interpersonal
El foco afectivo interpersonal
La TDI adopta un enfoque teórico hacía la formulación psicodinámica. La tarea primordial de la
fase inicial es identificar un patrón interpersonal inconciente, dominante y recurrente: el FAI. Este
patrón se anima en una aproximación inconciente al apego, dependencia y posibilidad de
intimidad, aproximación que es capturada en una descripción del estilo de apego dominante en el
paciente. En conjunto con el, se elige un patrón específico que será el foco de trabajo, este se
sustenta en una representación específica del self en relación con un otro que caracteriza el estilo
interpersonal del paciente y lleva a dificultades en sus relaciones porque organiza el
comportamiento interpersonal. Esas representaciones x lo general están ligadas a uno o más
efectos específicos. Se considera que los afectos son respuestas a la activacuon de una
determinada representación self-otro en la mente del paciente.
Aunque las experiencias pasadas influyen en el funcionamiento actual en las relaciones objetales
internas, ellas no son el principal foco de la TDI. Son parte de las formulaciones para encuadrar
sus dificultades en el contexto de su experiencia vivida. El FAI guía las intervenciones de la
terapeuta durante la fase intermedia y ofrece al foco de para ayudar al paciente a empezar a
hacer algunos cambios. Con frecuencia los cambios en el funcionamiento y. Consiguiente alivio
de los síntomas pueden ocurrir a través de pequeños cambios en una área interpersonal
circunscrita. Mientras se decide cuál será el foco, es importante tener en mente que la TDI es una
terapia breve que no busca facilitar un cambio a nivel de carácter. Mar bien, el FAI sed elige en
términos de su relevancia más inmediata en la aparición de la depresión y/o ansiedad del
paciente.
El foco en el aquí y ahora

77
Es fundamental el foco en el aquí y ahora y denota tres actividades relacionadas: el foco en el
afecto del paciente, en las dificultades actuales del paciente y en su relación presente con el
terapeuta.

Foco en el afecto
Durante la terapia se debe prestar mucha atención al estado afectivo del paciente durante la
sesión, lo que incluye también su patrón de regulación afectiva. Las estructuras cognitivas y
afectivas del self y otras representaciones regulan las interacciones interpersonales del paciente,
especialmente con las figuras de apegos actuales. Ayudar al paciente a pensar en lo que siente,
en determinados desencadenantes de sentimientos y como los maneja es tarea central de la TDI.
Se relaciona con el uso activo que hace el paciente de la expresión emocional de la terapeuta
para formar su apreciación de un evento y usarla como guía del comportamiento. La tarea del
terapeuta es de trasformara experiencia del paciente en algo emocionalmente dirigible, así, a
medida que el terapeuta responde, ofrece al paciente una experiencia de ser compré dudo que
me permite desarrollar gradualmente la sensación de que su propia conducta tiene significado y
comunica algo.
La capacidad de reflexionar sobre lo que estamos sintiendo refuerza nuestra capacidad de regular
el afecto. El patrón de activación emocional de cada paciente es diferente. Nuestra comprensión
sobre esto descansa en un seguimiento cuidadoso del estado emocional del paciente durante la
sesión. Este seguimiento involucra varias intervenciones racionadas entre si:
• Ayudar al paciente a reconocer sus sentimientos como propios
• Ayudar al paciente a diferenciar sentimientos de acciones
• Facilitar conversaciones sobre la conexión entre sentimientos y acciones, lo que a su vez
facilita la comprensión de si mismo y toma consciencia de las motivaciones que s atribuye a los
demás.

Foco en la exploración de las dificultades actuales


La TDI se focaliza en la exploración de las dificultades actuales en la vida del paciente y no en
tratar de establecer relaciones con los orígenes infantiles de fichas dificultades. Se hace énfasis
en Inter envío es que ayuden al paciente a sentir que está vivas y presentes en sus relaciones
actúes, incluyendo la relación con la terapeuta y sobre las cuáles él puede realizar un cierto grado
de cambio.

Foco en la relación terapéutica


La TDI usa activamente la relación terapeuta – paciente para ayudar a este a explorar el FAI en la
inmediatez de esta relación. El balance relativo que pone la TDI entre interpretación de la
transferencia y otro tipo de intervenciones es dictado por el paciente momento a momento. Lo que
guía la interpretación de la transferencia es en que medida la exploración del FAI representanta
un atractivo emocional más alto, en oposición con un foco en lo que está ocurriendo en las
relaciones fuera de la terapia.

78
Foco en la mente del paciente
La TDI se enfoca consistentemente en sus estados mentales conscientes e inconscientes y lo que
el imagina o cree con convicción que está sucediendo en la mente de los demás. La capacidad de
mentalizar sustenta en cada uno de nosotros el esfuerzo cotidiano que significa tener una mente.
En algunos pacientes está capacidad está debilitada t este déficit sería clave en su
psicopatología. Aun cuando esté déficit no constituye un rasgo central en los pacientes ansiosos y
deprimidos, si se evidencian fallas en la mentalización que contribuyen a sus dificultades en las
relaciones.
Un objetivo en la TDI es estimular la propia capacidad del paciente de reflexionar su experiencia.
La meta no es solo trabaje un conflicto icc sinó también usar el FAI para estimular la capacidad
del paciente de pensar y sentir su experiencia. Trabajar en la elaboración del FAI implica
entonces cuestionar los patrones automáticos con respecto a la manera de relacionarse en las
situaciones de apego y que el paciente tomé más conciencia de la manera en que sus estados
mentales conducen su comportamiento y estado afectivo.

La actitud terapéutica
Hay un consenso sobre que el terapeuta sea lo menos intrusivo posible y mantenga hacía el
paciente una actitud más neutral que prioriza la reflexión e interpretación por sobre la acción. Las
relaciones del paciente frente a la terapeuta se vuelven el foco de exploración y ofrecen
oportunidad de entender la transferencia y a través de eso, el mundo relacional interno del
paciente.
En la TDI nos pare e que una actitud distante de la terapeuta resulta poco útil. Es importante
buscar la neutralidad y un relativo anonimato, pero eso no debiera conducir al distanciamiento
emocional. Más que ser fría, la terapeuta debería estar activamente involucrada y
emocionalmente sintonizada con la experiencia subjetiva del paciente: ella también es una
participante del proceso terapéutico y tendrá fuertes sentimientos en respuesta a lo que el
paciente le comunica.
La actitud analítica se trata a lo sumo de una forma especial de escuchar: en vez de tratar de
solucionar problemas o dar consejos, el terapeuta empatiza con la experiencia subjetiva del
paciente y al mismo Tiempo tiene curiosidad acerca de su significado icc. También tiene que ser
capaz de tomar perspectiva con respecto a una interacción con el paciente para reflexionar y
comentar sobre ella, para ayudarle a entender cómo se relaciona con los demás y modelar con
ello una actitud reflexiva. En otras palabras, en TDI se adhiere a los principios centrales de una
actitud analítica pero cuando resulta apropiado se hace un esfuerzo por contextualizar las
intervenciones de la terapeuta. La terapeuta se esfuerza por adoptar una posición de no saber y
curiosidad que priorizan la exploración conjunto de los estados mentales del paciente y cómo se
relacionan con el proceso interpersonal identificando que han acordado como foco de terapia.
La retroalimentación consciente e inconsciente del paciente sobre la terapia es importante. Es
todo parece de distintas modalidades: concreta y directamente por medio del formulario de
seguimiento de resultados, forma indirecta a través de las narrativas que siempre traes estrés en
vías de comunicación inconsciente acerca de la terapia.

79
Por lo tanto, aunque la actitud básica en terapia dinámica interpersonal es analítica y se funda en
un interés por las comunicaciones conscientes e inconscientes del paciente y en el uso de la
transferencia la brevedad en tratamiento requiere que la terapeuta sea más activa.

TEÓRICO SEMANA 6
- American Psychiatric Association (2014). Trastornos de ansiedad (Subtipos: Trastorno de
ansiedad generalizada y Ataque de Pánico). En Manual diagnóstico y estadístico de los trastornos
mentales (5ª ed., pp. 189–234). Arlington, VA: American Psychiatric Publishing.
CRITERIOS DIAGNOSTICOS
A. Ataques de panico imprevistos recurrentes. Un ataque de panico es la aparicion subita (desde
el estado de calma o desde un estado de ansiedad) de miedo intenso o de malestar intenso que
alcanza su maxima expresion en minutos, en ese tiempo se producen 4 o mas de estos sintomas:
-Palpitaciones- aceleracion de la frecuencia cardiaca
-sudoracion
-temblor o sacudidas
-sensacion de dificultad para respirar o asfixia
-dolor o molestia en el torax
-nauseas o malestar abdominal
-sensacion de mareo, inestabilidad, aturdimiento, desmayo
-escalofrios o sensacion de calor
-parestesias (hormigueo o entumecimiento)
-desrealizacion (sensacion de irrealidad) o despersonalizacion (separarse de uno mismo)
-miedo a perder el control o volverse loco (Sintoma Cognitivo)
-miedo a morir(Sintoma Cognitivo)
B- Un mes o mas despues de alguno de los ataques sucedio una o las dos situciones siguientes:
-Inquietud o preocupación continua sobre otro ataque y sus consecuencias, x ej miedo a perder el
control, volverse loco, tener un ataque al corazón
-cambio significativo de la conducta, mala adaptación relacionada al ataque de panico. x ej: dejar
de hacer ejercicio fisico, evitar cierto lugar comportamientos con el objetivo de evitar el ataque de
panico
C-La alteracion del ataque de panico no puede ser atribuible al consumo de sustancias (drogas,
medicacion) ni a otra afeccion medica (hipotiroidismo, tratornos cardiopulmonares)
D- La alteracion del ataque de panico no se explica por otra alteracion, x ejemplo como efecto de
ansiedad, fobia, etc
CARACTERISTICAS DIAGNOSTICAS

80
El TRASTORNO de pánico, se refiere a los ataques de pánico inesperados y recurrentes. Miedo y
Malestar intenso en poco tiempo durante el cual ocurren 4 o mas de los 13 sintomas fisicos o
cognitivos mencionados.
Recurrente= significa mas de una crisis de panico inesperada
Inesperado= significa que no hubieron señales previas de que esto ocurriria o un desencadenante
puntual. El ataque parece ocurrir de la nada. Durante el sueño (ataque de panico nocturno) o
cuando el S esta relajado.
Hay ataques de panico esperados, donde hay señales previas al ataque, en las personas con
trastorno de panico pueden aparecer ataques esperados e inesperados. Que los ataques sean
esperados no es condicion para que no haya trastorno de panico.
Si el ataque es esperado o inesperado depende del criterio del clínico que hará un interrogatorio
pertinente y de la percepción del individuo quien dirá si había o no señales previas.
La frecuencia y gravedad de los ataques son variables, el numero y tipo de sintomas pueden
variar de un ataque al otro (para diagnosticar trastorno debe haber presencia de al menos 4
sintimas en mas de un ataque de panico inesperado).
Preocupacion caracteristica de la gente que sufre estos ataques de las implicancias o
consecuencias que los ataques pueden tener en su vida
-Preocupaciones medicas, de padecimeinto d eenfermedad coronaria, a pesar de los rdos
medicos satisfactorios
-Preocupaciones sociales, verguenza o ser juzgados negativamente porque ellos creen que estas
crisis de panico indican que se esta “volviendo loco”, que pierde el control o que padece de cierta
debilidad emocional.
Preocupación como evitación. Pueden haber cambios desadaptativos de la conducta para evitar
el advenimiento de otra crisis o minimizarla.
CARACTERISTICAS ASOCIADAS QUE APOYAN EL DIAGNOSTICO
-Un tipo de ataque de panico inesperado—> el nocturno, despertar del estado de sueño con
panico, es diferente a que el ataque se genere una vez que nos despertamos!
Este tipo de ataque se produce al menos 1 vez en 1/3 o 1/4 de la poblacion con trastornos de
panico
-Ansiedad relacionada a problemas de salud fisica y mental mas que a la crisis de panico en si
mismo, por ejemplo temor a efectos seundarios de una medicacion
-Miedo a no controlar futuros ataques de panico, conductas desadaptadas para evitar las crisis de
panico, por ejemplo no hacer ejercicio, no tomar transporte publico, no ingerir ciertos
medicamentos, abuso de sustancias (alcohol y drogas)
PREVALENCIA
Mas afectadas las mujeres que los hombres, proporcion 2:1
Baja prevalencia en menosres de 14 años aunque puede aparecer en niños.

81
Aumentan los casos de Trastorno de Panico en la edad de la adolescencia y edad adulta,
disminuye en la vejez.
DESARROLLO Y CURSO
Media de edad de aparicion 20/24 años, despues de los 45 años es poco comun, al igual que en
la niñez (puede deberse a la dificultad de estos para explicar los sintomas).
El curso normal, sin tratamiento, es cronico con oscilaciones. Algunos individuos pueden tener
brotes episodicos con años de remision, otros pueden presentar una sintomatologia mas intensa y
continua. Solo unos pocos alcanzan una remision completa sin recaidas.
Este Trastorno en adolescentes y adultos suele ser comorbido con otros tipos de trastornos
(ansiedad, depresion, trastorno bipolar).
Menos casos de brotes en adolescentes que en adultos jovenes, aunque tambien puede deberse
a que los adolescentes estan menos dispuestos que los adultos a hablar de sus sintomas en el
ataque de panico.
FACTORES DE RIESGO Y PRONOSTICO
TEMPERAMENTALES: Afectividad negativa y la sensibilidad a la ansiedad son malas porque
predisponen al ataque de panico
Una historia de crisis de miedo (es que se dan algunos de los sintomas del ataque de panico, sin
ser muchos) puede predisponer al ataque y al trastorno de panico posterior.
Angustia por separacion infantil puede peor no constituye siempre un factor de riesgo.
AMBIENTALES: Historia de maltratos fisico o abusos sexuales infantiles es mas comun es
trastornos d epanico que en otros trastornos de ansiedad.
Fumar es un factor de riesgo
Factores de estres (enfermedad o muerte de familiares, historia con consumo de sustancias,
factores de estres interpersonales) pueden ser riesgo de ataques y trastorno de panico.
GENETICOS Y FISIOLOGICOS:
Se cree que hay una disposición genética para los ataques y trastorno de panico, sin embargo se
desconoce de que genes se trata. Hay aumento de riesgo en hijos de personas con ansiedad,
depresion y trastornos bipolares.
ASPECTOS DIAGNOSTICOS REL CON LA CULTURA
La cultura puede tener incidencia en distintas maneras de clasficar ataques inesperados y
esperados.
Las preocupaciones asociadas al ataque de panico y sus consecuencias varian segun la cultura,
tambien varian segun los grupos de edad y genero.
ASPECTOS DIAGNOSTICOS REL CON EL GENERO
Las caraacts clinicas no difieren segun el genero.
RIESGO DE SUICIDIO

82
Diagonostico de trastorno de panico- 12 meses, mayor tasa de intentos de suicidio o de ideación
suicida
CONSECUENCIAS FUNCIONALES DEL TRASTORNO DE PANICO
Discapacidad social, ocupacional y fisica, altos costos economicos, mayor cantidad de consultas
medicas por angustia lo que genera que la persona se ausente del trabajo o la escuela, lo que
puede conducir al desempleo o abandono escolar.
DIAGNOSTICO DIFERENCIAL
-Si no se dan ataques inesperados entonces puede tener que ser diagnosticado como otro
trastorno de ansiedad especificado o no especificado.
-No se diagnostica T de Panico si los ataquea de panico son consecuencia de otras afecciones
medicas, x ej: hipertiroidismo, trastornos convulsivos, alteraciones cardiopulmonares, etc. Análisis
de laboratorio o el examen fisico sirven para descartar esto
-No se diagnostica T de Panico si el ataque de panico es consecuencia del consumo de
sustancias, si el ataque se vuelve a presentar cuando el S no esta bajo los efectos de ella si es
pertinente el diagnostico. Cuando los ataques se dan despues de los 45 años o con la presencia
de sintomas atipicos como amnesia, trastornos en el habla, perdida del control de esfinteres,
perdida de la conciencia, etc se puede sospechar del consumo y en ese caso habria que
diagnosticar consumo de sustancias
-Hay otros trastornos mentales que tienen ataque de pánico como característica asociada x ej
otros trastornos de ansiedad o trastornos psicóticos.
COMORBILIDAD
El Trastorno de pánico se presenta rara vez aislado de otra patología, es común que se presente
junto con otros trastorno de ansiedad como la agorafobia puntualmente, la depresión mayor, el
trastorno bipolar y posiblemente con el trastorno por el consumo de alcohol.
Si bien el trastorno de pánico en gral tiene una edad de inicio anterior al de las enfermedades
comorbidas, el ataque de pánico suele producirse despues del trastorno comorbido y puede
considerarse como un empeoramiento del mismo.
Un porcentaje de pacientes con Trastorno de Panico termina desarrollando un trastorno por
consumo de sustancias, lo que para alguno significa un intento de parar la ansiedad con alcohol o
medicamentos.
El trastorno de Panico es comorbido con muchos sintomas y afecciones medicas generales,
mareos, perdida de la estabilidad, arritmias cardiacas, hipertiroidismo, asma, EPOC, sindrome de
intestino irritable, entre otros.

2. DSM 5- Trastornos de Ansiedad: TRASTORNO DE ANSIEDAD GENERALIZADO


CRITERIOS DIAGNOSTICOS
A. Ansiedad y preocupación excesivas sobre diversos temas que esta mas presente que días
ausente por un periodo minimo de 6 meses.
B. El individuo no puede controlar esa preocupacion

83
C. Se suman a la ansiedad y preocupacion TRES o mas de los sgtes sintomas que tienen q estar
presentes mas dias que ausentes en los ultimos 6 meses:
-Inquietud o sensación de estar con los nervios de punta
-Facilidad para Fatiga
-Dificultad para concentrarse o facilidad para quedarse en blanco
-Irritabilidad
-Tension Muscular
-Problemas de sueño
D. La ansiedad, preocupacion y sintomas fisicos provocan un malestar clinico significativo o
deterioro en lo social y/o laboral u otras areas de funcionamiento.
E. El deterioro no es atribuible a los efectos de una sustancia
F. la alteracion no se explica mejor por otro trastorno.
CARACTERISTICAS DIAGNOSTICAS
La intencidad, duracion y frecuencia de la ansiedad y preocupacion son desmedidos.
El S no puede controlar la preocupacion y esta interfiere en la atencion a otras areas de su vida.
Los adultos con TAG se preocupan desmedidamente por cosas cotidianas de la vida, por ejemplo
las responsabilidades en el trabajo, cosas que le pasa a los hijos, etc.
Los niños con TAG suelen preocuparse por su competencia o rendimiento/desempeño.
Las preocupaciones puede transladarse de una cosa a otra.
≠ entre ansiedad no patologica y TAG=
-TAG- preocupacion:
*excesiva q interfiere en en el funcionamiento psicosocial del S.
*Mayor duracion
*muy angustiosas
*se producen casi sin desencadenantes
*se refieren a aspectos variados de la vida cotidiana, cuantos mas aspectos preocupen, mas
chances de padecer TAG
*Malestar subjetivo asociado a las preocupaciones y al deterioro de las areas psicosociales
-Las preocupaciones de la vida cotidiana:
*no son excesivas
*se sienten manejables
*pueden aplazarse si surgen cosas mas graves

84
*no se asocian a sintomas fisicos en gral (agitacion y nervisismo)
CARACTERISTICAS ASOCIADAS QUE APOYAN EL DIAGNOSTICO
Tension muscular
Temblores
Contracciones Nerviosas
Inestabilidad
Molestias musculares/Dolor
Sudoracion
Nauseas- diarrea
Respuesta de sobresalto exagerada
Sindrome de colon irritable/dolores d ecabeza—> sintomas asociados al estres.
NO son frecuentes= Hiperactividad vegetativa- agitacion, aumento de la frecuencia cardiaca,
dificultad para respirar, mareos
PREVALENCIA
Mas chances en mujeres que en hombres
Mas chances de diagnostico en la edad media de vida
Disminuyen las chances en edad avanzada
Las personas de contextos mas desarrollados tienen mas chances de EXPRESAR que han
sufrido los sintomas asociados al TAG
DESARROLLO Y CURSO
- 30 AÑOS, aunque puede detectarse mas temprano (rara vez antes de la adolescencia), es mas
tardia que en otros trastornos de ansiedad
- Tasas de remision completa muy bajas
- Expresion clinica del trastorno a lo largo de la vida- sintomas cronicos y oscilantes
- niños y adolescentes se preocupan mas por el rendimiento deportivo y academico/ desempeño
(suelen ser muy formales, perfeccionistas e inseguros, repitiendo tareas para asegurarse su
rendimiento. Son muy propensos a buscar aprobacion y seguridad)y adultos por la familia y
enfermedades fisicas ppias
- Suele sobrediagnosticarse TAG en niños, hay que asegurarse bien que la preocupacion no se
explica mejor por otro trastorno.
FACTORES DE RIESGO
TEMPERAMENTALES:
Inhibicion de la conducta

85
Afectividad negativa
Evitacion del daño
AMBIENTALES:
No se ha comprobado la intervención de factores ambientales especificos en el desarrollo de TAG
aunque se lo ha asociado a padres sobreprotectores y a adversidades en la infancia.
GENETICOS Y FISIOLOGICOS:
1/3 del riesgo de sufrir TAG es genetico
ASPECTOS DIAGNOSTICOS RELACIONADOS CON LA CULTURA
- El tema de la preocupacion puede variar de cultura a cultura
- Hay que evaluar el contexto social y cultural para definir si una preocupacion es excesiva o no.
ASPECTOS DIAGNOSTICOS RELACIONADOS CON EL GENERO
55/60% de los casos diagnosticados son mujeres
Tienen similar sintomatologia
Diferente patron de comorbilidad: mujeres- otros trastornos de ansiedad y depresion. Varones:
Trastornos por consumo de sustancias
CONSECUENCIAS FUNCIONALES DEL TAG
-Preocupacion Excesiva—>Dificultad para atender otras cosas y resolver de forma eficaz y rápida
en la casa o el trabajo
—> Consumo de tiempo y energia
—> Deterioro facilitado por los sintomas: Irritabilidad/ nerviosismo, fatiga, falta de concentracion,
tension muscular, trastorno dle sueño
—> Las personas que la padecen tienen dificultad para fomentar la confianza en sus hijos
—> discapacidad moderada a grave, en US el TAG genera la perdida de 110 millones de dias al
año por incapacidad.
DIAGNOSTICO DIFERENCIAL
-Preocupacion y ansiedad como causa de otra afeccion medica x ej hipertiroidismo
-Trastorno de ansiedad inducido por medicamentos o sustancias, x ej consumo elevado de cafe
-Trastorno de ansiedad social- ansiedad y preocupacion por razones sociales y la evaluacion de terceros ≠
personas con TAG se preocupan y estan ansiosos haya o no haya un tercero
-Trastorno Obsesivo- Compulsivo, aca la prepcupacion por cuestiones futuras no es normal como
si lo es para el TAG, en el TOC las ideas obsesivas son inadecuadas, pensamientos intrusivos y
no deseados.
-No se debe dagnosticar TAG si las causas de los sintomas pueden ser explicados mejor por
causa de estres postraumatico

86
COMORBILIDAD
TAG acompañada de otros trastornos de ansiedad.
Es menos comun la comorbilidad con trastornos por consumo de sustancias (no dijimos que es
comun esta comorbilidad en hombres???), trastornos de conducta, tratsornos psicoticos, etc

-Bleichmar, H. (1999). El tratamiento de las crisis de pánico y el enfoque Modular


Transformacional". Aperturas Psicoanalíticas, 3. Recuperado de
http://www.aperturas.org/articulos.php?id=94&a=El-tratamiento-de-las-crisis-depanico-y-el-
enfoque-Modular-Transformacional

S/ F, Objetivo del Psicoanálisis: echar luz sobre el inconciente dsd la conciencia, no es facil pero
no imposible.
F—> nos habló de Ataques de Panico en 1895= temores de muerte inminente o de volverse loco
DIFERENCIADO de la Angustia Expectante, no ligada a nada.
Hoy esta diferenciación se mantiene en el DSM IV: Ataque de Pánico Vs Trastorno de Ansiedad
Generalizada
TAG—-> Expectativa ansiosa de supuestos peligros (no delimitados- lo cual aumenta la ansiedad
xq no hay posibilidad de huida) que el S tiene para si o para terceros.
Para Freud, la etiología del Ataque de Pánico es la libido estancada x falta de satisfacción sexual.
En la actualidad, al hablar de crisis o ataque de pánico debemos tener en cuenta:
1-Cuales son las Representaciones, significados y significantes que despiertan la angustia, Cual
es la fuente de angustia?
2-Cual es el proceso cerebral/ hormonal/ corporal que se activa en estos atques?
3-Cual es la Representación conciente o inconciente que el S tiene sobre la angustia
4-Realimentacion de los circuitos anteriores: fuente de angustia, activacion de sistema cerebral,
hormonal, corporal en la crisis de panico, representacion de la Angustia.
Por medio del psicoanálisis se trata de disminuir los factores que desencadenan el ataque de
angustia—> se ataca la fuente de angustia, se van elaborando los conflictos que ponen en
funcionamiento a la angustia.
ANGUSTIA SEÑAL
1era crisis de panico: Angustia Automatica
2da crisis de panico: Angustia Señal que activa los mecanismos de atención por la primer crisis
Una vez que se desencadena la primer crisis de pánico, la angustia ya no depende de las causas
que la originaron por primera vez sino de la situacion que quedó anudada al desencadenamiento
Ataque de Pánico: Situación de desiquilibrio Psicologico. Primero se da por Angustia Automatica,

87
luego, por las reacciones corporales que esta angustia automatica desencadenó. La mente asocia
que aquella crisis provoca Angustia Señal (de estar en alerta), pone en alerta al S porque esa
primera situación de desencadenamiento se puede volver a desencadenar.
Lo importante de esto es que Mi cuerpo reacciono con sensación de ahogo por la irrupción de la
Angustia Automatica, luego cuando me vuelva a sentir mal mi cuerpo va a reaccionar con
sensacion de ahogo y esto provoca angustia, esta vez NO automatica, sino señal.
Ante situaciones que provocaron angustia la primera vez ahora provoca angustia tambien ya no
automatica sino señal de que aquello (la crisis de pánico y sus efectos corporales) volverá a
ocurrir.
DOS NIVELES EN QUE LAS REPRESENTACIONES ICCTES PUEDEN PROVOCAR
ANGUSTIA
Y CRISIS:
- Encadenamientos discursivos Inconcientes: Pienso o digo algo (u otros lo hacen) que me lleva a
la crisis
- Encadenamiento de la crisis con indicios o señales asociados por semejanza o continuidad. Por
ejemplo: la crisis advino en det lugar o en contexto de oscuridad, cuando vuelvo a pasar por ese
lugar o a estar a oscuras infiero que un nuevo episodio puede irrumpir.
REPRESENTACION CTE O INCTE DE LA ANGUSTIA: CODIFICACION DE LA ANGUSTIA
Terapia Cognitiva—> propone “Tecnicas de Exposicion Interoceptiva”, son ejercicios fisicos que
fomentan la reacción corporal de la Crisis de pánico, por ejemplo saltar, girar hasta marearse,
bajar y subir la cabeza bruscamente.
(De esta misma manera dde el conductismo se trata las fobias- acercandose al objeto de la fobia)
De esta manera se le “muestra” al S que NO CORRE PELIGRO ALGUNO. Esta tecnica no es
despreciable pero tampoco es suficiente.
En Psicoanálisis trabajamos con la Representación del Self en Peligro—> El S llega a la
formación de esta representación por varios caminos:
-Se produjo una fijación en situaciones traumáticas donde el S se sintio amenazado o impotente
-Hay conflictos psiquicos que lo hacen sentir en peligro
-Los discursos parentales tiene el poder de de amenazarlo desde el imaginario
-Por identificarse con la representacion de Self en peligro que tiene de si mismo
MULTIPLES REPRESENTACIONES DEL SELF: ORGANIZACION MODULAR
El S tiene representaciones de su ppio self en ≠ terrenos/módulos, separados pero su funcionamiento se
complementa- organización modular del Self:
-Narcisismo: cuanto valgo, soy mejor o peor que el otro
-Autoconservacion: corro peligro? estoy seguro? Tiene que ver con cuanto puede controlar la
angustia sin desorganizarme (volverse loco)

88
En los casos de ataque de pánico hay una desorganización en el modulo de Autoconservacion, la
Representación del S sobre si mismo es en falta de control sobre su mente, su cuerpo y de sus
irrupciones de angustia.
Las perturbaciones en un sector del Self pueden dañar o perturbar otras áreas por ejemplo si hay
alteración en el modulo del Apego (separacion del otro), esto tiene consecuencias en el modulo
de autoconservacion (sentimiento de desproteccion- estar en peligro)
Esta “influencia” de un modulo sobre otro se da por:
-Difusion: perturbación en un modulo causa la perturbación en el otro xq lo que le pasa al primero
transforma lo que pasa en el segundo
-Desplazamiento defensivo: Una perturbación surge para ocultar otro conflicto que la subyace.
GENESIS DEL SENTIMIENTO DE CONTROL/DESCONTROL DE LA ANGUSTIA
Ante la Angustia infantil:
-En el mejor de los casos: Alguien viene en mi auxilio para proveer calma y tranquilidad. Luego
esta figura se internaliza y se apela a ella para autoapaciguamiento—> si esto no sucede, la
sensación de tranquilidad siempre va a depender de la presencia imaginaria de un otro.
-En el peor de los casos: Alguien me castiga por angustiarme, asi la Angustia se internaiza como
señal de Peligro, conflicto o persecucion. En los casos mas severos, no hay forma de calmar la
angustia, nadie puede ayudarlos ante la irrupcion de angustia.
ARTICULACION E/ CTE, ICTE Y NIVEL NEUROBIOLOGICO
La terapia cognitivo conductual, psicoanalisis y psicofarmacologia son efectivas para el
tratamiento de los Ataques de Panico, porque? porque las tres actuan en distintos eslabones de
la cadena de esta problematica:
-El Psicoanálisis opera sobre las fuentes o causas INCONCIENTES de angustia
-La teoria cognitivo-conductual opera sobre la reaccion CONCIENTE ante la angustia
-La Farmacologia opera sobre el nivel Neurofisiologico
Tanto el factor neurofisiologico como el factor representacional operan sobre el Ataque de Panico.
Lo representacional puede producir cambios en el nivel neurofisiologico de forma permanente, sin
embargo a veces lo representacional es tan fuerte que los psicofarmacos o la terapia cognitivo
conductual sobre la reaccion conciente ante la angustia no son suficientes ni del todo eficaces
para revertir el Ataque de Panico.
BIDIRECCIONALIDAD ENTRE CONCIENTE E INCONCIENTE
El objetivo de toda terapia psicoanalítica es aumentar el conocimiento/ el saber de la
concienciacomo? rescatando lo desconocido por ella.
Transferencia—> factor necesario para que la palabra del analista sea escuchada y tenida en
cuenta. Para que esa palabra sea EFECTIVA, el contenido debe ser adecuado.
Las palabras del analista estan dirigidas a la conciencia pero repercuten en el inconciente del
paciente—> no solo el inconciente determina y modifica el estado de la conciencia sino que la

89
conciencia provoca cambio a nivel inconciente x ejemplo el poder del discurso conciente para
modificar un estado de animo, afectar los contenidos de los sueños, fantasias inconcientes, etc.—
> BIDIRECCIONALIDAD ENTRE CONCIENTE E INCONCIENTE
Los psicoanalistas buscamos por medio de nuestras intervenciones:
-Ampliar el saber conciente del paciente
-Resignificar experiencias del pasado
-Identificar que del pasado se transfiere y se repite en y con las figuras actuales
-Aclarar cuales son los mecanismos defensivos que predominan en la conducta
Todo esto CONMUEVE el INCONCIENTE
A pesar de esto hay sectores del Icte que no se conmueven, a los que la palabra conciente no
llega, al igual que no todo lo inconciente es pausible de advenir conciente.
FOCO DE LA ACCION TERAPEUTICA: EL SENTIMIENTO DEL SELF EN CONTROL
Hay dos focos terapéuticos para tratar el Ataque de Pánico:
1- Fuentes de Angustia
2- Reacción ante la Angustia
1- Fuentes de Angustia:
Se trabaja con los conflictos intrapsiquicos e interpersonales
Con Situaciones Traumáticas
Con fenomenos del deficit
Con rasgos de caracter
Todo esto para disminuir los afluentes de ansiedad
ADEMAS++++
Para tratar los A de P, es necesario:
-Potenciar el sentimiento de self en control
-Hacer sentir al paciente que tiene recursos para afrontar la angustia
Lo mas traumatico en la crisis de panico es la perdida abrupta del control sobre el cuerpo y la
mente (generado por la irrupcion de angustia).
El S carece de recursos para enfrentar tal situacion de indefension y omnipotencia ante la
angustia.
Estamos preparados, en gral, para afrontar un peligro que viene desde afuera peor no estamos
listos cuando el peligro es por una desregulacion brusca en el interior.
PARADOJAS EN EL SETTING CLASICO PARA EL TRATAMIENTO DE LA CRISIS DE
ANGUSTIA

90
El divan no sirve para reforzar la seguridad, nos deja en indefensión, incertidumbre y
desconcierto. o al setting del psicoanalisis clasico
El primer objetivo es recomponer el sentimiento de seguridad basico, se requiere la intervención
de un analista que pueda restablecer relaciones objetales de confianza, no sirve de nada el
analista que guarda silencio y es observador, de mantenernos asi el paciente se siente
permanentemente alerta, a la espera de la irrupcion de angustia.
Si abandona el tratamiento no es RTN, es que nuestro acercamiento no fue el ideal para afrontar
sus miedos, todo lo contrario, los “alimentamos”
GENERAR SENTIMIENTO DE CONTROL
Se recomienda el tratamiento en fases:
1- Describir en que consiste el tratamiento, que vamos a hacer, que es lo esperable, así
disminuimos la sensacion de estar en pelotas
2- Nivel informativo: Explicar que es lo que le sucede al paciente y cuales son nuestras hipótesis
para trabajar con lo que le provoca angustia
Explicar que sus sintomas son reacciones fisicas normales al estar alerta.
Esta fase informativa no provoca cambios en las manifestaciones del Paciente
Es importante ir comprobando que el paciente nos sigue, no para saber si nos entendio sino para
hacerlo participe del proceso, NO PASIVIZARLO!
-Crits‐Christoph, P. (2002). Psychodynamic‐lnterpersonal Treatment of Generalized Anxiety
Disorder. (Trad. L. Valladares). Clinical Psychology: Science and Practice, 9(1), 81- 84.*

“Tratamiento Psicodinamico Interpersonal de Trastornos de Ansiedad Generalizada”


Dos autores proponen la integracion de tecnicas basadas en la ACEPTACION con tratamientos
de la terapia cognitivo conductual para el TAG haciendo foco en la PREOCUPACION como una
forma de EVITACION—> MECANISMO DE DEFENSA, Dado que el TAG involucra multiples
elementos es probable que el tratamiento de este trastorno requira del uso de varias tecnicas de
distintas teorias corriente cognitiva, conductual, interpersonal, psicodinamica y aceptacion.
Aproximadamente un 50% de los pacientes con TAG logran un cambio clinico significativos con
Terapia cognitivo conductual, hay que agregar aportes adicionales para aumentar esta proporcion
Las tecnicas basadas en la Aceptacion x ejemplo. Porque la preocupacion, que es una forma de
evitacion es caracteristico de los TAG y se recurre a ello como una forma de evitar pensar en
cosas mas perturbadoras
Preocuoacion—> forma de evitacion—> mecanismo de defensa para no pensar en temas y
recuerdos perturbadores
Tratamiento PSICODINAMICO-INTERPERSONAL para los TAG de APOYO EXPRESIVO
CONCEPTUALIZACION PSICODINAMICA DE LA PREOCUPACION Y ANSIEDAD

91
La preocupacion es en los TAG una forma de evitar otras cuestiones perturbadoras como
Mecanismo de Defensa. Esto es lo que F planetaba sobre la defensa que se activa para evitar
pensar en cosas dificiles, conflictos y traumas.
Frente a una pequeña señal de peligro —> ansiedad—> esto le envia una señal al Yo para estar
alerta por la amenaza—> los mecanismos de defensa se activan para mantener la amenaza lejos
de la conciencia—> asi evita desarollar un trauma mayor
El tratamiento se basa en el peligro, que el paciente entienda que el peligro no es tan grande
como lo imagina. Eliminar la estrategia utilizada para evitar la posible emergencia de ansiedad
disminuye la ansiedad en si.
Teoria interpersonal psicodinamica: el niño con cuidadores sobreprotectores o indiferentes
desarrollan una “ansiedad de base”, falta de confianza en si mismo y en los demas.
Segun Fairbairn, otro factor que incide en el desarrollo de la ansiedad es la dependencia del niño
con el cuidador primario y el miedo de perdida de identidad Klein, evoco la ansiedad
comoproducto de que el niño no puede contar con el cuidador cuando lo necesita.
En todos los casos, la representacion internalizada del yo y de los otros, producto de los
intercambios con el cuidador primario afecta las relaciones interpersonales futuras y producen
ansiedad.
Impacto de las relaciones interpersonales entre cuidador y niño y la ansiedad!
TERAPIA PSICODINAMICA DE APOYO-EXPRESIVA DEL TAG
Incidencia de los factores interpersonales + preocupación como mecanismo de defensa de
evitación
La Técnica de apoyo- expresivo se basa en las siguientes hipotesis:
-Como consecuencia de experiencias interpersonales traumaticas en etapas tempranas de la vida y en la
actualidad! (≠ con el psicoanalisis clasico que piensa en solo relaciones interpersonales de la infancia- aca se
abarcan relaciones actuales tmb), se generan deseos, expectativas, creencias y sentimientos sobre uno mismo
y los otros sobre:
…Que uno debe obtener amor, estabilidad y proteccion de los otros pero que corremos el riesgo
de que ese otro nos abandone, maltrate, decepcione o critique…
ANSIEDAD conectada a esos sentimientos y creencias MUY FUERTE—> la persona con TAG
evita pensar en los recuerdos que generaron esos temores
como?—> desarrollando una PREOCUPACION cognitivamente EXCESIVA por eventos actuales
de la vida.
Una vez establecido este conjunto de creencias, sentimientos y deseos sobre mi y los demás
“patologico” se forma un sistema de retroalimentacion ciclica que recrea las circunstancias que
generaron la ansiedad
La tecnica de apoyo- expresivo opera sobre los patrones de relacion de esos vínculos
interpersonales conflictivos
Este Core conflictual relationship Theme tiene Tres elementos:
-Deseos o expectativas

92
-Respuesta percibida o esperada de los otros
-Respuesta del Yo
La 1er tarea expresiva- exploratoria del terapeuta en el tratamiento de apoyo-expresivo es
formular el conflicto relacional central (CCRT- Core Conflictual Relationship Theme) para cada
paciente y esto es lo que guiara las intervenciones
La ppal tarea de apoyo del terapeuta es crear y mantener una alianza terapéutica positiva
La ansiedad responde a diversas fuentes, la ppal es el temor a no obtener lo que uno espera de
las relaciones
El componente de preocupacion de la ansiedad es un mecanismo de defensa. Otros sintomas de
la ansiedad, como sintomas somaticos tambien son RESPUESTAS - mecanismos de defensa, es
decir centrarse en los sintomas corporales como forma de evitar las emociones.
Los acontecimientos de la vida pueden sumar razones “reales” a la ansiedad que deviene de las
relaciones pasadas
Modelo basado en la aceptacion= preocupacion como mecanismo de defensa + TCC= Hincapié
en las relaciones interpersonales (pasadas y actuales), como nucleo de donde proviene esta
ansiedad = Modelo de apoyo expresivo= Este es un modelo de tratamiento breve de apoyo
expresivo
(SE- Supportive expressive phychodynamic model) psicodinamico.
En un tratamiento breve de 12 a 20 sesiones, de TCC se dedica poco tiempo al tema de las
relaciones interpersonales, se enseñan habilidades de relajacion, monitorear pensamientos
automaticos, examinar evidencias de las creencias del paciente, generar interpretaciones
alternativas y otras tecnicas de la TCC orientadas a la aceptacion.
En el modelo SE se destina extenso tiempo para que el paciente cuente en detalle sus
experiencias interpersonales, recuerdos, sentimientos y comprension de los acontecimientos. Rol
activo del terapeuta para alentar el relato especifico sobre la interaccion con otros.
En este modelo el terapeuta es menos directivo que en la TCC y deja al paciente explayarse a su
propio ritmo sin embargo es directivo, no se comporta como en el psicoanalisis clasico, como
pantalla en blanco.
No es puro psicoeducacion de enseñar tecnicas de relajacion, monitoreo, metodos de solucion de
problemas y competencias para la aceptacion sino que tiene un buen equilibrio entre esto y
relatos sobre las relaciones interpersonales, no se pone al paciente en el lugar pasivo de
recepcion de informacion—> esto (no lugar pasivo) disminuye la preocupacion.
ACEPTACION DE LAS CONTRIBUCIONES PSICODINAMICAS
Hay una superposicion conceptual entre las TCC y los modelos psicodinamicos en cuanto al
abordaje del TAG.
Critica pelotuda de los CC a los modelos psicodinamicos por tacharlos de menos empíricos.
Al final plantea la misma puta cosa que dijo mil veces y que todos sabemos, en el abordaje de
TAG es conveniente integrar de forma FLEXIBLE distintos modelos, psicodinamicos/
interpersonales, cognitivos, conductuales y basados en la aceptación.

93
-Fonagy, P. Roth, A. y Higgitt, A. (2007). Psicoterapias psicodinámicas: práctica basada en la
evidencia y sabiduría clínica. (Trad. M. González Bas). Aperturas Psicoanalíticas, 27 (Original en
inglés, 2005). Recuperado de http://www.aperturas.org/articulos.php?id=61
En teórico 1

PRÁCTICO SEMANA 6
Etchevers, M., Putrino, N., Giusti, S., Helmich, N., Cantaro, M., Tabullo, Á., y Argibay, P. (2015).
Relación entre paciente y terapeuta: investigación, práctica y docencia. En E. J. Huaire Inacio, Á.
M. Elgier y G. Maldonado Paz (Comps.), Psicología cognitiva y procesos de aprendizaje. Aportes
desde Latinoamérica (pp. 271-284). La Cantuta-Chosica, PE: Universidad Nacional de Educación
Enrique Guzmán y Valle.
LA RELACION TERAPEUTICA EN PSICOTERAPIA
Diversos estudios revelaron que los resultados de la terapia estan relacionados con el vinculo/
relacion establecido entre paciente y terapeuta.
Antecedente:
Transferencia de F. repeticion de modelos infantiles inconsientemente en un objeto/ persona
actual.
La contrapartida de este antecedente es la Contratransferencia, es el influjo inconsciente que el
paciente ejerce en el analista, la recomendacion es detectarlo y dominarlo a partir del análisis
propio del analista.
Posteriormente, otros autores (dentro y fuera del psicoanalisis) reconocieron el valor terapéutico
del analisis de la contratransferencia.
El concepto de alianza terapeutica permitio incluir cierta flexibilidad terapeutica en una epoca en
donde el psicoanalisis tradicional/clasico no dejaba mucho espacio para que los terapeutas se
adapten a las necesidades unicas de cada paciente y a dar lugar a una poblacion cada vez mayor
de pacientes no neuroticos.
Tambien permitio incluir otra manera de entender lo que sucedia en la sesion entre terapeuta y
paciente, siendo que a partir de esta nocion, NO TODO lo que sucede en terapia es producto de
la transferencia de mociones infantiles reprimidas inconcientes. Esta dimension comprende los
aspectos NO TRANSFERENCIALES DE LA RELACION TERAPEUTA/ PACIENTE.
ALIANZA TERAPEUTICA, la mayoria de las definiciones actuales coincide en:
-Naturaleza colaborativa de la relacion entre ambos
-El vinculo afectivo entre paciente y terapeuta
-Capacidad conjunta para acordar problemas a resolver, objetivos y estrategias
La APA creo la DIVISION 29: FUERZA DE LA TAREA EN PSICOTERAPIA CON APOYO
EMPIRICO: RELACION TERAPEUTICA con el objetivo de A)detectar los elementos que hacen
eficaz la Relacion Terapeutica y B)determinar que tipo de terapia se ajusta mejor a cada paciente.

94
Los estudios que analizan la RT fueron a la vez analizados (sometidos a un gmeta analisis") y se
descubrio que los pacientes evaluan la RT relativamente estable durante la terapia mientras los
terapeutas y observadores externos si pueden detectar variaciones en la Relacion durante el
tratamiento. Esto nos deja de conclusion que si el paciente mantiene estable su percepcion de la
RT es importante establecer una relacion positiva desde el inicio.
Antes los estudios se basaban en encontrar relacion entre la AT y los Rdos del tratamiento, ahora
lo que buscan las investigaciones es
A) centrarse en las caracteristicas y habilidades profesionales que deben poseer los psicologos
para favorecer una buena RT y
B)El estudio de la ruptura dentro del proceso terapeutico.
CARACTERISTICAS Y HABILIDADES DE LOS TERAPEUTAS: EMPATIA
En diversas y numerosas investigaciones, psicologos y estudiantes de psicologia senalaron a la
EMPATIA como una caracteristica esencial para el desempeno del rol de terapeuta y de su
profesion.
Otras investigaciones indican que la capacidad empatica del terapeuta puede predecir los
resultados de la terapia, hay entre ambas variables una correlacion positiva. La alianza
terapéutica tambien es predictor de los resultados de la terapia.
Otros autores han senalado que la empatia y la Relacion terapeutica explican mejor la varianza
de resultados terapeuticos que las intervenciones terapeuticas especificas. (es decir que es mas
importante tener buena empatia y rel terapeutica que acertarle a la intervencion especifica)
Un estudio que comparaba la eficacia de tratamientos psicoanaliticos vs psicodinamicos tras un
seguimiento de 5 anos arrojo que los pacientes presentaban menos sintomas cuando los
terapeutas psicoanaliticos se mostraban menos distantes y mas asertivos.
En vaaarios estudios se llego a la misma conclusion empatia= buenos resultados, mejorias.
Empatia.> gCapacidad de comprender los pensamientos y sentimientos de los demas,
atendiendo y comprendiendo que los mismos son ‚ a los propiosh
Es una caracteristica que todo profesional de la salud mental deberia tener para ejercer como
psicoterapeuta.
Empatia = Buena Relacion Terapeutica = Buenos Resultados Terapeuticos.
LAS RUPTURAS DE LA RELACION TERAPEUTICA
La Relacion Terapeutica no es estatica, esta en constante cambio y re-negociacion entre
terapeuta y paciente conciente e inconciente. Esta relacion puede sufrir rupturas a causa de
factores que provengan del paciente o del terapeuta y pueden no ser detectadas conscientemente
por ninguno de los dos ni danar significativamente el progreso terapeutico o bien pueden detener
de forma prematura el tratamiento. Pueden variar en intensidad, duracion y frecuencia.
Dos senales de ruptura inminente que el terapeuta deberia detectar:
-Confrontacion del paciente, se opone al terapeuta
-Abstinencia, el paciente responde con conductas de retirada, aplazo o incomplimiento.
Causas:

95
-desacuerdo en los objetivos y tareas entre paciente y terapeuta, estrategia de intervencion ante
la ruptura directa o indirecta.
-desacuerdos en el vinculo relacional, directos o indirectos, estrategia de intervencion ante la
ruptura directa o indirecta.
Estas rupturas pueden ser reveladoras para el terapeuta porque develan modos de relacionarse
del paciente dentro y fuera de la terapia. Ayudan a una comprension mas profunda.
FORMACION EN RELACION TERAPEUTICA. VENTAJAS EN LA PRACTICA PSICOLOGICA.
Una buena formacion en habilidades interpersonales favorece la relacion terapeutica, esto nos
habla de la importancia en la formacion de grado y posgrado del psicologo en este ambito.
Si bien muchos manuales y guias de tratamiento y practica hablan de la importancia de la relación
terapeutica, no hay mucha explicacion de cuales son las cualidades del terapeuta o cuales son
las intervenciones que fomentan una relacion curativa.
Si bien la RT es fundamental, aisladamente no es suficiente para llevar a cabo un buen
tratamiento, son necesarias otras herramientas tecnicas especificas.
Es ideal que la formacion incluya:
-Entrenamiento en las reglas de funcionamiento
-En las habilidades relacionales
-Manejo d elos propios sentimientos
-Creatividad en el proceso terapeutico
Etchevers, M., Giusti, S. y Helmich, N. (2017). Revisión de las nociones de la Alianza Terapéutica,
sus antecedentes conceptuales y aportes a la investigación. Revista Universitaria de
Psicoanálisis, 17, 57-67. Disponible en:
http://www.psi.uba.ar/investigaciones/revistas/psicoanalisis/trabajos_completos/revista17/
helmich.pdf
En teórico semana 5
Greenson, R. y Wexler, M. (1969) - La relación no transferencial en la situación analítica. En
International Journal of Psicoanálisis. Vol. 50, parte I.

INTRODUCCION
Para la Teoría Psicoanalítica el FOCO CENTRAL esta puesto en el ANALISIS DE LA
TRANSFERENCIA. Es mas, para los ortodoxos todo lo que no pertenezca al ámbito de la
transferencia es irrelevante, aunque de esto podemos deducir que al menos, reconocen que hay
interacciones personales que están claramente diferenciadas de la transferencia.
En la actualidad, algunos Psicoanalistas si han empezado a ponderar las “Interacciones NO
Transferenciales” o tambien denominadas “Reales” (pensando en que un poco la interacción
transferencial no es real porque los afectos no le pertenece al medico). Sin embargo no son
claros sobre como manejar este material.
-Es importante:

96
Reconocer
Clarificar
Diferenciar
Promover …
…las reacciones Libres o Reales entre paciente y terapeuta.
Sin olvidarnos de la relación transferencial que es clave para la terapia psicoanalitica pero
incluyendo la relación no transferencial con el paciente.
DEFINICIONES OPERATIVAS
TRANSFERENCIA:
Impulsos, sentimientos, fantasias, defensas y actitudes orientados hacia una persona del
presente, en nuestra profesión: el analista, a quien no le pertenecen, no le corresponden porque
en verdad se trata de fenomenos que son una Repeticion de una Relacion con personas
significativas de la infancia.
Mociones afectivas que son inconscientemente desplazadas a una persona del presente por falso
enlace.
Esta repetición es:
-no selectiva del pasado
-está distorsionada de la realidad, es inadecuada.
Se diferencia de…
RELACION REAL/ NO TRANSFERENCIAL:
Es Relevante y Adecuada, las respuestas no transferenciales son realistas.
De todas formas es relativo, porque no hay relación transferencial sin componente de realidad ni
relación Real sin transferencia. Aunque teórica y técnicamente sea necesario diferenciarlas.
RELACION ENTRE ALIANZA DE TRABAJO, TRANSFERENCIA Y REL REAL.
ALIANZA DE TRABAJO:
-Relación razonable, real y NO neurótica que el paciente tiene con su analista, es lo que le
permite trabajar intencionalmente en el análisis.
-El paciente debe tener conciencia de su enfermedad o padecimiento y entender que el terapeuta
lo puede ayudar
-La transferencia positiva y sobrevaloracion del terapeuta ayudan a la formación y mantenimiento
de la Alianza de Trabajo pero la mayor fuerza de la Alianza de Trabajo proviene de la Relación
Real entre analista y terapeuta, xq? porque los componentes transferenciales pueden
transformarse en una gran fuente de resistencia.
-El analista contribuye a la Alianza de Trabajo con:
*busqueda del insight del paciente.

97
*respetando y cuidando la personalidad del paciente, su parte sana y la enferma.
*ayudandolo a discernir entre lo real y lo fantaseado, lo correcto y lo falso, lo adecuado y lo
distorcionado en su reaccion y en la de los demas.
Por medio de las REACCIONES TRANSFERENCIALES el paciente trae el material inaccesible a
la consulta.
Por medio de la ALIANZA DE TRABAJO el paciente logra comprender el insight del analista y sus
interpretaciones y le permite reorganizar, integrar y asimilar el trabajo del analisis.
Para que haya Alianza de Trabajo el paciente debe tener la capacidad de establecer con el
analista una RELACION REAL NO TRANSFERENCIAL.
ANALIZAR—> es abreviar/resumir todos los procedimientos que buscan aumentar el insight del
paciente sobre si mismo.
El PROCEDIMIENTO ANALITICO + importante es la INTERPRETACION, todos los demas están
de alguna manera subordinados a ella, la confrontacion, clarificacion, elaboracion—> estos
pueden no estar orientados a ampliar el conocimiento de lo inconciente per se, pero contribuyen a
la fortaleza yoica que le permitirá al paciente comprender (preparan al yo para insight).
xEj: la Abreaccion, por medio de la descarga emocional masiva, el Yo queda libre de energia para
poder ocuparse del insight.

PROCEDIMIENTOS ANTIANALITICOS:
-Son aquellos que obturan la comprensión y capacidad de insight del paciente
-Es cualquier medida que implique la perdida de la capacidad del Yo de observar, pensar y
recordar
-xEj: Gratificaciones transferenciales innecesarias, seria ceder ante la demanda del paciente
REVISION HISTORICA
En el ambito del psicoanalisis no hay consenso sobre la Rel. No Transferencial.
Es la Rel directa entre paciente y analista que existe independientemente de la transferencia, la
Rel Real o no transferencial es el medio en el cual ocurre la transferencia.
Algunos Pacientes pueden soportar la distancia del analista (Distancia Optima) y otros necesitan
mas cercania, el analista en esos casos debera permitir que se sienta su presencia.
En los pacientes Neuróticos habrá prevalencia de la Interpretacion, cuyo medio de desarrollo es la
Transferencia. En cambio en aquellos pacientes que sufren psicosis o patologias relacionadas
con el deficit (patologias del narcisismo, etc), habra prevalencia de Relacion Real o “no
transferencial”, esta Relacion Real ayuda a cultivar Relaciones Objetales reparadoras con el
analista.
EJEMPLOS CLINICOS
-Dependiendo de la patologia del paciente y la terapia que se aplique en rigor a la primera, sera la
importancia a otorgarle a la Relación Real en la terapia psicoanalítica.

98
-Es importante conocer con mucha objetividad sobre nosotros mismos, nuestros pacientes, la
cultura, el medio, etc para poder determinar si una reacción, actitud o respuesta corresponde a
una transferencia o a una Respuesta Real.
-Si bien los analistas tenemos mas chances de conocer o detectar cosas sobre el paciente, ellos
tambien pueden saber de nosotros por medio de como nos comunicamos, el manejo del silencio,
como interpretamos, como saludamos, etc, porque todo eso habla de nosotros y nuestra
personalidad mas alla de nosotros como profesionales.
-Es importante entender que ante comentarios hirientes o negativos del paciente sobre nosotros
no debemos plantarnos en un lugar defensivo alegando que se trata de Resistencias que se
sirven de la Transferencia de mociones pulsionales que no nos pertenecen. Hay que saber
escuchar, quizas no se trata de material transferencial pausible de interpretación, quizas es algo
que proviene de la Relacion Real.
La forma de abordar esto no deberia ser rta silenciosa sino un intercambio franco al respecto
entre ambos—> no hacerse el boludo
Siempre hay interjuego de relacion Transferencial y Relacion Real, incluso en una misma sesión
quizas…
xEj. paciente escritor que se sentia amenazado por las relaciones calidas y amorosas, toma el
aspecto calido del analista (Relacion Real) para enojarse con él porque es “debil" y no le da
directivas claras y “agresivas” sobre que hacer como si haria el analista de su esposa, al cual
admira, a su ppio analista lo desprecia. Esto se relaciona con su vinculo transferencial con él, su
madre amorosa y tierna es el analista. Se interpreta que el paciente no respeta de modo
defensivo a su analista, porque teme los afectos calidos, lo ponen en situacion vulnerable y de
dependencia.
Las reacciones transferenciales son ambivalentes, y no se puede pensar que la permanencia en
el tratamiento depende de que la transferencia positiva neutralize la hostil sino que es la Alianza
Terapeutica basada en la Relacion Real entre paciente y analista la que mantiene el tratamiento y
la fuerza para el trabajo eficaz y continuo.
Lo mismo pasa con las interpretaciones incorrectas, el paciente no huye una vez producidas
aunque las perciba, eso es gracias a la relacion real. La rel. Real es el soporte al arduo y duro
trabajo de la elaboracion.
En psicoanalisis el fracaso de la tarea o la crisis sobreviene cuando un error en la interpretación
coincide con el debil relacionamiento real que no es capaz de soportar los embates del
dispositivo.
Los errores tecnicos pueden suceder y s epueden sortear, los errores de actitud humana no.
Las Reacciones transferenciales y no transferenciales tmb existen fuera de la situacion analitica,
el analista debe ayudar al paciente a diferenciar fantasia de realidad en su vida en gral…
En pacientes psicoticos o fronterizos es incluso mas importante aun el ayudarlos a percibir y
distinguir las percepciones de la realidad, no trabajar en el ámbito de la relación real y dedicarnos
exclusivamente a la interpretación de la transferencia y la resistencia lleva al retraimiento del
paciente. Es importante brindarle elementos/percepciones de la realidad en los que se pueda
apoyar y utilizar como si fuesen propios.

99
Ejemplo clinico: Paciente con crisis psicotica luego de separacion de su esposo y durante el
proceso de divorcio. No cuestionar la verdad o falsedad del relato del paciente, no demostrar
contradicciones sino que el terapeuta busca demostrarle a la paciente que alli hay alguien que la
escucha, que toma ese material con calma y objetividad SIN afirmar si eso es real o no—> El
analista busca consolidar la Alianza Terapéutica. Las preguntas del analista buscan ampliar el
material y no a tenderle ninguna trampa para que caiga la veracidad del relato.
CONSIDERACIONES TECNICAS
Todo tipo de pacientes (neuroticos, psicoticos y fornterizos) tienen Relaciones transferenciales
dentro del dispositivo analítico y en su vida “exterior” al análisis.
Para Greenson y Wexler SOLO SON ANALIZABLES los que ademas puedan establecer una
RELACION REAL con su analista, una relación libre de transferencia xq ésta es la CONDICION
PARA METERSE EN EL ANALISIS: establecer la Alianza Terapéutica.
Aquellos que no tengan esta capacidad para establecer una Rel. Real, deben pasar por una etapa
de “psicoterapia preparatoria” en donde se ayuda al paciente a construir una Relacion Objetal
basada en percepciones, juicios y respuestas confiables.
COMO ESTABLECER ESTA RELACION REAL?
-Para crear una Rel Real hay que crear una atmósfera analítica productiva, como?
*Esfuerzo+disciplina+Restricción x parte del analista
*El analista debe demostrar su preocupación por ALIVIAR el sufrimiento del paciente y
administrando dosis tolerables de Insight al paciente
*Explicar porque aplicamos determinado método y porque trabajamos de det. manera, xEj porque
apelamos a la asociacion libre, porque elegimos el uso del divan, porque no repondemos det
preguntas, etc.
*Dosificando las Intepretaciones de modo que el paciente no tenga que lidiar con demasiados
insights nuevos y dolorosos a la misma vez.
*Ser razonables con las circunstancias del paciente y ellos con las nuestras (horarios, honorarios,
etc)
*Si no tenemos una interpretacion no apresurarnos para demostrar comprension, tampoco dejar ir
al paciente en silencio: Podemos decir/verbalizar que no estamos seguros de que fue lo que
sucedio/ transcurrio en la sesion y tmb podemos pedirle al paciente que nos diga que se le ocurre
a él que pudo haber sido lo que trajo a la sesion.
*Escuchar los insights del paciente y respetarlos por el esfuerzo significativo aunque no estemos
100% de acuerdo con el contenido.
*Admitir errores de nuestra tecnica y de nuestra labor
*En pacientes psicoticos es importante que la sesion sea cara a cara para ayudar en la
adaptación a la realidad y a la vez mantener distancia fisica y psicologica adecuada—> esto
ayuda a establecer relaciones objetales reparadoras y reestablecer la representacion objetal
interna.
CONSIDERACIONES TEORICAS

100
Nuestra meta Terapeutica será:
-Aumentar las funciones Yoicas sanas del paciente
-Aumentar la capacidad de establecer Relaciones Objetales
-Brindar la misma importancia a la Relacion Transferencial y a la Relacion Real o No
Transferencial
-Si buscamos que nuestros pacientes desarrollen funciones yoicas sanas y relaciones objetales
plenas debemos permitirles experimentar en la situación analítica tanto los aspectos reales como
los no reales de sus relaciones con otros objetos.
La consideración de la Relación No transferencial es “subversiva” para el psicoanálisis clásico y
tradicional.

TEÓRICO SEMANA 7
-American Psychiatric Association (2014). Trastornos depresivos (Depresión mayor y Trastorno
depresivo persistente/Distimia). En Manual diagnóstico y estadístico de los trastornos mentales
(5ª ed., pp. 155–188). Arlington, VA: American Psychiatric Publishing.
DSM 5- TRASTORNOS DEPRESIVOS
Lo incluyen:
-Trastorno de desregulacion disruptiva del estado de animo
-Trastorno de depresion Mayor—> es el trastorno clasico de este grupo
-Trastorno Depresivo persistente- DISTEMIA
-Trastorno disforico pre menstrual
-Trastorno depresivo inducido por consumo de sustancias
-Trastorno depresivo por otra afección medica
-Trastorno depresivo especificado
-Trastorno depresivo no especificado
Rasgo común:
*Animo triste, vacio o irritable
*cambios somaticos y cognitivos que afectan significativamente la capacidad funcional del S.
Los diferencia:
*Duracion
*presentacion temporal
*supuesta etiologia
TRASTORNO DE DEPRESION MAYOR

101
CRITERIOS DIAGNOSTICOS
A. Cinco o mas sintomas presentes en un periodo de 2 semanas y al menos uno de los sintomas
es:
-Estado de animo deprimido
-Perdida de interes o de placer
1. estado de animo deprimido la mayor parte del dia casi todos los dias segun se desprende de lo
que el S dice o lo que otros informan sobre el S. En niños y adolescente pueden tener animo
irritable.
2. disminucion del interes o del placer por todas o casi todas las actividades la mayor parte del dia
casi todos los dias.
3. Perdida de peso importante sin hacer dieta o aumento de peso significativa/ perdida o aumento
del apetito
4. insomnio o hipersomnia
5. Agitacion o retraso psicomotor casi todos los dias
6. Fatiga o perdida de energia casi todos los dias
7. Sentimiento de inutilidad o culpabilidad exagerada casi todos los dias (autorreproche o culpa
por estar enfermo)
8. Disminucion en la capacidad para pensar o concentrarse o de tomar decisiones casi todos los
dias. Memoria afectada.
9. Pensamiento de muerte recurrente, ideas suicidas e intentos de suicidio.
B. Estos sintomas causan un MALESTAR significativo y DETERIORO en lo social, laboral y otras
areas de funcionamiento importantes para el S.
C. estos episodios no pueden adjudicarse a otras afecciones medicas o a consumo de sustancias
Nota: Una perdida significativa puede conllevar respuestas como perdida de peso/apetito,
insomnio, rumiacion sobre la perdida, animo triste, etc y pueden simular un episodio depresivo
pero estos sintomas son propios de una respuesta normal ante una perdida significativa.
Quien deberá definir si es depresion mayor o rta normal ante una perdida significativa sera un
profesional medico que sepa sobre la historia clínica del paciente.
D. El episodio de Depresion Mayor no se explica mejor por un trastorno esquizoafectivo,
esquizofrenia, trastorno delirante u otro trastorno especificado o no psicotico.
E. No presenta a la vez episodios maniacos o hipomaniacos.
CARACTERISTICAS DIAGNOSTICAS
El ppal rasgo del episodio de depresion mayor es el estado de animo triste o depresivo y la falta
de placer o perdida del interes en casi todas las actividades casi todos los dias por al menos 2
semanas.

102
En algunos pacientes con episodios leves su funcionamiento puede parecer normal pero en
verdad esta haciendo tremendo esfuerzo.
Muchos pacientes se muestra con irritabilidad aumentada. Este animo irritable es mas comun que
la tristeza en niños y adolescentes.
Es normal una reduccion significativa del apetito o interes sexual (al igual que se reduce el interés
por actividades de las que antes gustaba como hobbies, trabajo, actividades sociales, etc)
Cambios motores como agitacion o lentificacion se hacen evidentes en el habla tambien (pausas,
tarda en responder, como que le cuesta pensar, hablan con volumen bajo, etc.
Fatiga sin haberse sometido a una actividad fisica o que demande esfuerzo excesivo, actividades
normales le cuestan el doble. Esto trae aparejado que es posible que pierda eficacia al realizar
sus tareas habituales.
Dismunicion del sentimiento de si, impotente, minusvalido, estas evaluaciones negativas sobre si
mismo no tienen sustento real, se utilizan cuestiones nimias de la vida como confirmadores de su
condicion inferior.
Pensamiento suicida, la idea de que los demas estarian mejor si él estuviese muerto. Suicidio
como unica salida al dolor emocional intenso y para rendirse ante los obstaculos que aparecen
para él como insuperables.
PREVALENCIA
Mujeres sobre hombres un 1,5/ 3. inicio comun en la adolescencia.
DESARROLLO Y CURSO
El trastorno depresivo puede aparecer por primera vez a cualquier edad pero la probabilidad de
que se inicie aumenta en la pubertad.
El curso del trastorno depresivo es bien ≠ en los pacientes, puede haber algunos donde hay largos periodos
de remisión o con solo 1 o 2 sintomas leves.
La cronicidad del trastorno depresivo aumenta las posibilidades de desarrollo de otras afecciones,
como trastornos de la personalidad o de ansiedad, consumo de sustancias lo que hace casi
inviable la remision del trastorno depresivo, por esto es importante pedirle al paciente cuando
consulta, que recuerde cuando fue la ultima vez que presento estos sintomas por mas de 2
semanas.
Recuperacion: al 3er mes del inicio del tratamiento en 2/5 pacientes y al año en 4/5 pacientes.
El riesgo de recurrencia disminuye cuanto mas largos son los periodos de remision. En cambio
hay mayor riesgo de recurrencia en persona jovenes, en aquellos cuyo episodio anterior fue grave
y en las personas que han presentado varios episodios.
La presencia de sintomas depresivos leves durante la remision es un factor que predice la
recurrencia.
Muchas enfermedades comienzan con episodios depresivos, es muy comun que los trastornos
bipolares sean primero diagnosticados como depresivos.
FACTORES DE RIESGO Y PRONOSTICO

103
TEMPERAMENTALES: La afectividad negativa es un riesgo para el comienzo de un trastornos
depresivo mayor, peden desarrollar trastorno depresivo ante situaciones vitales estresantes.
AMBIENTALES: Acontecimientos vitales estresantes son precipitantes de episodios depresivos
mayores
GENETICOS Y FISIOLOGICOS: Los familiares directos de pacientes con trastorno depresivo
tiene 3 o 4 veces mas riesgo que la poblacion general.
MODIFICADORES DEL CURSO: Es comun que se presenten episodios de depresion mayor con
otros trastornos como el de personalidad de ansiedad y consumo de sustancias, estos trastornos
son dificiles de detectar detras de los sintomas de la depresion.
Las enfermedades medicas crónicas o discapacitantes aumentan las posibilidades de episodios
de depresión mayor, las cuales se ven aun mas complicadas por la aparicion de la depresion.
TRASTORNO DEPRESIVO PERSISTENTE (DISTIMIA)
A. Estado de animo deprimido durante la mayor parte del dia, casi todos los dias durante al
menos dos años. En niños y adolescentes en vez de estado de animo depresivo puede ser
irritable y la duracion minima de un año.
B. Presencia de dos o mas de los sgtes sintomas:
-Bajo apetito o apetito aumentado
-Insomnio o hipersomnia
-Fatiga/baja energia
-Baja autoestima
-Falta de concentracion y dificultad para tomar decisiones
-Desesperanza
C. Durante 2 años, o 1 en niños y adolescentes, el S no ha estado sin sintomas del criterio A y B
por mas de 2 meses seguidos.
D. Los criterios para la depresion mayor pueden estar presentes non stop por 2 años
E. Sin episodios maniacos o hipomaniacos durante el trastorno
F. El trastorno no se explica mejor por otras afecciones psicologicas o medicas ni por el consumo
de sustancias
G. Los sintomas implican malestar significativo y alteracion en la vida social, laboral, etc.
CARACTERISTICAS DIAGNOSTICAS
Aquellos pacientes que presenten sintomas del trastorno depresivo mayor por dos años deben
ser diagnosticados con trastorno depresivo persistente ademas de trastorno depresivo mayor
DESARROLLO Y CURSO
En general es de inicio temprano (infancia, adolescencia o juventud- antes de los 21 años) y por
definicion tiene caracter cronico.

104
Al ser de comienzo temprano tiene mayor comorbilidad con trastornos de la personalidad y
trastornos por consumo de sustancias
FACTORES DE RIESGO Y PRONOSTICO
TEMPERAMENTALES: Menos posibilidad de mejora (peor pronostico) en personas con
afectividad negativa, mayor gravedad de síntomas, presencia de trastornos de ansiedad o de
conducta, funcionamiento global comprometido.
AMBIENTALES: Factores de riesgo durante la infancia seria la perdida de los padres o
separación de ellos.
GENETICOS Y FISIOLOGICOS: Los pacientes con trastorno depresivo persistente / distemico, al
igual que aquellos con trastorno depresivo mayor, tengan familiares de grado 1 con trastorno
depresivo
Varias regiones cerebrales implicadas en los trastornos depresivos persistentes (la amigdala, el
hipocampo, cortex pre frontal y el cingulo anterior La repercusión en la vida social y laboral del
trastorno depresivo persistente es mas grave que para los casos de trastorno depresivo mayor.
DIAGNOSTICO DIFERENCIAL
1- Trastorno depresivo Mayor: Si el paciente presenta animo deprimido mas do o mas síntomas
definidos para el tratorno depresivo eprsistente por dos años o mas, entonces es trastorno
depresivo persistente.
Ahora, si hay criterio para diagnosticar trastorno depresivo mayor en esos dos años (intento de
suicidio, agitacion o retraso motor, culpa y sentimiento de inutilidad) se lo codifica como un
especificador del trastorno depresivo persistente “con episodios intermitentes de depresión
mayor” con episodio actual o sin episodio actual
2- puede co existir con trastornos de la personalidad, deben coexistir los dos diagnósticos

-Bleichmar, H. (2003). Algunos subtipos de depresión, sus interrelaciones y consecuencias para


el tratamiento psicoanalítico. Aperturas Psicoanalíticas, 14.
http://www.aperturas.org/articulos.php?id=245&a=Algunos-subtipos-de-depresion-
susinterrelaciones-y-consecuencias-para-el-tratamiento-psicoanalitico

F. en duelo y melancolia planteo la depresion como “la reaccion ante la perdida de un objeto real
o
imaginario”
Hay un anhelo por el objeto que no puede satisfacerse, a la perdida de objeto se le suma un
deseo/anhelo por recuperarlo (deseo irrealizable)
Perdida de objeto + anhelo/deseo por recuperar ese objeto + Representacion de que ese deseo
es irrealizable (esto se relaciona con el sentimiento d ela persona de encontrarse sin poder para
modificar las cosas)—> papel central d ela impotencia o indefension en el desarrollo del
fenomeno depresivo

105
El deseo insatisfecho en estado de depresion es un deseo que apunta a un estado ideal de
bienestar. El objeto que se quiere recuperar es el que provee bienestar y felicidad
Es importante ≠ entre respuestas o reaccion depresiva (indefension ante el dolor o el sufrimiento) vs. estados
de infelicidad o dolor (discrepancia entre un ideal y la realidad)
ESTADO DEPRESIVO:
-Fijacion a un deseo irrealizable- recuperar el objeto perdido
-Representacion del si mismo como impotente/ indefenso para recuperar el objeto perdido
-Afecto depresivo + Inhibicion psicomotriz
DISTINTAS CAUSAS QUE DESENCADENAN EN DEPRESION
1- EXPERIENCIAS TRAUMATICAS
Son situaciones de la realidad que colaboran a crear el sentimiento de indefension, impotencia y
desesperanza, como ser:
-la perdida temprana o abandono de las figuras parentales
-sometimiento prolongado a figuras patologicas/tiranicas
-enfermedades severa
-circunstancias en donde se cuestiona el sentimiento de valia o de identidad.
Estas situaciones pueden inscribirse en la mente como un sentimiento de que nada puede
hacerse para cambiar esta realidad- indefension/impotencia por parte del S. El sujeto pierde
potencia como agente activo de su vida.
2-IDENTIFICACION CON PADRES DEPRESIVOS
Depresion parental—> efectos en la infancia + ambiente triste que generan estos padres.
Mas alla de esto, puede haber una identificacion del hijo con este rasgo de caracter del padre.
Este niño deviene sujeto al identificarse con una/un madre/padre depresiva/o, esta identificación
es inconciente y se da de forma muy sutil con mensajes, dichos, fantasias, la manera de ver la
vida (gralmente frustrante y abrumadora para estas personas), formas de pensar de estos padres.
De esta manera la realidad del niño devendrá algo fuera de control o manejable y asi el niño se
plantara en la vida como agente de cambio de su ppia vida o como impotente ante el devenir del
destino siempre abrumador, injusto, negativo, etc.
Trasmision Intergeneracional en el origen de la patologia.
3-DEPRESION DEBIDO A UN TRASTORNO NARCISISTA PREVIO
Trastorno Narcisista, dos manifestaciones:
A-Los que tienen autoestima baja. Dependen de los objetos.
B-Los que no pueden vincularse con nadie, solo les importa el si mismo. Son omnipotentes.
Atacan y denigran a los objetos.
La depresion puede desencadenarse del tipo A de Trastorno narcisista…

106
-Directamente—>profundo y persistente sentimiento de impotencia, de ser incapaz de cumplir
metas y satisfacer sus deseos. La realidad se le presenta como abrumadora
-Indirectamente—> como consecuencia de las defensas empleadas para batallar este sentimiento
pobre de si mismo, por ejemplo encerrarse para no enfrentarse con la realidad abrumadora o con
la posibilidad de fracaso evita relacionarse con nadie, la persona renuncia a relacionarse con
otros, se pierde experiencias de aprendizaje, de conocer el mundo, de ampliar su circulo, se
encierra y se pierde la vida.
O del Tipo B de Trastorno Narcisista…
Cuando hay un colapso del sentimiento de omnipotencia luego de haber ignorado la realidad por
tanto tiempo, depresión por las consecuencias dañinas de su narcisismo en las relaciones
interpersonales, el autocuidado y la relacion/ajuste con la realidad.
4- AGRESION Y DEPRESION
No hay consenso en el campo del psicoanalisis sobre la relacion entre la depresion y la agresion.
-M Klein dice que la agresion es condicion necesaria para la depresion
-Otros postulan que lo central en la depresion es la baja autoestima producto de la repeticion de
situaciones donde el S se siente indefenso/impotente. La agresion puede o no estar presente.
-Agresion como defense ante la falla del objeto externo.
En la depresion la agresion esta dirigida a la respresentacion interna del objeto
AGRESION Y DETERIORO DEL OBJETO INTERNO
“Matarme para matarla dentro mio”
La denigracion psiquica del objeto conduce a su perdida como objeto valioso y estimulante y asi
el S queda vacio de objetos estimulantes El objeto denigrado se diferencia de los objetos
idealizados que son aquellos deseados por el sujeto al ser idealizados y estimulantes.
-Objeto por “posesion narcisista” es cualquier objeto cuyo valor recae sobre el valor del propio
Self. Quiero/admiro al objeto= quiero/valoro el Self.
Puede ser una cosa o una persona, el juicio de valor hecho sobre ese objeto sea positivo o
negativo es trasladado al propio sujeto por identificacion parcial.
-“Objeto de la actividad Narcisista” es el objeto que le permite al S realizar una actividad que le da
valor narcisista. Es el ajedrez y el oponente para un jugador de ajedrez, es el objeto sin el cual no
se podria llevar a cabo esa actividad que agrega valor narcisista al Self.
Trabajo, hobbie o profesion narcisisticamente valiosa para el Self.
Cuando una persona ataca sus objetos de actividad narcisista, estos quedan sin valor y por
consecuencia su vida se invade de un sentimiento de vacio, aquello que le daba valor y sentido a
su vida queda degradado, en consecuencia su vida tambien.
La depresion Narcisista y un inmenso sentimiento de culpa son el resultado de esta denigracion
del objeto de actividad narcisista.
4- ANSIEDADES PERSECUTORIAS

107
Las angustias persecutorias pueden derivar en depresion. Sea que estas angustias provengan de
personajes reales de quienes se viva una amenaza real o fantasías de peligros y persecución.
Estas derivan en depresion porque las defensas (evitacion, conductas masoquistas, renuncia a
logros, rituales obsesivos, etc) que se levantan para combatir estas ansiedades persecutorias
perturban el desarrollo del yo, las relaciones interpersonales y la relacion con la realidad limitando
las capacodades del sujeto
5-DEFICITS YOICOS
A veces el sentimiento de impotencia o incapacidad son productos de limitaciones reales del
yofalta de recursos yoicos y no es solamente un problema de la representacion del self.
Pueden ser deficits cognitivos, afectivos, relacionales, falta de control de impulsos y de la
ansiedad, problemas con la relacion con la realidad, etc que sirven para no caer en reacciones
defensivas.
Por ejemplo no se cuenta con el potencial yoico para superar un duelo y evitar que devenga en
duelo patologico donde se idealiza al objeto perdido (fijacion en el objeto primario) sin contar con
la capacidad de reemplazar ese objeto por otro, se idealiza al perdido como si no hubiese otro
objeto mas perfecto que este
6- INTERJUEGO ENTRE ≠ FACTORES
Se decribieron diferentes caminos que llevan a la depresion por distintas causas, con distintas defensas que
se ponen en marcha dando lugar a distintos tipos de depresion pero se pueden combinar ≠ factores que
determinen la depresion.
-Depresion como rdo de la conjuncion de distintos factores que se presentan simultaneamente.
-Depresion como rdo de una serie secuencial de factores, donde un factor provoca mecanismos
defensivos y consecuencias que a la vez activan otro factor o refuerzan otro, cadena que da lugar
a la depresion.
UTILIDAD DE DESARROLLAR UN MODELO INTEGRADOR DE LOS TRASTORNOS
DEPRESIVOS
Modelo integrado para entender y abordar los trastornos depresivos…
-Permite considerar aportes de ≠´s autores.
-Permite considerar a la depresion como un trastorno que tiene distintos destinos, no es una
categoria cerrada y es producto de diferentes condiciones internas al sujeto y externas a él.
Habra personas mas o menos vulnerables al desencadenamiento de la depresión como
respuesta a estas condiciones.
-Pensar la depresion en terminos de diagnostico dimensional, es decir describir tanto la
personalidad como la patologia como productos de la intersección de distintas dimensiones.
-Permite pensar intervenciones que sean mas apropiadas de acuerdo al subtipo de depresion,
ntervenciones psicoanaliticas pueden ser productivas para det subtipo y no para otro. Prestando
atencion al timing de la interpretacion y a la causa de la depresion, interpretar aquello que esta
directamente relacionado con el desarrollo del trastorno

108
-Jiménez, J. P. (2003). El tratamiento psicoterapéutico de la depresión: Una visión crítica
integrada. Aperturas Psicoanalíticas, 13. Recuperado de http://aperturas.org/articulos.php?
id=228&a=El-tratamiento-psicoterapeutico-de-ladepresion-una-vision-critica-integrada

Practica medica basada en la evidencia—> movimiento de los 90´s


Difusion de resultados medicos, se comparte el saber cientifico medico mas facil por internet para
mantener a la comunidad cientifica actualizada de nuevos hallazgos y resultados comprobados en
la practica clinica.
Sin embargo en la actualidad o bien no hay pruebas cientificas en las cuales basar la practica o si
las hay los profesionales clinicos no las usan. La experiencia clinica no es suficiente ya que puede
estar sesgada y el Rdo es que esos pacientes no estan recibiendo la mejor atencion disponible.
Esforzarse para estar al dia en los avances de la especialidad es un imperativo etico profesional.
La medicina basada en pruebas es la utilización a conciencia de las mejores pruebas disponibles
para tomar las decisiones sobre los cuidados de los pacientes
En este txt se toman las mejores pruebas disponibles sobre el tratamiento de la depresion
UNIPOLAR.
RESULTADOS DE LA INVESTIGACION EMPIRICA SISTEMATICA SOBRE EL TRATAMIENTO
DE LA DEPRESION
Evidencia del tratamiento Psicoterapeutico para la depresion unipolar.
-Si hay evidencia empirica de eficacia de un tratamiento hay que ser críticos porque esta
evidencia puede haber sido probada en un grupo pequeño de pacientes
-Si no hay evidencia no quiere decir que el tratamiento sea inutil, no es lo mismo que haya
ausencia de evidencia que afirmar que hay ausencia de beneficio.
-Se tienen en cuenta muchas variables para determinar que un tratamiento es efectivo, xej tipo y
magnitud de los beneficios de este tratamiento, tipo y magnitud de los daños del tratamiento, cual
es la evidencia de los beneficios y cual es la de los daños esto determina el grado de certeza de
los beneficios y daños esperados para ese tratamiento.
-La decision por un tratamiento implica que se analizen los beneficios y daños de ese tratamiento
y otras alternativas, se hara un balance entre estos.
-El curso de la depresion depende del subtipo (depresion mayor, distemia o depresion doble)
-Alto nivel de recaida, 75% en un lapso de 10 años sufrio recaida y 10% presentaron depresión
persistente (cronificacion)
-La reduccion en la tasa de recaida es un buen indicador del exito del tratamiento
CONCLUSIONES SOBRE EL TRATAMIENTO DE LA DEPRESION
-Para analizar el tratamiento psicoterapeutico en pacientes con depresion es necesario distinguir los efectos
de la psicoterapia en los ≠ tipos de pacientes—>los pacientes con depresion severa tienen peor respuesta a
las terapias psicologicas.

109
—>para muchos pacientes la depresion es un trastorno cronico y la probabilidad de recuperación
es baja.
-Los estudios de resultados muestran que las terapias mas eficaces para dar tratamiento a la
depresion son la terapia cognitivo conductual, la psicoterapia interpersonal y la terapia dinámica
exploratoria estructurada
-El tratamiento breve tiene efectos a corto plazo y pobres efectos en el largo plazo.
Lamentablemente no hay evidencia de tratamientos largos.
-El tratamiento farmacologico de larga duracion es el unico que muestra inequívocamente
efectividad en prevenir recaidas.
LAS PSICOTERAPIAS PARA EL TRATAMIENTO DE LA DEPRESION VALIDADAS POR LA
INVESTIGACION SISTEMATICA
Se han estudiado los resultados de la terapia cognitivo conductual, la psicoterapia interpersonal
(version modificada de la terapia psicodinamica y en menor medida los de la psicoterapia
psicodinamica.
1- TERAPIA COGNITIVO CONDUCTUAL
Teoria que surge de la teoria de condicionamiento clasica (aprendizaje condicionado: estimulo—>
respuesta condicionada) + teoria de aprendizaje social.
Para la terapia cognitiva, las cogniciones son aprendidas y mantenidas a través del reforzamiento.
Por eso las cogniciones patologicas se ‘atacan’ directamente a traves d ela terapia y no como
hacen las terapias psicodinamicas basandose en el levantamiento de sus determinantes
inconcientes.
Los vinculos entre los sintomas y las cogniciones son menos complejos que para el enfoque
psicodinamico
Aaron Beck, uno de los padres de la terapia cognitiva inicio sus desarrollos con los estudios de la
relacion entre pensamiento y depresion que dieron origen al tratamiento para la depresion y la
conducta suicida: Terapia breve, orientada a problemas, altamente estructurada y utiliza distintas
tecnicas como la escritura, calendario de actividades, biblioterapia, roleplaying, etc.
El objetivo es LOGRAR UNA MODIFICACION COGNITIVA dandole al paciente herramientas
practicas de autoayuda que pueda seguir aplicando una vez finalizada la terapia.
Pondera la relacion terapeutica como el escenario para lograr el cambio congnitivo pero sin tener
en cta los aspectos emocionales o de apoyo como factores curativos.
A los tenicos congnitivo conductuales no les importa tanto el origen de estas ideas y conductas
mal adaptadas, sino como se mantienen estos rasgos mal adaptados ya sea por interaccion con
el entorno o por el sistema de creencias del individuo.
Depresion= causada por pensamientos y sentimientos depresivos —> impactan la conducta del S
—>lo llevan a niveles de inactividad —> tienen consecuencias a nivel neuroquimico—> a la vez
exacerba los sentimientos y pensamientos depresivos
Los S depresivos tienen un impacto depresivo en quienes lo rodean generando rechazo social y
evitacion en los demas.

110
Es un gran circulo vicioso:
-El afecto actual sesga como se evalúa, procesa, almacena y recupera la información.
-La inactividad conductual exacerba el proceso depresivo.
-La conducta social contribuye también inadvertidamente al proceso de mantenimiento de la
depresion.
-La representacion pobre de si mismo, autoestima disminuida es un sesgo de confrmacion ya que
se centran en aquellos eventos que confirman esto mas que en los que lo desconfirman,
ampliando la autovaloracion negativa.
La terapia cognitivo conductual hace foco en estas creencias y pensamientos que reproducen la
conducta depresiva o disfuncional—>el objetivo de la intervencion cognitivo conductual es
cambiar estas creencias mal adaptadas con tecnicas terapeuticas (deteccion/identificacion y
desafio de creencias disfuncionales)
La interpretación de las razones de la existencia de estas creencias disfuncionales no es central
para la terapia cognitivo conductual como si lo es para la terapia dinamica o de tradición
psicoanalitica.
La TCC es superior a las terapias psicoanaliticas en cuanto al registro de resultados, en especial
para el estudio de la eficacia de sus tecnicas.
2-PSICOTERAPIA INTERPERSONAL(PIP)
Tratamiento breve de frecuencia de una sesión semanal, generalmente usado en la etapa aguda
de la depresión, donde se hace foco en la conexión entre el momento de aparicion de los
sintomas depresivos y los problemas interpersonales actuales. Se le da mas importancia a los
aspectos de las relaciones actuales que a los apectos cronicos de la personalidad.
1era fase de tratamiento:
En el diagnostico el terapeuta detecta y visibiliza como el funcionamiento actual del paciente, sus
relaciones sociales y sus expectativas en esas relaciones pueden haber sido causa de su
depresion. El terapeuta vincula los sintomas depresivos con alguna de las 4 areas
interpersonales:
-Duelo patologico
-conflicto de roles interpersonales
-transición de roles
-deficit interpersonales
2da fase de tratamiento:
El terapeuta plantea estrategias especificas para la problematica que desarrolla los síntomas
depresivos, si es un duelo patologico, estrategias para superarlo y crear nuevos vínculos
interpersonales, si es en torno a conflictos interpersonales explorando los problemas de la
relación y posibles opciones para resolverlos.
Fase final:

111
Se ayuda al paciente a ponderar los logros del tratamiento y a desarrollar estrategias para
detectar y enfrentar los sintomas depresivos en caso que reaparezcan en el futuro.
En la psicoterapia interpersonal el foco esta puesto en las relaciones sociales y en las acciones
interpersonales mientras que para la terapia psicoanalitica clasica el foco esta puesto en los
procesos mentales inconcientes y en el rol que estos deseos, fantasias y conflictos inconscientes
tienen en la determinacion de la conducta y la psicopatologia.
En ambas se da especial importancia a los sucesos de la temprana infancia pero la psicodinámica
pondera las relaciones de objeto mientras la interpersonales las relaciones interpersonales,
aunquelas relaciones de objeto son una inferenca hecha a partir de las relaciones
interpersonales.
3-TERAPIA PSICOANALITICA
Psicoanalisis Clasico= Terapia de tiempo ilimitado de frecuencia alta (3 a 5 sesiones semanales),
el objetivo no es remover síntomas o conductas problematicas sino reestructurar la personalidad.
Luego de la 2da guerra mundial —> Psicoterapia psicoanalitica= es focal y de menor intensidad y
frecuencia que el psicoanalisis clasico. El cambio surge por la necesidad de adaptase para que
pueda ser incluida en el servicio de salud mental publico.
Estos modelos focales no buscan la reestructuracion de la personalidad sino que se busca la
comprension cognitiva (insight) de los problemas personales para iniciar un cambio sintomatico.
Psicoterapia psicodinamica exploratoria de larga duracion= se puede extender por un año o mas
con una frecuencia de 2 a 3 sesiones por semana

Psicoterapia focal= dura entre 4 y 6 meses con una frecuencia de 1 a 2 sesiones por semana. Ambos tienen el
objetivo de resolver conflictos psiquicos inconcientes una se restringe a ciertos aspectos (la focal) mientras
la otra abarca un amplio espectro de conflictos. a ≠ de las TCC, se preocupan por ir mas alla del cambio
sintomatico.
Enfoque psicoanalitico integrado de Bleichmar para el tratamiento de la Depresion:
-Hay una fijacion en un deseo que ocupa un lugar central en la economia psiquica del S y no
puede ser reemplazado por el paciente deprimido. Es mas, su no realizacion y su imposibilidad de
ser reemplazado es la que conduce a una catastrofe psicologica.
Deseo que puede pertenecer a distintas areas:
* deseo de apego: de estar en contacto y comunicacion con el otro o mas patologico de fusion
con el objeto
* deseos narcisistas: de controlar impulsos y emociones, de ser querido y apreciado hasta los
mas patologicos de querer cumplir con ideales de perfeccion fisica, mental o moral o de tener
control total sobre si mismo o sobre otros.
* deseos relacionados con el bienestar de la persona querida, es decir que si la persona se ve
como agente de sufrimiento del otro se desarrollara una depresion cuyo componente principal
sera la culpa.

112
-Sentimiento de desamparo e impotencia por no poder cumplir con ese deseo + Representación
de si mismo como impotente e incapaz de cumplir ese deseo.
Ademas de este sentimiento de impotencia se pueden sumar=
- Agresion volcada contra si mismo por la culpa inconciente o por la agresion destinada a objetos
internos del Self
- Ataque a objetos externos con la consiguiente perdida de relaciones interpersonales y
oportunidades sociales.
- Super yo severo
- Aspiraciones narcisistas exageradas por la falta de correlación entre el yo percibido y el ideal del
yo
- pensamientos automaticos negativos
El core de la cuestion es encontrar junto con el paciente la via por medio de la cual éste llego a
deprimirse y esto nos llevara al foco dinamico donde nos centraremos para la terapia. Foco en
tantoconstructo teorico tecnico que tiene lugar en la mente del terapeuta y guia su practica. no es
algo impuesto por el terapeuta sino que surge de negociar entre él y el paciente.
EL PROBLEMA DE LA INDICACION DIFERENCIAL EN EL TRATAMIENTO DE LA DEPRESION
-HASTA DONDE SON TAN DIFERENTES LAS DISTINTAS PSICOTERAPIAS?
BULLSHIT- todas las tecnica susan algo d ela otra y tienen rdos positivos para el paciente.
-TERAPIA COGNITIVO CONDUCTIAL O PSICOTERAPIA DINAMICA?
Hay que tener en cta 3 niveles para definir la psicoterapia mas indicada segun el caso particular
-Nivel de sintomas: los trastornos con poco sintoma y bien delimitados tienen buen pronostico si
se tratan con TCC, cuando la problematica depresiva tiene caros componenetes de conflictos
inconcientes puede estar mejor indicada la psicoterapia dinamica focal
-Nivel de la personalidad: Cuanto mas comorbilidad haya (presencia simultanea de otros
trastornos) mas indicada esta la psicoterapia dinamica, mas que nada la exploratoria prolongada.
Esta es la indicacion para el tipo distimia
-Nivel de la motivacion para el tratamiento: La TCC es la indicada para aquellos pacientes que no
sufren tanto por sus sintomas y quieren cambios rapidos, obviamente deben estar bien
predispuestos a trabajar con sus terapeutas.
Cuando los pacientes quieren saber mas de si mismos, el tratamiento psicoanalitico es el correcto
porque permite integrar cuestiones particulares en contextos mas amplios y complejos en cambio
la potencialidad de la TCC consiste en traducir situaciones ocmplejas a mecanismos psicológicos
simples, cuando esta reduccion sea posible y adecuada entonces lo ideal sera la aplicacion de la
TCC.
Cuanto mas flexible sea un terapeuta menos distincion rigida entre orientaciones terapéuticas
habra porque podra integrar lo bueno de cada una en un mismo tratamiento.

PRÁCTICO SEMANA 7

113
American Psychiatric Association (2014). Trastornos de ansiedad (Subtipos: Trastorno de
ansiedad generalizada y Ataque de pánico). En Manual diagnóstico y estadístico de los trastornos
mentales (5ª ed., pp. 189–234). Arlington, VA: American Psychiatric Publishing.
En Teórico, semana 6
-Bleichmar, H. (2001). Distintos tipos de intervenciones psicoterapéuticas en los trastornos de
angustia. La concepción modular versus el principio de homogeneidad. Revista de la Sociedad
Argentina de Psicoanálisis, 4.

Concepcion Modular del psiquismo Vs. Ppio de Homogeneidad


Sistema Modular
-Sistema Complejo
-Para ser estudiado requiere la diseminacion en elementos mas pequenos pero entendiendo que
el funcionamiento total depende de la articulacion de estos subsistemas- Modulos
-Modulos-> leyes de funcionamiento y origenes ‚ + independientes y autonomos entre si, aun
cuando se articulan
->Sistema Narcisita
Sistema Sensual-Sexual
Sistema de la Hetero-Autoconservacion (cuidado del otro y de uno mismo)
Sistema de Apego
Sistema de las Defensas
Principio de la homogeneidad Organizacional
-Lo que pasa en un sistema (privilegiado sobre los demas) transforma lo que sucede en los otros
-Todos los sistemas obedecen a las mismas leyes de organizacion, son una misma cosa
-Regulacion homogenea entre cuerpo y mente, si algo no funciona en uno, se expresa en el otro x
ej. Alexitimia, la persona no puede leer emociones en los otros ni expresar las ppias, como
resultado presenta enfermedades psicosomaticas.> Incapacidad de simbolizacion/enfermedad
psicosomatica. Regulacion Homogenea psiquis/cuerpo
Sin embargo, la practica muestra que personas sin problemas de simbolizacon tambien presentan
enfermedades psicosomaticas.> la clinica muestra que las cosas son mas complejas.> MENTE Y
CUERPO SE ARTICULAN COMO DOS MODULOS DIFERENTES, COMPLEJOS,
INDEPENDIENTES Y AUTONOMOS.
Con esta informacion se encara el estudio de los Trastornos de Angustia, en especia el
TRASTORNO DE PANICO
Tres momentos:
1- Fuentes de Angustia

114
2- Reaccion del Cuerpo frente a la angustia
3- Codificacion conciente e inconciente que se realiza sobre lo que sucede- Representacion de la
Angustia y sus efectos en el cuerpo y en la mente en la crisis de panico
FUENTES DE ANGUSTIA:
El Yo lucha con el Ello, S Yo y Realidad.> de ahi las causas de angustia.
+ de angustia-> fenomenos neurobiologicos y corporales (caracteristicos de la crisis de angustia) -
Fenomenos cardiovasculares
- Sensacion de ahogo
- Opresion en el pecho
- Mareo
- Etc
Crisis de Panico
1- Expresion en el cuerpo del malestar psicologico- estado de angustia.
Hay cierta predisposicion biologica a reaccionar con Ataque de Panico frente a la fuente de
angustia, no se trata de una pobre capacidad de simbolizacion de un conflicto psiquico (fuente de
angustia), otras personas responden con un ataque de asma, diabetes, ulcera, dermopatia, etc.
Interrelacion Mente/cuerpo (ambos Modulos Autonomos INTERACTUAN) = predisposición
biologica + angustia = Crisis de Panico.
2- Vuelta al psiquismo con la Codificacion/ Representacion de lo que esta pasando.
Una vez producida la descarga corporal, la mente decodifica, le da un significado a lo que le
ocurre en el cuerpo
Tres Areas de Intervencion Posible:
1-Representacion q produce angustia
2-Reaccion Corporal
3-Representacion de lo que sucede en el cuerpo
1.
Psicoanalisis.> Trabajan con las fuentes de angustia. - Conflicto / - Descarga Corporal
Si se analizan los conflictos - la angustia .> asi el paciente mejora su afeccion psicosomatica.
Obvio esto no impide recaidas en el futuro A veces estas recaidas se intensifican en el fin del
analisis, no es casual.> el S se siente desprotegido, “Self en Peligro” (El divan no es
recomendado, los silencios tampoco- todo lo que aumente su sensacion de desproteccion y
ausencia de control.> aumenta su angustia)
No se recomienda ninguna intervencion que aumente su sensacion de falta de control, por
ejemplo comentarios sobre lo oculto, lo icce, lo que esta detras. El paciente tiene que estar en
control, tener un rol activo en la terapia, no pasivizarlo.

115
Dependiendo la estructura de la personalidad donde se inscribe la Crisis de Panico, los aspectos
formales de la tecnica, encuadre, etc que usaremos para dar tratamiento. x ej una persona afin al
apego puede interpretar el uso del divan como dejarse cuidar, o entregarse a los brazos de otro q
viene a calmar mi angustia
Trabajo sobre lo imaginario: Reconstruir una imagen de Self en control ≠‚ Self en peligro Pensar
con cada intervencion, esto fomenta su imagen de self en control o lo deja desprotegido, falto de
control?
Se recomienda el uso de psicofarmacos (al igual que para las crisis de asma, diabetes, entre
otras enfermedades psicosomaticas)
2.
Reaccion en el cuerpo del Ataque de Panico es una Reaccion adaptativa normal frente al peligro:
Taquicardia- aumento del ritmo cardiaco- preparacion para la accion/huida
Hiperventilacion- conseguir mas oxigeno para estar en mejores condiciones fisicas para atacar o
huir
Hay que aclararle al paciente que la suya es una reaccion normal pero aumentada y desajustada
(la situacion no amerita que su cuerpo responda de esta manera) debido a cierta “Sensibilidad
especial de el”
Esto le ayuda a entender que su cuerpo no enloquecio. El Ataque de Panico deviene una
Reaccion gguardadah y cada vez se necesitan estimulos menos intensos para generarla..->
WARNING!! USAR PSICOFARMACOS PARA EVITAR LA SENSIBILIZACION CRECIENTE +
PSICOTERAPIA para completar el circuito
La teoria Freudiana de la libido estancada= ataque de panico responde al Modelo
Homogeneizador de la mente donde lo que no funciona en un sistema impacta y determina
linealmente lo que sucede en otro: libido estancada (psi/mente) .> ataque de panico (cuerpo). Un
sistema compensa a otro.
Esto no es adecuado, hay gente que pasa por altos niveles de angustia y esto no desencadena
un ataque de panico ni otra enfermedad psicosomatica.> es + acertado pensar que estos
pacientes tienen una sensibilidad y un cuerpo que reacciona de forma ≠‚ Aqui el sistema modular
(psi-mente) provoca efectos en otros sistemas modulares (cuerpo), pero
estas reacciones siguen leyes propias del sistema, no son la misma cosa.
3.
Dar una significacion ≠‚ a la experiencia
Reinscripcion de la experiencia o reactivar otras experiencias que empujen a las experiencias
patologicas a un segundo plano.
Mente= Sistema de Representaciones y creacion de significados ctes e icctes
Cuerpo= Es captado/ representado por la mente pero tiene existencia independiente de ella.

116
-Sue, D; Sue,D W y Sue, St (2010). Trastornos de Ansiedad. En Psicopatología. Comprendiendo
la conducta anormal. (pp 119-123). México: Cengage Learning
Trastornos de ansiedad desde una perspectiva de vías múltiples
Un número de diferentes factores puede desempeñar un papel en la adquisición de miedos o
fobias, incluyendo factores biológicos como vulnerabilidades con base genética y factores
psicológicos como variables de la personalidad, una sensación de control o dominio del individuo,
o experiencias de condicionamiento directo o indirecto. Además, las reglas y normas culturales
también pueden influir en la expresión de los trastornos de ansiedad.
Dimension biologica
Dos factores biológicos principales afectan los trastornos de ansiedad; la estructura cerebral y las
influencias genéticas.
- Estructura cerebral: La amígdala (parte del cerebro que interviene en la formación y
memoria de los eventos emocionales) desempeñan una función central en los trastornos
de ansiedad. Cuando se presenta una amenaza, alerta a otras estructuras cerebrales
como el hipocampo y la corteza prefrontal, disparando una respuesta de miedo o ansiedad.
Hay dos vías hacia la amígdala. Una va directamente desde el estímulo sensorial temido a
la amígdala en tan sólo milésimas de segundo. La segunda y más lenta, viaja primero
desde la corteza prefrontal que evalúa el estímulo y puede llevar a cabo la respuesta inicial
de miedo.
La utilización de las técnicas de neuroimagen como la tomografía por emisión de positrones
(TEP) y la imagen por resonancia magnética (IRM) nos permiten entender el circuito cerebral que
participa en los trastornos de ansiedad y medir estructuras características de los sistemas
cerebrales involucrados. Con la neuroimagen podemos determinar qué partes del cerebro se
activan o no se activan cuando un individuo es expuesto a un estímulo atemorizante y comparar
la actividad cerebral de las personas con trastornos de ansiedad o sin ellos. La reactividad
incrementada en la amígdala se ha observado en pacientes con trastornos de ansiedad cuando
se les expone a estímulos emocionales específicos. Además, es posible ver diferencias en el
metabolismo de ciertas áreas del cerebro en individuos con trastornos de ansiedad incluso
cuando no hay estímulos atemorizantes. Las técnicas de neuroimagen también nos permiten
observar los efectos de la medicación y la psicoterapia. La medicación parece “normalizar” los
circuitos de ansiedad en el cerebro. Las psicoterapias también producen cambios neurobiológicos
similares a aquellos que se observan en la medicación para los trastornos de ansiedad.
- influencias genéticas: Los genes parecen aportar una contribución modesta a los
trastornos de ansiedad. Estudios previos han intentado estimar la heredabilidad de las
características de ansiedad al examinar familias. Es probable que muchos genes e
interacciones entre ellos contribuyan a la ansiedad. Incluso entonces, estos genes pueden
indicar sólo una predisposición al desarrollo de la ansiedad. No garantizan que las
características se expresarán; esto depende de las interacciones con otros genes y con el
ambiente. La búsqueda de los factores biológicos incluye la identificación de genes e
interacciones genéticas, el entendimiento de la relación de los diferentes sistemas
cerebrales y la determinación de cuáles combinaciones de neurotransmisores están
involucradas en los trastornos de ansiedad.

117
- Interacciones biológicas, psicológicas y sociales: Características como ansiedad y miedo
pueden resultar de influencias genéticas y ambientales y son codificadas por sistemas
neurales en el cerebro durante un periodo crítico del desarrollo.
ejemplo del texto: Un niño que tiene una predisposición genética a ser miedoso puede
estar expuesto a conductas de su cuidador que provoquen ansiedad o que no le den apoyo
cuando la experimenta; entonces se refuerza el circuito neural subyacente asociado con la
ansiedad. Sin embargo, si el niño se desarrolla en un ambiente de apoyo que promueva la
interacción con otros, las conductas ansiosas no emergerá.
La interacción gen-ambiente ha recibido recientemente una gran atención y se muestra
más promisoria que los enfoques de modelo simple. Sin embargo, otras variables también
intervienen. No todos los niños con el alelo corto desarrollan inhibición conductual, incluso
en ambientes familiares estresantes, y algunos con alelos largos son inhibidos. Este hecho
apoya la idea de que hay genes o combinaciones de genes específicos que predisponen a
un individuo a desarrollar características conductuales.
Dimension psicologica
Las variables psicológicas como la sensación propia de control también pueden estar
involucradas en el desarrollo de los trastornos de ansiedad. Las experiencias tempranas pueden
desempeñar una función en la determinación de la vulnerabilidad de los niños y deben
considerarse en las teorías cognitivas.
Los teóricos psicoanalistas se enfocan principalmente en la importancia de la relación padres-
hijos en el desarrollo de los trastornos de ansiedad. En este sentido, los trastornos de ansiedad
se consideran manifestaciones de las experiencias de una infancia problemática.Los teóricos
cognitivo-conductuales enfatizan la importancia de los procesos cognitivos en el desarrollo y
mantenimiento de los trastornos de ansiedad, debido a que la atención del individuo se enfoca
demasiado en un estímulo amenazante y a que los pensamientos catastróficos o irracionales
refuerzan el miedo y la ansiedad.
Dimensiones sociales y culturales
El estrés ambiental diario es un factor social que puede producir ansiedad, especialmente en
individuos vulnerables desde los puntos de vista biológico o psicológico. Aquellos que tienen
ingresos más bajos presentan mayores tasas de trastornos de ansiedad y tragedias como
aquellas relacionadas con ataques terroristas, tiroteos en instituciones educativas y desastres
naturales también incrementan la tasa de trastornos de ansiedad en la población. El género
también desempeña un papel en el desarrollo de los trastornos de ansiedad. Con excepción del
trastorno obsesivo-compulsivo, las mujeres son más propensas a sufrir trastornos de ansiedad
que los hombres debido a la falta de poder y estatus que mantienen en la sociedad, y al estrés
crónico asociado con la pobreza, la falta de respeto y las decisiones limitadas. Las mujeres
también están más expuestas a los traumas que los hombres; las hormonas del estrés producidas
por estos eventos pueden hacer a las mujeres más vulnerables a la depresión y la ansiedad.
Los factores culturales como los problemas de aculturación también contribuyen a los trastornos
de ansiedad entre las minorías étnicas. El incremento de ansiedad entre algunos grupos
culturales quizá se deba a estándares culturales que difieren en términos de interacciones
sociales apropiadas. En otros casos, puede ser el resultado de la exposición a discriminación y
prejuicio

118
HASTA ACA PRIMER PARCIAL – SE TOMA EN SEMANA 8

119

También podría gustarte